You are on page 1of 56

The Residents Guide to the LMCC II

3rd Edition
The Licentiate of the Medical Council of Canada Exam, part II, also known as the MCCQE II, was the traditional means of qualifyin for a eneral license to practice medicine in Canada! "ow that #oth the internship year and the eneral license are no lon er a$aila#le, many residents $iew the exam as a stressful and expensi$e exercise in futility! %hile the process is stressful and expensi$e, it need not #e futile! &reparation for the exam can #e an enli htenin re$iew! 'cenarios tend to repeat o$er the years, the pass rate is reater than ()* on the first attempt, and there is an option to rewrite, so don+t panic! The exam is an ,'CE -,#ser$ed 'cenario Clinical Exam. in which the candidate pro resses throu h a series of stations! /our startin point is determined alpha#etically! 0t each station there is a physician examiner and either a real person posin as a patient or a telephone o$er which you must speak to a patient or another physician requestin assistance! The most recent sessions -since 1((2. contain six short cases known as )3minute couplets, in which the candidate is allotted ) minutes to assess a patient and ) minutes to write short answers to questions related to the case! There was also a series of six lon er cases in which the candidates were presented with a more in$ol$ed clinical pro#lem, such as a resuscitation or psychosocial counselin session, lastin 14 minutes each! The physician examiner may ask one or two questions in the last minute of a 143minute station! There is one minute #etween stations durin which you can look at a #rief description of the patient and consider your approach! ,ccasionally 5pilot6 questions will #e included in the exam, which will not count towards the final mark #ut are used to test new questions! /ou will not know which questions are 5pilot6 questions! The content of the exam is eneral medicine! This means family practice 7 emer ency medicine! The followin topics appear consistently8 Pediatrics 9 diarrhea, de$elopment, neonatal :aundice, asthma Obs/Gyn 9 amenorrhea, $a inal #lood, a#dominal pain, &I;, ,C&, electi$e a#ortion counselin Suturing 9 choice of suture, tetanus $accine Chest Pain 9 read C<=, EC> Resuscitation 9 fluid resuscitation after #lood loss, 0?C@s Overdose 9 0'0, TC0 Needle stic 9 0I@', hepatitis, $accinations Psychiatry 9 depression, mania, schiAophrenia Neurosurgery 9 #ack and neck radiculopathies, carpal tunnel -"ote that e$ery history should include name, a e, occupation, past medical history, family history, medications, dru sBalcohol, re$iew of systems.

!""#
!$ %irst year university student& " 'ee s (regnant& considering abortion$ Ta e a history and counsel$ %indings) tear*ul& guilty& slee( disturbance& has not engaged social su((orts$ +istory8 com#ine a pre nancy history with a social history and a screen for depression! Pregnancy8 &atient I@ -name, a e, occupation.! >T&0L -num#er of estations, term pre nancies, premature #irths, a#ortions, li$e children., history of pro#lems, if any, with pre$ious pre nancies! Current pre nancy, esta#lish estational a e ->0. #y last menstrual period -LM&. if re ular periods and sure dates -if unsure a datin ultrasound would #e needed.! The >0 is the num#er of weeks from the first day of the LM&! The E@C is first day of LM& C 2 days 9 3 months! 0sk a#out use of alcohol, smokin , dru s, domestic $iolence -)4* #e ins in pre nancy., maternal illnesses durin the pre nancy -particularly dia#etes, ru#ella, toxoplasmosis, herpes, CMD, thyroid dysfunction, ;T", hypercoa ulation.! Ese of #irth control, if any! &ast medical history, family history of pre nancy related pro#lems, medications! Social8 'tatus of any relationships at present includin relationship with the child+s father! 'ocial supports -family, friends, #oyfriend., do they knowF 0re they helpin F EmploymentBfinancialBeducational status of the patient, does the patient feel prepared to raise a childF

Psychiatric8 ;ow does the patient feel a#out this decisionF ;ow is she copin F Co$er mnemonic for ma:or depression! M'I>EC0&'8 mood -depressed., sleep -increased or decreasedGif decreased, often early mornin awakenin ., interest -decreased., uiltBworthlessness, ener y -decreased or fati ued., concentrationBdifficulty makin decisions, appetite andBor wei ht increase or decrease, psychomotor acti$ity -increased or decreased., suicidal ideation 9 positi$e dia nosis of ma:or depression requires fi$e of these o$er a H week period, one of the fi$e must #e loss of interest or depressed mood! 'ymptoms do not meet criteria for mixed episode, si nificant socialBoccupational impairment, exclude su#stance or >MC, not #erea$ement! Counseling8 Make empathetic statements, e! ! 5This must #e $ery hard for you!6 +ealth 'hile (regnant8 recommend a#stinence from harmful a ents -alcohol, smokin , dru s. while pre nant and use of medications only after consultin with a physician, treatment for pre nancy3related illnesses as a#o$e, and healthy eatin ha#its! Social su((orts8 @iscuss the importance of en a in social supports, and consider a $isit with #oth the patient and her partner or other supportin person! ,bortion8 &ro$ide information on local a#ortion ser$ices! Make the patient aware that the estational a e limit after which many practitioners will not perform an electi$e a#ortion in Canada is H4 weeks, #ut that this is a late limit and her decision should #e made sooner, #efore 1I weeks would #e #est! Inform the patient that further ad$ice is a$aila#le from pri$ate ynecolo ists who perform a#ortions and counselors at electi$e a#ortion centers! ,ffer to refer the patient if she wishes! -e(ression .anage.ent8 "ormaliAe the patient+s depressed mood in $iew of her circumstances! If there is e$idence of ma:or clinical depression, arran e close follow up to monitor for suicidal ideation, refer to psychiatry! @o not prescri#e medications at this time -#ecause of the pre nancy.! /$ /0 year old *e.ale 'ants an oral contrace(tive$ Ta e a history and counsel$ +istory8 "ame, a e occupationBschool le$el! %hy does patient want an ,C&F ;as she #een on it #efore or other forms of contraceptionF If so, why was it stoppedF ;ow lon has the patient #een sexually acti$eF ;ow many partnersF Current contraception used! Is there a possi#ility that the patient could #e pre nantF ,#tain the date of last menstrual period! Pregnancy history8 >T&0L -num#er of estations, term pre nancies, premature #irths, a#ortions, li$e children., history of pro#lems, if any, with pre$ious pre nancies! Gynecological history8 0sk a#out sexually transmitted disease -'T@s., &I@, mi raine, fi#roids, dia#etes, throm#oem#olic disease, heart pro#lems, cancer, li$er disease! @ate of last &ap smear, history of a#normal &ap smear and follow3upBtreatmentF %hen did the patient start menstruatin F Menstrual history8 re ularity and len th of cycle and duration of periods, hea$iness of flow -num#er of pads required., crampin , associated discomfortBpain, #loatin , mood swin s -&M'.! Medications, dru s, alcohol, smokin , past medical history -especially #reast cancer., family history, re$iew of systems! Counseling8 Contraindications to OCP8 current pre nancy, undia nosed $a inal #leedin , acti$e cardio$ascularBthrom#oem#olic diseases -includes coronary and carotid disease, symptomatic mitral $al$e prolapse, cere#ro$ascular disease, moderate3se$ere ;T", acti$e @DT., proliferati$e retinopathy, history of #reast cancer or other estro en dependent tumors -li$er, #reast, uterus., impaired li$er function -o#structi$e :aundice in pre nancy., con enital hyperlipidemia, a e J 3) and smokin , %ilson+s disease! Relative contraindications to OCP8 smoker J 3) years old, dia#etes, mi raines, fi#roids! Mechanis. o* action o* OCP8 standard preparations contains estro en and pro esterone or :ust pro esterone, pre$ents o$ulation #y interferin with feed#ack of hormone si nalin , atrophic endometrium, chan e in cer$ical mucous -mucous plu Gthou ht to #e due to pro esterone component.! ,vailable (re(arations8 H1 day $s! HK day ta#lets -2 place#o days.! ,ther preparations8 @epo in:ections q3m -@epo3&ro$era 9 medroxypro esterone, restoration of fertility may take up to 13H years, irre ular menstrual #leedin ., implants q)y -"orplant 9 le$onor estrel, six capsules inserted su#dermally in arm, irre ular menstrual #leedin .! Lon er term preparations offer lower cost o$er the duration of action -#ut reater one3time cost. and reater con$enience! 1ene*its o* all the hor.onal contrace(tives8 0?C@Es8 ,nemia reduced, often clears ,cneL 1eni n #reast disease and cysts decreasedL Cancer -o$arian decreased., Cycles re ulated, Increased Cer$ical mucous which reduces 'T@sL -ysmenorrhea

decreased, decreases 2ctopic pre nancy rates and of course8 $irtually no chance of pre nancy when taken as directed -(K3 ((!)*.! Ris s o* hor.onal contrace(tives8 sli ht wei ht ain is usual -) l#s., increases risk of @DT especially in com#ination with smokin , may stimulate estro en3receptor positi$e #reast cancers, #ut does not appear to cause them, may ha$e to try two or three different preparations to arri$e at the one for the patient! 0lso note that hormonal contracepti$es do not pro$ide as much protection a ainst sexually transmitted diseases, compared to #arrier methods! -irections8 'tart ,C& on the first day of the next menstrual period! &lace packa e in an o#$ious location to help you to remem#er! Take at the same approximate time each day! Ese additional contraception for the first two months, as ,C& contraception is not relia#le until then! If you miss a day, take two pills the next! If you miss two days, take two pills for the next two days and use an alternati$e method until the next period! >i$e prescription for ,C& of choiceGany family mem#ers -sistersBmother. on ,C&F %hat works for themF 0rran e follow up! 3$ !4 year old boy 'ith e(ile(sy docu.ented by neurologist& co.es to you because he does not 'ant to see his (arents *a.ily doctor$ 5ants a drivers license$ Ta e a history and counsel$ +istory o* sei6ure disorder8 &atient I@! 0 e of onset -primary eneraliAed rarely #e in M 3 or J H4 years old.! &recipitants8 'leep depri$ation, dru s, Et,;, TD screen, stro#e, emotional upset! @escri#e seiAures -Nacksonian marchF 'ali$ation, cyanosis, ton ue #itin , incontinence, automatisms, motor $s! $isualB ustatoryBolfactory., frequency, duration, what #ody parts affected and in what order -motor 9 frontal lo#e, $isualBolfactoryB ustatory hallucinations O temporal lo#e., promontory si ns -presence of aura8 implies focal attack., post3ictal state -decrease in le$el of consciousness, headache, sensory phenomena, ton ue soreness, lim# pains, Todd+s paralysis 3 hemiple ia., de ree of control achie$ed with medications, at what dose and for how lon , corro#oration from family if possi#le! %as a CT scan done when seiAures were first dia nosedF "um#er and description of recent seiAures, are they different from pre$ious seiAuresF Is the patient ha$in any new symptoms such as headache, mornin $omitin , new neurolo ical deficits! If the dru worked in the past why does the patient #elie$e it isn+t workin nowF 'ide effects of antiepileptics8 drowsiness, poor concentration, poor performance in school, ataxias, peripheral neuropathy, acne, nysta mus, dysarthria, hypertrichosis -excessi$e hairiness., in i$al hypertrophy -phenytoin.! Medications, dru s and alcohol, smokin , aller ies, past medical history, family history, re$iew of systems! Co.(liance8 Is the patient takin medsF %hy notF &ro#lems at school or homeF 0sk a#out relationship pro#lems! @epression screen as in P1( a#o$e! 'ocial supports! Physical e7a.8 neurolo ic exam includin mini mental, cranial ner$es, #ulk, tone, power, sensation, cere#ellar exam, deep tendon reflexes! Treat.ent8 @iscuss importance of compliance with medication and a$oidin dan erous acti$ities such as dri$in until ood control is achie$ed! Ministry of Transportation re ulations require 1 year seiAure free #efore they will rant a dri$er+s license in Canada! Inform the M,T of the patient+s seiAure disorder if you ha$e not already done so and inform the patient that this is required #y law! If alcohol is an issue, inform the patient that chronic alcohol intake may decrease #lood le$els of antiepileptics -$ia increased li$er meta#olism., and excess alcohol intake can precipitate seiAures #y lowerin the seiAure threshold there#y precipitatin a seiAure! It is enerally recommended that the patient not drink at all! Qati ue and concomitant illness can also lower seiAure threshold! The patient should consult a physician #efore takin other medications, as they may also lower the seiAure threshold! The same is also true of sedati$es, cocaine, amphetamines and insulin! Qati ue and other illnesses can also lower seiAure threshold, in addition to $arious other medications! If patient is ha$in stress mana ement, anxiety issues, he may require further counselin ! ,utline a treatment plan consistin of8 EE>, CT head, meta#olic screen, medications -if not done already., and follow up appointments! >et the parents in$ol$ed if possi#le! 'end #lood for serum @ilantin -phenytoin. le$els if patient is on this already! If @ilantin le$els are therapeutic, #ut the patient is ha$in se$ere side effects or poor seiAure control, a second dru may #e added -usually car#amaAepine or $alproic acid.! @iscuss what to do in the e$ent of seiAure, counsel parents if possi#le! ?ystanders are not to insert o#:ects into the patient+s mouth! Turn patient on his side while seiAin ! Call am#ulance or take to Emer ency if seiAure doesn+t stop in ) minutes! 0rran e re ular follow up to monitor pro ress and serum @ilantin le$els!

Indications and i.(ortant side8e**ects o* .a9or antie(ile(tic drugs -rug Indication -ose8related Carba.a6e(ine &artial or eneraliAed @iplopia, diAAiness, headache, nausea, :Tegretol; tonic3clonic seiAures drowsiness, neutropenia, hyponatremia Phenytoin :-ilantin; <al(roate :2(ival& -a(a ene; 2thosu7i.ide :=arontin; &artial or eneraliAed tonic3clonic seiAures, status epilepticus 0ll eneraliAed seiAures or partial seiAures 0#sence seiAures "ysta mus, ataxia, nausea, $omitin , in i$al hyperplasia, depression, drowsiness, paradoxical increase in seiAures, me alo#lastic anemia Tremor, wei ht ain, dyspepsia, nausea, $omitin , alopecia, peripheral edema "ausea, anorexia, $omitin , a itation, drowsiness, headache, lethar y

Idiosyncratic Mor#illiform rash, a ranulocytosis, aplastic anemia, hepatotoxic effects, 'te$ens3Nohnson, terato enicity 0cne, coarse facies, hirsutism, #lood dyscrasias, Lupus3like syndrome, rash, 'te$ens3Nohnson, @upuytren+s, hepatotoxic effects, terato enicity 0cute pancreatitis, hepatotoxic effects, throm#ocytopenia, encephalopathy, terato enicity =ash, erythema multiforme, 'te$ens3 Nohnson, Lupus3like syndrome, a ranulocytosis, aplastic anemia

>$ >? year old 'o.an co.(lains o* @diseased sto.ach$A +as had negative investigations by several other doctors$ Ta e a history and (er*or. a .ental status e7a.ination$ B) 5ithout loo ing at the (atient again& describe her a((earance$ 5hat is your diagnosisC +istory *or de(ression8 I@ -name, a e, employment status, marital status, li$in arran ements., chief complaint, ;&I8 M'I>EC0&' 9 mood -depressed., anxiety, diurnal $ariation in mood and acti$ity with nadir in the early mornin , irrita#ility, chan e in sleep pattern -increased or decreased, if decreased usually mornin awakenin ., interest -decreased., anhedonia -ina#ility to en:oy pre$iously en:oya#le acti$ities., uilt, hopelessness, worthlessness, ener y -fati ue., concentration, memory, difficulty makin decisions, appetite and wei ht ain or loss, psychomotor retardationBa itation, suicidal ideation! 0lso inquire a#out past psychiatricBmedicalBsur ical history, aller ies, meds, family history, personal history, re$iew of systems! Suicidal ideation8 does patient intend to harm self, reason for suicidal thou hts, current plan, lethality of plan, access to lethal means, has patient i$en away priAe possessions or written final notes to lo$ed ones, pre$ious attempts! -iagnosis o* .a9or de(ression8 Mnemonic for ma:or depression 9 M'I>EC0&'8 mood, sleep, interest, uilt, ener y, concentration, appetite, psychomotor, suicide, positi$e dia nosis of ma:or depression requires fi$e of these o$er a H week period! ,ne of the fi$e must #e either loss of interest or depressed mood! Cannot #e a mixed episode and symptoms must cause socialBoccupational impairment! Cannot #e >MC or su#stance a#use or #erea$ement! -0 dia nosis of depression cannot #e made in the face of #erea$ement within the past two months or dru or alcohol a#use!. -ysthy.ia8 0 dia nosis of dysthymia requires depressed mood for most of the day, more days than not, for at least two years! &resence while depressed of at least H of8 poor appetite or o$ereatin , insomnia or hypersomnia, low ener yBfati ue, low self esteem, poor concentration or difficulty makin decisions, hopelessness! "e$er without depressed mood for more than H months at a time, no M@E, manic, mixed or hypomanic episodes in past, not >MC or su#stance a#use! Manic e(isode8 Expansi$e, ele$ated or irrita#le mood x 1 week with 3 of followin 8 >'T&0I@ 9 randiosity -or inflated self esteem., sleep -less need for., talkati$e, pleasura#le acti$ities -with painful consequences., acti$ity increased - oal directed or psychomotor., ideas -fli ht of., distracti#ility! "ot mixed episode! 'e$ere enou h to cause psychotic featuresBimpaired socialBoccupational functionin ! "ot su#stance a#use or >MC! -i**erential *or de(ression8 Check for #ipolar mood disorder -manic3depressi$e., schiAophrenia, psychotic depression and o#sessi$e3compulsi$e disorder! 0sk a#out manic episodes, paranoia, hallucinations -esp! $oices., o#sessi$e thou hts, pre$ious psychiatric pro#lems, family history of psychiatric disorders, su#stance a#use, relationship pro#lems, pro#lems at work -#asically a mental status exam.! Medical causes o* de(ression8 ask a#out hypothyroidism, adrenal dysfunction, hypercalcemia, mononucleosis! Consider chronic fati ue syndrome! @ru use, smokin , aller ies, past medical history includin psychiatric history and history of a#use! Qamily history, re$iew of systems! Mental status8 appearance, #eha$ior -dress, roomin , posture, ait, apparent a e, physical health, #ody ha#itus, expressions, attitude 3 cooperati$eF, psychomotor acti$ity, attention, eye contact., speech -rate, rhythmBfluency, $olume, tone, quantity, spontaneity, articulation., mood -su#:ecti$e emotional state in patient+s own words., affect -Quality 9 euthymic, depressed, ele$ated, anxiousL =an e 9 full, restrictedL 'ta#ility 9 fixed, la#ileL 0ppropriatenessL Intensity 3 flat, #lunted., suicidal ideation

-low, intermediate, hi h 9 poor correlation #etween clinical impression of suicide risk and pro#a#ility of attempt., thou ht process -coherent, fli ht of ideas, tan entiality, circumstantiality, thou ht #lockin , neolo isms, clan in , perse$eration, word salad, echolalia., thou ht content -delusions 9 #iAarre $s! non3#iAarre, o#sessions, preoccupations, pho#ias, recurrent themes., perceptual distur#ances -illusions, hallucinations, depersonaliAation, derealiAation., insi ht, co nition, :ud ment! Multia7ial Su..ary8 0xis I 9 clinical disorders 9 @'M IDL differential dia nosis 0xis II 9 personality disorders 9 @'M IDL mental retardation 0xis III 9 >MC -as they pertain to 0xis I and other 0xes. 0xis ID 9 psychosocial and en$ironmental pro#lems 0xis D 9 lo#al assessment of functionin ->0Q. 9 43144 Mini8Mental Status8 orientation to timeBplace -) pts, year, season, month, day, day of weekL ) pts, country, pro$ince, city, hospital, floor., memory -3 pts, honesty, tulip, #lackL 3 pts, delayed recall., attentionBconcentration -) pts, serial 2s, 5%,=L@6 #ackwards., lan ua e tests8 comprehension -3 pts, three point command., readin -1 pt, 5close your eyes6., writin -1 pt, complete sentence., repetition -1 pt, 5no ifs, ands or #uts6., namin -H pts, watch, pen., spatial a#ility -1 pt, intersectin penta ons. Patients a((earance8 slo$enly -iagnosis8 ma:or depression Treat.ent o* .a9or de(ression8 pharmacotherapy, psychotherapy, family therapy! 'tart fluoxetine -proAac. H4 m q0M, may increase to R4 m q0M after 1 week! Takes H3R weeks to work! Explain side effects of sleep distur#ance, anor asmia, nausea -use ''=I summary #elow.! ,d.it8 when patient is acti$ely suicidal, i!e! plans are specific, patient has i$en away possessions, written final notes to lo$ed ones! Qor suicidal ideation without current intent, dischar e only with written contract with the patient to inform you immediately if they feel likely to make an attempt! >i$e num#er to call and arran e re ular follow up #efore patient lea$es the office! SSRI antide(ressant su..ary Qluoxetine -&roAac. 9 enerally ener iAin Qlu$oxamine -Lu$ox. 9 more sedatin 'ertroline -Soloft. 9 used in the elderly &aroxetine -&axil. 9 used in mixed anxietyBdepress Start at) :.g O-; H4 )4 )4 143H4 Thera(eutic) :.g O-; H43K4 1)43344 )431)4 H43I4

D$ 40 year old 'o.an 'ith acute con*usion$ Per*or. a *ocused (hysical e7a. e7cluding .ental status$ Neurologic e7a.8 what follows is a practical, re ionally or aniAed neurolo ic exam which can #e completed in less than ei ht minutes -with practice.! It #e ins with the patient sittin , then standin , then lyin down! "ote that e$ery physical exam should include $itals, althou h in this case the examiner will ask you to mo$e on! &atient sittin -shoes and socks remo$ed.8 GCS/MMS2 9 only if patient poorly responsi$e! The examiner will remind you to omit the mini mental status exam! Glasgo' Co.a Scale 2ye O(ening -E. 'pontaneous R To speech 3 To &ain H ne$er 1 <erbal Res(onse ,riented and con$erses Confused con$ersation Inappropriate words Incomprehensi#le sounds "one -D. ) R 3 H 1 1est Motor Res(onse ,#eys commands LocaliAes pain %ithdrawal to pain 0#normal flexion -decorticate. 0#normal extension -decere#rate. "il -M. I ) R 3 H 1

Mini8Mental Status8 orientation to timeBplace -) pts, year, season, month, day, day of weekL ) pts, country, pro$ince, city, hospital, floor., memory -3 pts, honesty, tulip, #lackL 3 pts, delayed recall., attentionBconcentration -) pts, serial 2s, 5%,=L@6

#ackwards., lan ua e tests8 comprehension -3 pts, three point command., readin -1 pt, 5close your eyes6., writin -1 pt, complete sentence., repetition -1 pt, 5no ifs, ands or #uts6., namin -H pts, watch, pen., spatial a#ility -1 pt, intersectin penta ons. Cranial nerves 9 C" III, ID, DI8 Extra3ocular mo$ements -patient follows your fin er or the handle of a reflex hammer in an ;3pattern, check for diplopia in the center and at the extremes of the $isual fields! C" II8 Disual fields #y confrontation 9 -one eye at a time. patient holds own hand o$er one eye and counts fin ers flashed in left and ri ht fields simultaneously -upper and lower. or identifies the wi lin fin er as it enters each quadrant on confrontation if too confused to count! ?y acuity8 'nellen card! Qundi 9 check for papilledema usin ophthalmoscope! C" II, III, D1 7 DII8 &upillary li ht reflex and accommodation, corneal reflex! C" D8 Qacial sensation to li ht touch in the ophthalmic, maxillary and mandi#ular di$isions of the tri eminal ner$e! Tri eminal motor8 clench teeth, lateral :aw mo$ement! C" DII, <II8 Qacial muscle power 9 raise eye#rows, squint eyes closed, show teeth, protrude ton ue, o#ser$e palatal mo$ement on sayin 50h6, puff cheeks, platysma! C" DII, I<, <8 >a reflex, o#ser$e symmetric mo$ement of palate, swallowin , taste! C" DIII8 >ross hearin 9 ru# thum# and index li htly while approachin the patient+s ear, note when they #e in to hear the sound! C" <I8 'ternocleidomastoid power and trapeAius power! 1ody 9 &ronator drift 9 arm strai ht out in front of patient with palms up, eyes closed! Look for curlin of fin ers from the ulnar side, pronation and downward mo$ement of the affected side! ;offman+s reflex 9 with the patient+s relaxed hand in a palm3down position, squeeAe and flick the nail of the index or lon fin ers #etween your thum# and lon fin er! Thum# flexion indicates a positi$e test and denotes upper motor neuron lesion -similar to ?a#inski+s reflex.! Cere#ellar tests 9 fin er3 nose rapid alternatin mo$ements -dysdiadocokinesis., heal3shin! ?ody power 9 deltoids, #iceps, triceps, wrist extension and flexion, fin er a#duction and adduction, psoas -hip flexion 9 lift each knee off the #ed a ainst resistance., quadriceps, hamstrin s, ankle dorsiflexion -test plantar flexion while standin .! ?ody sensation 9 li ht touch, pin prick, and cold temperature -use a meta T3#asin or turnin fork. are tested on the distal upper lim#s -forearms and hands. and lower lim#s -forele and feet.! Di#ration sensation is tested usin a C31H4 ;A tunin fork on the :oint capsules of the most distal :oints at which the $i#ration can #e felt! ?ody reflexes 9 #iceps :erk, #rachioradialis, knee, ankle, ?a#inski! &atient standin Gait 9 o#ser$e for wide3#ase, &arkinsonian festinationBshuttle, consistent lateraliAin falls! ?alance 9 ti ht3rope walk, stand with arms crossed and feet in line, =hom#er -feet to ether, eyes closed.! &lantar flexion power 9 hold patient+s hands for #alance, ask patient to stand on one foot, then raise themsel$es up on the toes! &atient lyin supine Tone 9 passi$e rapid alternatin forearm rotation, passi$e rapid el#ow flexionBextension with one thum# on the #iceps tendon to feel for co 3wheelin ! =apid liftin of the relaxed le from #ehind the knee 9 heel remains on the #ed in normal tone! 4$ , young .an is brought to the 2.ergency de(art.ent 'ith an e(igastric stab 'ound sustained in a bar *ight$ There is a nurse in the roo.$ Manage$ Ensta#le means either an acutely chan in condition, $ital si ns dan erously #eyond normal ran es -e! ! low ?&., or any life3 threatenin condition which is inadequately controlled! This patient is unsta#le due to sta# wound with unknown consequences which is an acutely chan in and inadequately controlled pro#lem and therefore requires sta#iliAation or resuscitation! The 0CL' and 0TL' format is useful8 i!e! primary sur$ey, orders, secondary sur$ey, orders! "ote that the simulated patient may ha$e an impressi$e simulated in:ury! @o not allow this to distract you from the primary sur$ey! Pri.ary survey8 -mnemonic 0?C@. ,ir'ay8 check for patent airway -look in mouth., is airway threatened #y #loodF ;i h chance of aspiration due to poor le$el of consciousness, neck or face swellin ! If airway is compromised, immediately place an oral airway or intu#ate! 1reathing8 is patient #reathin , check ,H sat, may require immediate manual #a 3$al$e mask followed #y intu#ation with positi$e pressure $entilation! Circulation8 ?&, ;=, rhythm on monitor, acti$e hi h $olume #leedin ! &atient may require immediate chest compressions, defi#rillation or cardiac dru s if there is an unsta#le rhythm -Dentricular fi#rillation, unsta#le $entricular tachycardia, &E0, asystole, &'T, or #radycardia.!

-ebilitation8 refers to the >las ow Coma 'cale ->C'.8 Glasgo' Co.a Scale 2ye O(ening -E. 'pontaneous R To speech 3 To &ain H ne$er 1 <erbal Res(onse ,riented and con$erses Confused con$ersation Inappropriate words Incomprehensi#le sounds "one -D. ) R 3 H 1 1est Motor Res(onse ,#eys commands LocaliAes pain %ithdrawal to pain 0#normal flexion -decorticate. 0#normal extension -decere#rate. "il -M. I ) R 3 H 1

Note8 'tandard painful stimulus is ru##in the knuckle on the sternum! Qor withdrawal, apply pressure on the #ase of the nail #ed with a pen! @ecorticate posture is arm flexion with le extension on the same side of the #ody, may #e unilateral or #ilateral! Indicates a lesion a#o$e the #rainstem! @ecere#rate posture is arm and ipsilateral le extension, may #e unilateral or #ilateral, indicates #rainstem in$ol$ement! 0 >C' of K or less is considered an indication for intu#ation #ecause of the risk of poor protection of the airway from aspiration! Pri.ary orders8 oxy en, monitorin -EC>, ,H sat, automatic ?& cuff or arterial line., ID access8 need two lar e3#ore -1I au e, 1R if possi#le 9 femoral $ein cortice with H lumens., run wide open with normal saline for acutely low ?&, may need to #e more restrained if pulmonary edema is a pro#lem! Coma cocktail if dia nosis not known already8 thiamine 144 m ID, narcan 1 m ID, flumaAenil 4!1 m ID -1 amp @)4% is no lon er included in this cocktail #ecause of deleterious effects of hi h serum lucose on the in:ured #rain.! Initial in$esti ations8 C?C, lytes, urea, creatinine, 0?>, lucose, ioniAed Ca, CT3 M?, troponin, I"=B&TT, EC>, porta#le C<=, cross3ta#le lateral C3spine and hard collar if there is head in:ury or any si nificant trauma! Secondary survey8 ;ead to toe physical exam! Ditals! ;ead and neck8 inspect for lacerations and contusions, pupillary response, doll+s eyes -careful of neck, may not #e a#le to turn head enou h to do this., corneal reflexes, palpate facial #ones for sta#ility, look in nose and ears for #lood or C'Q leaks, hemotympanum! Check oral ca$ity, a reflex, palpate dorsal cer$ical spines for pain and ali nment, is the trachea midlineF Chest8 #reath sounds, heart sounds, radial pulses #ilaterally! 0#domen8 ri idity is an indicator for immediate eneral sur ery, auscultate for #owel sounds, palpate li$er and spleen! Lo roll patient onto #ack, inspect! =ectal! &alpate for pel$ic sta#ility and intactness of lon #ones! Secondary orders8 foley, "> tu#e if patient may o to sur ery or require charcoal! 'pecific inter$entions #ased on findin s! Qurther <3rays, CT head if cause of decreased le$el of consciousness unclear or if there may ha$e #een a seiAure! Clearing C8s(ines8 The principle of clearin C3spines is to rule out #oth #ony fractures and li amentous in:ury, either of which can make the spine dan erously unsta#le! Most emer ency physicians will clear the cer$ical spine in the case of an alert patient who has no pain on palpation of the dorsal spinous processes and a normal cross3ta#le lateral C3spine <3ray! If the patient has neck pain, flexionBextension plane films are done! These in$ol$e ently flexin , then extendin the neck and takin $iews at each extreme! The patient must #e alert enou h to warn the examiner of paresthesias in the hands or increased neck pain on mo$ement durin this procedure, which may indicate compromise of the neural elements! QlexionBextension $iews may #e done under fluoroscopy if the patient is not alert! +istory8 if a$aila#le8 ;ow did the patient o#tain the wound, was there mechanism of action for other in:ury, i!e! collapseBfall, precedin and su#sequent e$ents, did patient lose consciousness, duration of unconsciousness, did the patient fall, in:uries durin fall, medications and dru s, smokin , aller ies, past medical history, family history, re$iew of systems! Manage.ent8 Qor unsta#le #lood pressure, #lood loss is the most likely cause -C?C may #e normal with a lar e acute #lood loss., type and cross for R3I units dependin on estimated se$erity and han #lood as soon as possi#le! Is the patient+s a#dominal wound the first priorityF Examine for head in:ury, other in:uries! Explore wound with a lo$ed fin er on secondary sur$ey, if wound is more than superficial, consult eneral sur ery and prepare patient for immediate sur ical exploration in the ,=! ,#tain details of the history from family or witnesses if possi#le! #$ 30 year old 'o.an 'ith si7 'ee s o* e(ista7is& (etechiae and easy bruising$ Per*or. a *ocused (hysical e7a.$ %indings) (etechiae& bruises$ B) The (atient has a nor.al C1C e7ce(t *or (latelets /0 :nor.al !308>00;$ 5hat is the .ost li ely diagnosisC 5hat *our areas on history 'ould hel( to con*ir. this diagnosisC 5hat *our investigations 'ould you orderC

+e.atologic e7a.8 patient disro#ed to underwear, draped #elow the waist! &atient sittin Inspect the patient enerally for petechiae, a#normal skin tone, hair fallin out! Inspect the fin er and toe nails for dystrophy, flame hemorrha es, leukonychia, inspected the palm for erythema and @upuytren+s contracture! Look in the nose and mouth for #leedin , petechiae, masses! &alpate the anterior and posterior trian les of the neck, the supra and infra cla$icular areas, and the axillae for lymph nodes! &alpate the thyroid while standin #ehind the patient, ask her to swallow! Chest 9 from #ehind the patient, inspect the skin! &ercuss the lun fields for effusions and consolidations, auscultate the lun fields! &ercuss and auscultate the anterior lun fields! Listen o$er the aortic -ri ht upper sternal #order Hnd IC' 9 systolic O stenosis, diastolic O re ur itation, continuous O ?T shunt, ri ht or left., pulmonary -left upper sternal #order Hnd IC' 9 systolic O 0'@, pulmonary flow, continuous O ductus, L?T shunt., tricuspid -left lower sternal #order )th IC' 9 systolic O re ur itation !!! 'till+s murmur, diastolic O stenosis. and mitral -apex )th IC' and midcla$icular line 9 systolic O prolapse, re ur itationL diastolic O stenosis. areas! &atient lyin supine Compress the sternum and ri#ca e for pain -seen in multiple myeloma.! Inspect the a#domen! 0uscultate for #owel sounds! &alpate for enlar ement of the spleen and li$er! &ercuss the li$er! &alpate the roin for lymph nodes! "ote8 a$oid rectal exam as this trauma may cause #leedin ! Most li ely diagnosis8 idiopathic throm#ocytopenic purpura -IT&., also called immunolo ic throm#ocytopenic purpura or %edhof+s disease! %our *indings on history 'hich 'ould hel( to con*ir. the diagnosis8 1! =emittin 3relapsin course, H! Mild fe$ers, 3! 'plenic discomfort due to mild enlar ement, R! ?leedin after low doses of "'0I@! %our investigations8 ?lood smear, I"=B&TT -for hemophilia., serum ureaBcreatinine -for hemolytic3uremic syndrome., serum platelet3associated I > -for IT&.! ?$ 4/ year old .an (resents to the 2.ergency -e(art.ent 'ith !/ hours su(ra(ubic disco.*ort and inability to urinate$ Catheteri6ation yields !/00 cc urine$ Ta e a history$ B) 5hat is the .ost li ely cause o* this .ans (roble.C Give three other (ossible diagnoses$ 5hat *our investigations 'ould you orderC +istory8 name, a e, occupation! ;istory of suprapu#ic pain and ina#ility to urinate! ;istory of pain on urination, frank #lood in the urine, color of urine, difficulty initiatin or maintainin urinary stream, fe$er, renal pain, roin pain! &re$ious renal colicB dia nosed prostate hypertrophy, prostate cancer, prostatism, nephrolithiasis, ETIsF Mali nant symptoms8 ni ht sweats, wei ht loss, fati ue! Medications, dru sBalcohol, smokin , past medical history, past sur ical history, history of pel$ic radiation, TE=&, family history, re$iew of systems! Most li ely diagnosis8 #eni n prostatic hyperplasia! Other (ossible diagnoses8 ETI, prostatitis, prostate cancer! Qour in$esti ations8 ureaBcreatinine, urinalysis, prostate specific anti en -&'0., renal ultrasound! Treat.ent8 watchful waitin -)4* resol$e spontaneously., medical -alpha3adrener ic anta onists 9 TeraAosin, doxaAosin, tamsulosinL )3alpha3reductase inhi#itors 9 finasteride., sur ery -TE=& $s! open prostatectomy., minimally in$asi$e -stents, microwa$e therapy, laser a#lation, cryotherapy, ;IQE, TE"0.! "$ 4 .onth old child 'ho 9ust had a sei6ure$ Ta e a history *ro. the .other in the 2.ergency -e(art.ent$ %indings) short sei6ure 'ith T 3"$D C$ Never had sei6ures in the (ast$ -evelo(.entally nor.al$ B) 5hat is the .ost li ely diagnosisC 5hat 'ould you tell the .other about any (ossible recurrenceC 5hat advice do you give i* the child has another sei6ureC +istory8 "ame, a e! @escri#e seiAure duration, what #ody parts affected and in what order, premonitory si ns, post3ictal state -decrease in le$el of consciousness, headache, sensory phenomena.! &re$ious seiAureF 0sk a#out precedin trauma or illness or medications taken, the child+s temperature at the time of the seiAure! Menin itis si ns, neurolo ical! ;istory of pro#lems durin the pre nancy and #irth! @e$elopmental history! Child+s medical history, sur ical history, medications, aller ies!

Selected -evelo(.ental Milestones S(eech I months 1H months HR months H33 years Gross .otor I months ( months 1H months 1) months %ine .otor 1H months HR months Social I months ( months H years ) years

initiates sounds, eye contact H words #eyond mama and dada H33 word phrases short sentences roll o$er stand cruise walk pincer rasp turns pa es in a #ook stran er anxiety separation anxiety says 5no6 prints name

Most li ely diagnosis8 fe#rile seiAure -fe#rile seiAures usually I months to I years, associated with initial rapid rise in temperature, no neurolo ic a#normalitiesBe$idence of C"' infectionBinflammation #efore or after, no history of non3fe#rile seiAures, most common eneraliAed tonic3clonic, M 1) minutes duration, no recurrence in HR hours, atypical may show focal ori inBJ 1) minutesBJ 1BHR hoursBtransient neurolo ic defect.! Prognosis8 after a sin le fe#rile seiAure I)* will ne$er ha$e another seiAure! 34* will ha$e further fe#rile seiAures, 3* will o on to ha$e seiAures without fe$er and H* will de$elop lifelon epilepsy! Manage.ent8 find source of fe$er, Tylenol -antipyretics., L& to rule out menin itis if si ns of menin itis, counselin and reassurance to patient and parents if fe#rile seiAures! Treat.ent o* recurrence8 control fe$er with antipyretics -Tylenol., tepid #ath, fluids for comfort only and use 0ti$an -loraAepam. 1 m 'LB&, -or diaAepam )314 m &=. if a seiAure occurs at home! Turn patient onto hisBher side, do not force o#:ects or fin ers into mouth! ?rin to E= if seiAure does not stop within fifteen minutes! 'eiAures do not cause mental impairment unless they are prolon ed -J 34 min., althou h seiAures can #e a symptom of #rain dama e! &atient should #e in$esti ated with CT head and EE>! &rophylactic anticon$ulsant therapy is a consideration with repeated seiAures! !0$ 4? year old .an 'ith di**iculty s'allo'ing$ Ta e a history$ %indings) thro's u( a*ter eating$ Can s'allo' liEuids only$ 5eight loss and *atigue$ S.o er$ B) F8ray o* bariu. s'allo' sho'ing narro'ing o* contrast at TD84) describe the abnor.ality$ 5hat is the li ely diagnosisC 5hat investigation 'ould con*ir. the diagnosisC 5hat *urther investigations 'ould you orderC +istory8 -@yspha ia O difficulty swallowin . onset, chronolo y, description of pro#lem, a ra$atin and relie$in factors! Is the difficulty transferrin food from mouth to esopha us su estin oropharyn eal dyspha ia with food ettin stuck immediately after swallowin often with nasal re ur itationF %ith esopha eal dyspha ia food seems to #e stuck further down! Is the pro#lem worse with solids -su ests mechanical o#struction., or liquids -su ests neuromuscular dysfunction, often can+t swallow either solids or liquids.! Is there a sensation of a lump in the throatF - lo#us hystericus O transitory sensation of a lump in the throat related to anxiety.! &ro ression from solid swallowin difficulty to difficulty swallowin #oth solids and liquids su ests pro ressi$e o#struction such as from a worsenin stricture or rowin tumor! The com#ination of intermittent o#struction and chest pain su ests esopha eal spasm! 0sk a#out peptic ulcer, reflux, hiatus hernia, wei ht loss, ni ht sweats, fati ue, hematemesis, #lack stools, pain! Medications, dru sBalcohol, aller ies, smokin , past medical history, family history, re$iew of systems! Oro(haryngeal8 "eurolo ical cortical pseudo#ul#ar palsy -EM" lesion. due to #ilateral strokeL #ul#ar ischemia -stroke.L syrin o#ul#iaL tumor -LM".L peripheral polioL 0L'! Muscular M@L cricopharyn eal incoordination -failure of EE' to relax with swallowin ., sometimes seen with >E=@L Senker+s di$erticulum! 2so(hageal8 solid food only mechanical o#struction intermittent O lower esopha eal rin Bwe#L pro ressi$e heart#urn O peptic ulcerL a e J )4 O carcinoma! 'olid or liquid food neuromuscular disorder intermittent O diffuse esopha eal spasmL pro ressi$e reflux O sclerodermaL respiratory symptoms O achalasia!

-i**erential diagnosis8 Mechanical o#struction8 tumor, stricture -secondary to >E=@, trauma etc!.! "euromuscular o#struction8 achalasia -cardiac sphincter does not relax., cranial ner$e palsy, M', supranuclear palsy, stroke, motor neuron disease, myasthenia ra$is, muscular dystrophy! -escri(tion o* 1ariu. s'allo' *indings8 strin si n, raded narrowin of intra3esopha eal diameter extendin from T) to TK le$el! Most li ely diagnosis8 esopha eal cancer! Investigation to con*ir. diagnosis8 endoscopy with #iopsy! %urther investigations8 CT chest -for mediastinal and lymph node in$ol$ement., chest <3ray, li$er function tests, a#dominal ultrasound -for mets.! !!$ /3 year old 'ith 1P !40/!00 in both ar.s$ Per*or. a *ocused (hysical e7a.$ B) Give *our (ossible diagnoses$ 5hat *our investigations 'ould you orderC I* these investigations 'ere negative& give D ste(s in your initial .anage.ent (lan$ Physical e7a. *or hy(ertension8 com#ines exams for atherosclerosis, coarctation, hyperthyroidism and Cushin +s! &atient should #e disro#ed to underwear and draped #elow the waist! &atient 'ittin Take $itals -need ?& in all four lim#s 9 le s to #e done when patient is lyin down.! Inspect for cyanosis, arcus senilis in the eyes -si n of hi h cholesterol., #ul in $eins in the upper chest -'DC syndrome., supracla$icular fat pad, #uffalo hump, moon face, truncal o#esity, striae, nicotine stains on fin ers, clu##in , flame hemorrha es on nails, o#esity, hi h work of #reathin , intercostal indrawin , symmetric chest mo$ement, $isi#le apex #eat! Qundoscopy for retinopathy of hypertension8 -in order of increasin se$erity of dama e. constriction and sclerosis of retinal arterioles, hemorrha es, exudates, papilledema! Thyroid e7a.8 Inspect patient for proptosis, 5thyroid stare6 -upper lids do not o$erlap the irises.! ;a$e patient follow your fin er up and down to check for lid la and lo#e la ! Is skin thin dry and flaky or diaphoretic! &alpate thyroid standin #ehind patient, ask patient to swallow! Inspect nails for leukonychia and hands for tremor -can place a piece of paper on the hand held horiAontal to detect fine tremor. and clu##in Bthickenin of distal phalan es! Check #iceps reflexes with thum# held o$er the tendon, feel for slow return phase reflex of hypothyroidism! &alpate the apex, note whether it is laterally displaced -lateral to the mid3cla$icular line. and feel for thrill or hea$e, feel radial pulses in the arms simultaneously, note any delay! &ercuss the lun fields anteriorly and posteriorly! 0uscultate the lun fields anteriorly and posteriorly, listen o$er the aortic -ri ht upper sternal #order H nd interspace systolic O stenosis, diastolic O re ur itation, continuous O ?T shunt, ri ht or left., pulmonary -left upper sternal #order H nd interspace systolic O 0'@, pulmonary flow, continuous O ductus, L?T shunt., tricuspid -left lower sternal #order ) th interspace systolic O re ur itation !!! 'till+s murmur, diastolic O stenosis. and mitral -apex at ) th interspace and midcla$icular line systolic O prolapse, re ur itationL diastolic O stenosis. areas, as well as o$er the ri ht cla$icle, and #oth carotids! Listen for ru#! To #rin out an aortic murmur -typically aortic re ur itation., and coarctation #ruits, ask patient to lean forward, exhale and stop #reathin while you listen o$er the aortic and pulmonic areas! &atient Lyin 'upine 0uscultate for #ruits o$er the renals on the a#domen! ,#ser$e for pulsations due to a#dominal aortic aneurysm, palpate a#domen for hepatome aly! &alpate femoral pulses, and auscultate for femoral #ruits, palpate the popliteal pulses, inspect the le s and feet for $enous stasis or arterial insufficiency ulcers, palpate the dorsalis pedis and ti#ialis posterior pedal pulses! Qeel the ankles for pittin edema! Tibial 1P8 ?& cuff placed around calf, auscultate the ti#ialis posterior pulse posterior to medial malleolus on ri ht and left!

14

G<P8 =aise the head of the #ed 34 de rees and inspect the neck! 0 :u ular $enous pulsation hi her than R3) cm a#o$e the sternal notch -approximately the le$el of the ri ht atrium. is a#normal! If no ND& is $isi#le then you may ha$e to raise or lower the #ed for hi h or low ND&s respecti$ely! Check the hepato:u ular reflux -supine, mouth open, #reathin normally compress the li$er, the :u ular $enous pulse should either not rise or remain ele$ated only transiently, a sustained ele$ation J 1 cm in the ND& is patholo ical.! This is used to assess hi h :u ular $enous pressure and =D function! Classification of ?& @?&8 M (4 -normal., (4314R -mild ;T"., 14)311R -moderate ;T"., J 11) -se$ere ;T". '?&8 M 1R4 -normal., 1R431)( -#orderline isolated systolic ;T"., J 1I4 -isolated systolic ;T". %our (ossible diagnoses8 essential hy(ertension, renal +TN -renal artery insufficiency, renal parenchymal disease., endocrine -thyroid hormone, ,C&, primary hyperaldosteronism, hyperparathyroidism, pheochromocytoma, Cushin s. coarctation o* the aorta, others -enAymatic defects, neurolo ical disorders, dru 3induced -prolon ed corticosteroid use., hypercalcemia, watch for la#ile 5white3coat6 ;T".! Investigations8 =epeat ?& 3x o$er next I months, urinalysis, C?C, Cr, lytes, fastin serum lucose, cholesterol panel, 1H lead EC>, T';, renal EB'! Initial Manage.ent8 1! 'mokin cessation! H! 0lcohol restriction to low risk drinkin uidelines! 3! 'alt restriction -max! (43 134 mmol 9 332 per day.! R! 'aturated fat intake reduction! )! %ei ht reduction if ?MI J H) -at least R!) k .! I! =e ular aero#ic exercise -)43I4 min, 33Rx per week.! !/$ /! year old *e.ale 'ith bloody diarrhea$ Ta e a history$ %indings) ,bdo.inal cra.(ing$ Si7 'atery stools in the (ast *our hours containing .aroon colored blood$ %eels di66y and 'ea $ No (revious history o* diarrhea (reviously 'ell$ B) 5hat t'o *indings on history indicate the seriousness o* the (roble.C Give three (ossible diagnoses$ Give *our investigations a((ro(riate to this situation$ +istory8 name, a e, occupation! ,nset, duration, frequency, of diarrhea! 0ppearance of stools8 how well formed, is #lood on -analBrectal laceration. or admixed with stools, is #lood #ri ht red -lower tract #leed. or dark #rown3#lack -upper tract #leed, e! ! stomach.! &ain with #owel mo$ements, a#dominal pain or cramps with location, radiation, precipitatin factors and alle$iatin factors, quality, se$erity, timin with respect to defecation, as #loatin ! ;eart #urn, peptic ulcer, reflux, hiatus hernia! Extra3intestinal manifestations of inflammatory #owel disease8 ask a#out iritis, arthritis, mouth ulcers, anal ulcers, skin lesions, kidney stones! Infectious diarrhea8 inquire a#out fe$er, nausea, $omitin , wei ht loss, fati ue! =ecent tra$el, consumption of unusual foods or foods which may ha$e #een contaminated! =ecent exposure to anti#iotics! Qamily mem#ers sick at home! &el$ic pain, $a inal dischar e, $a inal #leedin ! &ast medical history, medications -especially "'0I@s, laxati$es, anti#iotics., family history of Crohn+s, ulcerati$e colitis, familial polyposis, re$iew of systems! T'o *indings 'hich indicate the seriousness o* the (roble. 8 patient feels diAAy and weak! Three (ossible diagnoses8 astroenteritis, #leedin peptic ulcer, inflammatory #owel disease! %our investigations8 C?C with differential, stool for o$a 7 parasites with culture 7 sensiti$ities, Clostridium difficile toxin! Endoscopy -#ut a#o$e first.! Type and cross for R units &=?Cs!

!""4
!3$ Middle aged 'o.an 'ith systolic e9ection .ur.ur radiating into the carotids$ Per*or. (hysical e7a.$ The physical exam for a patient with a heart murmur is a cardiopulmonary exam! &atient in sittin position8 take $itals Inspect for sur ical scars, trauma, #ony a#normalities, cyanosis, arcus senilis in the eyes -si n of hi h cholesterol., #ul in $eins in the upper chest -'DC syndrome., nicotine stains on fin ers, clu##in , flame hemorrha e on nails, o#esity, work of #reathin , intercostal indrawin , symmetric chest mo$ement, $isi#le apex #eat! &alpate the apex, note whether it is laterally displaced -lateral to the mid3cla$icular line. and feel for thrill or hea$e, feel radial pluses #ilaterally!

11

&ercuss the lun fields anteriorly and posteriorly! 0uscultate the lun fields anteriorly and posteriorly, always e$aluate heart sounds #efore murmur! Qirst listen for ' 1 and 'H, then look for '3 and 'R and any other unusual heart sounds! Listen o$er the aortic -ri ht upper sternal #order Hnd IC' 9 systolic O stenosis, diastolic O re ur itation, continuous O ?T shunt, ri ht or left., (ul.onary -left upper sternal #order Hnd IC' 9 systolic O 0'@, pulmonary flow, continuous O ductus, L?T shunt., tricus(id -left lower sternal #order )th IC' 9 systolic O re ur itation !!! 'till+s murmur, diastolic O stenosis. and .itral -apex )th IC' and midcla$icular line 9 systolic O prolapse, re ur itationL diastolic O stenosis. areas as well as o$er the ri ht cla$icle, and #oth carotids! Listen for ru#! To #rin out an aortic murmur -typically aortic re ur itation., ask patient to lean forward, exhale and stop #reathin while you listen o$er the aortic area! To #rin out a mitral murmur, ask patient to lie supine and roll partly onto the left side while you listen o$er the apex! In eneral, murmurs are accentuated #y increasin the dynamicity of the heart with mild exercise, such as askin the patient to walk up a fli ht of stairs! Murmurs are descri#ed in terms of where they are heard loudest, where the sound radiates, whether it occurs in systole or diastole, the pitch -e! ! hi h, low., quality -e! ! harsh, #lowin , musical., contour -e! ! crescendo, decrescendo or plateau. and its loudness raded out of six -e! ! IIBDI.! The murmur of aortic stenosis is loudest o$er the aortic area, radiates to the cla$icle or carotids, occurs in systole, has medium or hi h pitch, is harsh and crescendo3decrescendo! 0 mitral re ur itation murmur #y contrast, is loudest o$er the apex, also occurs in systole, radiates to the axilla, is medium to hi h in pitch, #lowin and plateau! Innocent murmurs are M3BI in intensity, peak early in systole, stop lon #efore 'H, are heard #est at the #ase of the heart -aortic and pulmonary areas., are not associated with clicks or hea$es, and EC> and C<= are normal! &atient lyin supine 0uscultate for #ruits o$er the renals on the a#domen! ,#ser$e for pulsations due to a#dominal aortic aneurysm, palpate a#domen, femoral pulses, and auscultate for femoral #ruits, palpate the popliteal pulses, inspect the le s and feet for $enous stasis or arterial insufficiency ulcers, palpate the dorsalis pedis and ti#ialis posterior pedal pulses! Qeel the ankles for pittin edema! G<P8 =aise the head of the #ed 34 de rees and inspect the neck! 0 :u ular $enous pulsation hi her than R3) cm a#o$e the sternal notch -approximately the le$el of the ri ht atrium. is a#normal! If no ND& is $isi#le then you may ha$e to raise or lower the #ed for hi h or low ND&s respecti$ely! Check the hepato:u ular reflux -supine, mouth open, #reathin normally compress the li$er, the :u ular $enous pulse should either not rise or remain ele$ated only transiently, a sustained ele$ation J 1 cm in the ND& is patholo ical.! This is used to assess hi h :u ular $enous pressure and =D function! !>$ D0 year old .an 'ith le*t8sided chest (ain$ Manage :.eans history& (hysical& investigations and treat.ent;$ %indings) bruise on chest 'all& nor.al CFR and 2CG$ -i**erential -iagnosis *or Chest Pain Cardiac 0n ina &ulmonary MI &neumonia wB pleuritis &ericarditis &neumothorax Myocarditis &E @issectin aorta &ulmonary hypertension Non8Cardiac >I Esopha eal reflux Elcer

M'TB"euro 0rthritis Chondritis =i# fractures ;erpes Soster

&sycholo ic 0nxiety &anic

+istory *or chest (ain8 descri#e the pain, location, radiation, quality, time of onset, duration, intensity, circumstances under which it occurs, a ra$atin and relie$in factors, associated symptoms such as nausea, shortness of #reath, diAAiness, diaphoresis, dependent edema! Le pain! =espiratory symptoms8 cou h, sputum, fe$er, hemoptysis! >I symptoms8 heart#urn, dyspha ia! &re$ious episodes, chronolo y of these! ;istory of trauma, asthma, #ronchitis, C,&@, pneumothorax, recent $iral illness and pre$ious chicken pox -;erpes Soster can cause chest pain., astritis, peptic ulcer, reflux! =isk factors for heart and lun disease8 smokin , hypertension, hyperlipidemia! &ast medical history, especially dia#etes, heart disease includin pericarditis, lun disease, >I pro#lems, sur ical history, and family history! Medications, dru use, smokin , aller ies, re$iew of systems! Physical e7a.8 Cardiopulmonary exam as in question P13! Investigations8 C<=, EC>!

1H

Treat.ent8 i$en a normal C<= and EC> with a chest wall #ruise as e$idence of trauma send patient home, recommend non3 prescription pain medication -Tylenol andBor i#uprofen. and ad$ise that the pain should su#side radually! 'ince the patient is at risk #ecause of his a e roup and male ender, explain the symptoms of myocardial infarct -MI. and ad$ise to return immediately if these occur! !D$ Houng .an 'ith recent onset bac (ain and li.($ Ta e history and (hysical$ , di**erential *or lo' bac (ain is8 1! @e enerati$e -(4* of all #ack pain. Mechanical -de enerati$e, facet :oint pain, muscle strainBspasm. 'pinal stenosis -con enital, osteophyte, central disc. &eripheral ner$e compression -disc herniation or rupture. H! Cauda Equina syndrome 3! "eoplastic8 primary or metastatic R! Trauma8 fracture -compression, distraction, translation, rotation. )! 'pondyloarthropathies8 e! ! ankylosin spondylitis I! @iscitisBosteomyelitis 2! =eferred8 aorta -a#dominal aortic aneurysm., renal -pyelonephritis., ureter -nephrolithiasis., pancreas -pancreatitis. K! Malin erin ?ecause disco enic and stenotic radiculopathy which ha$e not impro$ed o$er at least R weeks may #e treata#le sur ically, the priority of a history and physical for #ack pain is to differentiate radiculopathy from other causes and to identify the ner$e root! The most common disk herniation is a posterolateral LR3), which compresses the L) root! The herniation will also compress the LR root if the herniation is far lateral and the '1 root if it is more medial -central.! The second most common herniation is a posterolateral L)3'1, which compresses the '1 root! In the thoracic and lum#ar spine, the ner$e roots exit #elow the pedicles of the $erte#ra of the same num#er, while in the neck the ner$e root exits a#o$e the pedicle of the $erte#ra of the same num#er! L) compression produces radiation from #uttock to lateral calf, lateral calf pain, num#ness of the medial dorsum of foot -includin we# of reat toe., and ankle dorsiflexion weakness, '1 compression produces radiation posteriorly down le to heel, posterior calf pain, lateral foot num#ness and ankle plantar flexion weakness -with decreased ankle :erk.! +istory8 =ed fla s -?0CT&0I". ?8 #owel or #ladder dysfunctionL 08 anesthesia -saddle.L C8 constitutional symptomsBmali nancyL T8 chronic diseaseL &8 paresthesiasL 08 a e J )4L I8 ID dru userL "8 neuromotor deficits! @escri#e the pain, location, radiation -L) radiculopathy causes radiation from #uttock to lateral calf, '1 radiates posteriorly down le to heel., quality, duration, frequency, intensity, circumstances under which it occurs, a ra$atin and relie$in factors! ,nset and chronolo y, pre$ious episodes! &re$ious in$esti ations, treatment! &ain worse lyin down and #ilateral le weakness su ests spinal stenosis or ankylosin spondylitis! 'pinal stenosis is characteriAed #y worsenin of symptoms with standin and walkin , with relief on #endin and settin -a typical history of leanin on and #endin o$er the shoppin cart for relief of pain while shoppin is su esti$e of spinal stenosis.! 0nkylosin spondylitis is characteriAed #y mornin stiffness relie$ed #y acti$ity! &ain worse in #ack than in #uttock or le su ests mechanical #ack pain! &ain worse in #uttock or le than in #ack su ests radiculopathy! &redominatin symptoms of stiffness are su esti$e of ankylosin spondylitis! ?ack pain is recurrin and tends to #e nocturnal! Mornin stiffness impro$es o$er the day! May #e associated with wei ht loss, fe$er, fati ue, anemia! Qocus on :oint symptoms -typically lar e :oints., u$eitis -occurs in one third of cases., and family history! ;as the patient had a fe$er, wei ht loss, ni ht sweats -si ns of cancer., urinary tract infection -si n of urinary retention., :oint pain, u$eitis -inflammation of the u$eal tract8 iris, ciliary #ody, and choroids si n of ankylosin spondylitis.F 0sk a#out effect on acti$ities of daily li$in , functional limitations! 0ssociated num#ness, weakness! 0re the symptoms impro$in or worsenin F %hat are the patient+s conclusions a#out the pain and expectations of the physicianF Medications, dru s and alcohol, smokin , past medical history, family history, re$iew of systems! Cauda eEuina syndro.e8 Inquiry into #owel, #ladder, and sexual function to re$eal this rare syndrome is o#li atory and a source of frequent false alarms! ?ecause these functions may not reco$er once lost, cauda equina syndrome due to a sur ically treata#le lesion is a sur ical priority if the time course is su#acute and an emer ency if the loss of function is acute! The syndrome consists of saddle anesthesia -perineal num#ness., lax anus, impotence, urinary retention and #owel incontinence! "ote that this com#ination of si ns is due to preser$ation of sympathetic tone with loss of parasympathetic tone! 'ympathetic

13

tone is preser$ed #ecause it is carried extra3spinally, while parasympathetic si nals are carried $ia the inferior spine and ner$e roots! "ote that #owel contraction and penile erection are parasympathetically dri$en! Physical e7a.8 'tandin 0ssess ait, posture, ran e of motion includin rotation, lateral and forward flexion, extension -pain worse on forward flexion and relief on extension su est disco enic pain, pain worse on extension su ests facet :oint pain.! Qor ankylosin spondylitis8 %ri ht3'cho#er test positi$e when distance #etween the lum#rosacral :unction and a point 14 cm a#o$e -identified #y palpation on the erect spine., distract #y less than ) cm on full forward flexion of the spine! Modified 'cho#er -i!e! detection of decreased forward flexion of lum#ar spine.! Lateral flexion is impaired when the hand mo$es downward #y less than 3 cm on the ipsilateral thi h! Look for scoliosis on standin -shoulder hei hts equalF. and forward flexionL check for ri# hump! Inspect #ack for spina #ifida! &alpate for tender areas especially sacroiliac :oints, compress pel$is to elicit pain of sacroiliitis -hallmark of ankylosin spondylitis.! Muscle tone, percuss costo$erte#ral an les for renal pain! ;a$e patient walk on toes, heels! 0sk patient to stand on one foot at a time and push up into tiptoe for ankle plantar flexor stren th -'1.! 'ittin Tnee :erks -LR. with quadriceps exposed, watch contraction! 0nkle :erks -'1., rapidly dorsiflex each foot to test for clonus! ?a#inski! Compare calf irths for wastin #y measurin calf circumference 14 cm #elow ti#ial tu#erosity! Test power of quadriceps, hamstrin s, psoas -raise knee up a ainst resistance., ankle dorsiflexors! 0sk patient to strai hten #oth le s and compare this position to the de ree of forward flexion the patient was a#le to achie$e on standin ran e of motion! 'uspicion of malin erin is raised if the patient claims to #e una#le to #end from a standin position #ut is a#le to extend the knees from a sittin position! 'upine Qeel for lymph nodes at neck, cla$icle, axillae, roin! Test hip extensors -patient presses le into #ed while you try to raise it.! 'ensation in #oth le s8 li ht touch, pin prick 9 compare medial dorsum of foot -L). with lateral foot -'1. and lateral calf -L). with posterior calf -'1.! Di#ration and position sense in #i toes! 'trai ht le raise8 raise patient+s heel on #ed as far as patient will allow, note an le, note whether this reproduced the patient+s ipsilateral or contralateral radicular pain! ?owstrin test8 flex hip to (4 de rees, extend knee to the point of pain and press on the hamstrin tendon, which is medial, note reproduction of pain! &eripheral $ascular exam8 inspect for $enous stasis or arterial insufficiency ulcers, check femoral pulses and auscultate for femoral #ruits, feel popliteal, dorsalis pedis and ti#ialis posterior pulses! !4$ /D year old .an 'ishes to re*ill a (rescri(tion o* %iorinal *or tension headache$ Manage$ +istory8 description of headache pain, location -one3sided $s! #ilateralBocciput3$ertex @o you feel pain on one or #oth sidesF If one3sided, is it always the same sideF If present on #oth sides, did the pain start on one sideF Is it usually maximal on one sideF., quality -pulsatile $s! non3thro##in %hat kind of pain is it 9 ti htenin , pressin , thro##in , poundin , pulsatin , #urnin , etcF @o different types of pain occur at different times in any one attackF If so, what typesF., intensity, duration -at least 2H hours in mi raine, if not treated., onset includin time of day -mornin headache associated with raised intracranial pressure., pre$ious episodes, a ra$atin Brelie$in factors -e! ! cou hin and strainin worsen headache in raised IC& and chocolate or cheese can tri er mi raines., associated symptoms -aura, nausea, $omitin , photopho#ia 9 li ht, phonopho#ia 9 sound, osmopho#ia 9 odors, nuchal ri idity, weakness, num#ness, $isual distur#ances., medical history, medication history -when was this prescri#ed, do you ha$e the empty #ottle, has it #een prescri#ed #eforeF., current meds, aller ies, family history, su#stance a#use inquiry, smokin , aller ies, mood, stress, anxiety inquiry! =e$iew of systems! Red *lags *or headaches8 must rule out headaches resultin from menin itis, trauma -su#arachnoid hemorrha e, epidural hemorrha e., tumor, temporal arteritis! ;istory8 new onset, headache worse at ni ht, headache wakes patient at ni ht, fe$er, neck stiffness, seiAures, trauma, chan es in L,CB#eha$ior, $omitin , se$ere, $ery youn Bold patients! &hysical exam8 fundi a#normal, Terni B?rudAinski si ns -menin itis., focal neurolo ical findin s! >i$en #eni n history with no suspicion of raised IC& or focal deficits and a description of headache consistent with the common tension headache, a full neurolo ical examination is not indicated! 'u est to the examiner that you would perform a #rief neurolo ical screenin exam! /ou will #e told to mo$e on!

1R

Treat.ent8 Explain that Qiorinal is a com#ination preparation of #ar#iturate -#utal#ital., caffeine and 0'0 which is properly used only for the relief of occasional tension headaches! It is ha#it3formin , can precipitate a withdrawal syndrome includin a itation, delirium and seiAures and has additi$e sedati$e effects with other C"' depressants! The fact that this patient has consumed an entire prescription in four days su ests o$eruse due to dependence! ;e may also ha$e anal esic headache syndrome in which inappropriately used anal esics actually cause headaches! 'u est a dru holiday with weanin from caffeine and alcohol, proper sleep hy iene, diet, exercise and stress mana ement! Chronic headache may also #e a symptom of depression or anxiety, arran e follow up to e$aluate for these if the patient does not impro$e! !#$ 2lderly 'o.an in hos(ital (ost8o( day D o* total hi( re(lace.ent$ ,cute chest (ain& tachycardia& tachy(nea& shortness o* breath$ Manage$ 5orry about8 Life3threatenin causes of acute chest pain8 MI, &E, pneumothorax and tension pneumothorax, aortic dissection! ,ther causes an ina, astritis, reflux, peptic ulcer, pericarditis, herpes Aoster, musculoskeletal! -i**erential -iagnosis *or Chest Pain Cardiac 0n ina &ulmonary MI &neumonia wB pleuritis &ericarditis &neumothorax Myocarditis &E @issectin aorta &ulmonary hypertension Non8Cardiac >I Esopha eal reflux Elcer

M'TB"euro 0rthritis Chondritis =i# fractures ;erpes Soster

&sycholo ic 0nxiety &anic

+istory8 =apid cardiopulmonary history includin any history of hi h #lood pressure, heart pro#lems, smokin , C,&@! Physical e7a.8 Is a cardiopulmonary exam as in question P13 a#o$e with additional attention to risk factors for post3op complications -i!e! inacti$ityBdecreased mo#ility resultin in @DTB&E.! +o.ans sign8 pain in the calf on dorsiflexion of the foot 9 indicates throm#ophle#itis! Check that trachea is midline! Inspect sur ical wound! Is the patient on @DT prophylaxis or anti3coa ulationF Treat.ent8 =aise head of #ed! >i$e oxy en I LBmin #y mask! Monitor oxy en saturation! ,rder stat C?C, lytes, lucose, I"=B&TT, serial CT3M? and Troponin, 0?>, C<=, EC>! >i$e chewa#le 0'0 1I433H) m immediately! 'ecure ID access, #olus ID lasix R4 m , push if fluid o$erload is suspected, and $entolin if wheeAes are heard, i$e su#lin ual nitro spray or 4!3 m su#lin ual nitro if #lood pressure is adequate and 1 m morphine ID! =epeat nitro q)min x 3! May require additional morphine and nitro! =epeat CT3M? and Troponin qKh x 3! 2CG8 if EC> shows si nificant 'T ele$ation -more than one millimeter in two anatomically consecuti$e leads., or a new left #undle #ranch #lock, then the patient is ha$in an MI! ,rder stat Cardiolo y consult for possi#le lytic therapy or cardiac catheteriAation! If less se$ere si ns of ischemia are present -flipped T wa$es, 'T depression., follow with repeat EC>s until resol$ed! S!B3T38 This classic pattern -wide '3wa$e in lead I, Q3wa$es in lead III, T wa$e in$ersion in lead III. with ri ht axis de$iation and =??? are si ns of ri ht heart strain seen in massi$e &E! ,8a gradient8 0n ele$ated 03a -0l$eolar p,H3arterial p,H. radient is a si n of pulmonary em#olus #ut also occurs in any condition in which there is a $entilation3perfusion mismatch -e! ! pneumonia, pulmonary edema, C,&@.! It is determined from the 0?>8 03a O 213 -Qi,H. 9 1!H) -&aC,H. 9 &a,H Unormal8 1H in child H4 in 24 year oldV "ote that the inspired oxy en fraction -Qi,H. is not known unless the patient is on room air, a $entimask or mechanically $entilated! This is #ecause the patient #reathes in a proportion of room air which mixes with the oxy en deli$ered #y face mask or nasal pron s there#y dilutin it #y an unknown amount! =ou hly, howe$er, HLBmin i$es HI* Qi,H, 3LO34*, RLO3)*, and ILOR4*! R4* is considered the maximum inspired oxy en o#taina#le without a hi h flow mask such as a $entimask! ,1G nor.al values8 p; 2!3)32!R), p,H K43144 mm; , #icar#onate HR, pC,H R4

1)

Indications *or intubation8 0n 0?> showin poor p,H -in the I4s, or if less then K4 on hi h inspired ,H concentrations., ele$ated pC,H - reater than K4., acidemia, or >C' M K -not a#le to protect airway. may indicate need for intu#ation if these are not quickly correcta#le! Consult ICE! CFR signs o* P28 wed e3shaped infiltrate -;ampton+s hump. or oli emic area, unilateral effusion, raised hemidiaphra m! 0 normal C<= is also consistent, and usual, with &E! S(eci*ic investigations *or P28 CT chest -can only see &E which is lar e enou h to #e clinically si nificant., DBQ scan -conclusi$e when it shows hi h or low pro#a#ility., and serial -qHd. le @opplers for presence of @DT a#o$e the knee! Treat.ent8 if suspicion of &E is hi h, anticoa ulate -#efore waitin for tests. with heparin 2)44 E ID #olus, then infuse at 1H44 EBh! Measure &TT qIh, ad:ust dose for &TT 243(4s! If a dia nosis of &E is made, coumadin -warfarin. should #e started, continue anti3coa ulation for 3 months! !?$ Houng .other 'ith 4 'ee old baby has recently i..igrated *ro. Ghana$ Poor 2nglish s ills$ Concerned about 'hether she should have her baby i..uni6ed$ Counsel$ Counseling8 >eneral principles of counselin are to #e aware of communication #arriers such as lan ua e difficulties, to understand the patient+s o#:ecti$es, fears, preconceptions, to deal with these in an empathetic, non3:ud mental way and to normaliAe them to transmit information in a way that is consistent with the patient+s expectations and understanda#le to them, and to in$ite further questions and feed#ack! +istory8 0sk if the patient would prefer someone, perhaps a family mem#er to translate! 0sk a#out the patient+s concerns, what does she want to know and whyF Explain that $accines protect children from diphtheria, tetanus, pertussis, polio -@&T&., mumps, measles, ru#ella -MM=., influenAa -;i#. and hepatitis ? -;ep ?.! 0ll of which were once common and caused serious, sometimes fatal illness in Canada, and all of which are now hardly e$er seen #ecause of $accines! Explain that #ecause the $accines stimulate the immune system, some children ha$e a temporary sore arm -with induration and tenderness. at the in:ection site, malaise, mild fe$er or rash! 0ller ic reactions to $accines also occur includin urticaria, rhinitis, anaphylaxis! It is $ery rare to ha$e a more serious reaction -seiAures, encephalopathy ha$e #een reported.! 'tandard modern $accines are not known to cause disease or to ha$e lon 3term deleterious effects! Compare these risks with the risk of not ettin $accinated! Explain the recommended immuniAation schedule -#elow., i$e the patient some information pamphlets, in$ite further questions and ask her to return in two weeks for the child+s first immuniAation! Contraindications to vaccination8 pre$ious serious reaction to $accine! 'pecial contraindication to MM=, which is a li$e attenuated $accine suspended in e white protein and preser$ed with neomycin8 aller y to e or neomycin, pre nancy and immunocompromised state -except healthy ;ID positi$e children.! 'pecial contraindications to the pertussis component of @&T& -which is thou ht to #e the component responsi#le for seiAures and encephalopathic $accination reactions when they occur.8 pro ressi$e neurolo ic disorder and epilepsy! ;i# not to #e i$en after a e )! Td& should not #e i$en in first trimester of pre nancy! Reco..ended I..uni6ation Schedule)
-PTP O diphtheria, tetanus, acellular pertussis and inacti$ated polio, +ib O Hemophilus influenza type #, MMR O measles, mumps, ru#ella, +e( 1 O hepatitis ?, TdP O tetanus, diphtheria toxoid and pertussis, Td O tetanus and diphtheria toxoid, "?8 MM= is administered 'C while the others are IM!

/ .onths > .onths 4 .onths ! year !? .onths >84 years !/8!3 years !>8!4 years E!0yrs

@&T&, ;i# - i$en as &entacel. @&T&, ;i# @&T&, ;i# MM= @&T&, ;i# MM=, @&T& ;ep ? -3 doses 9 initial, 1 month, I months. Td& -certificate of immuniAations for hi h school. Td

!"$ >0 year old 'o.an& a((ears sad& reEuests slee(ing (ills$ Manage$ +istory8 "ame, a e, occupation! 'ocial situation! 'tressful life e$ents! @epression commonly presents with sleep distur#ance therefore screen and treat for depression as in question PR a#o$e! This should #e in addition to a sleep history! Medications, aller ies, dru sBsmokin Balcohol, past medicalBsur ical history, family history, re$iew of systems!

1I

Slee( history8 usual requirements, chronolo y of sleep pro#lems, stressor, sleep hy iene -when, where, re ularity, shifts at work, quiet, late, exercise, meals, alcohol, caffeine, prescription and non3prescription remedies, dru s and medications., sleep latency -time to fall asleep., nocturnal awakenin , early mornin wakenin , daytime somnolence, somnolence while dri$in , workin or durin con$ersation! Pro(er slee( hygiene8 re ular #ed and wake times, a$oid daytime naps, re ular exercise #ut not late in the e$enin , do not use the #ed for readin , TD, paperwork, etc!, a$oid caffeine, alcohol, smokin ! /0$ 40 year old 'o.an 'ith .ulti(le (ains investigated by several other doctors& all lab tests nor.al$ Manage$ +istory o* .ulti(le (ains8 should address the differential for multiple pains! -i**erential *or .ulti(le (ains) -e(ression 'ith so.ati6ation8 ma:or depression presents with a somatic complaintL commonly headache, stomach pains, sleep distur#ance, eatin distur#ance, or #owel ha#it chan es! This is a frequent presentation of depression in the elderly! Treat as in depression -see P1( a#o$e.! So.ati6ation disorder8 multiple non3intentional complaints in multiple or an systems #e innin #efore a e 34 that occur o$er se$eral years, with treatment sou ht and si nificant impairment in functionin ! @ia nostic criteria8 at least K physical symptoms that ha$e no or anic patholo y8 R pain symptoms at R different sites, H >I symptoms other than pain, 1 reproducti$e or sexual symptom other than pain, 1 pseudo3neurolo ical symptom -e! ! temporary #lindness.! Complications8 anxiety, depression, unnecessary medications or sur ery! ,ften a misdia nosis for an insidious illness so rule out all or anic illnesses -e! ! M'.! Treatment8 counselin , psychotherapy, close follow3up, reassurance! Conversion disorder8 psychic pertur#ation presents as one or two neurolo ical complaints affectin $oluntary motor or sensory function! &sycholo ical factors thou ht to #e etiolo ically related to the symptom as the initiation of symptoms is preceded #y conflicts or other stressors! 5La belle indifference6 9 patient+s inappropriately ca$alier attitude towards a serious symptom! Treatment8 anxiolytics -e! ! loraAepam 1 m &, qIh., relaxation therapy, counselin , close follow3up! Pain disorder8 e! ! chronic post3traumatic or post3sur ical pain! &ain not fully accounted for #y current tissue in:ury, exacer#ated #y psychic factors and associated with functional impairment! Treatment8 amitriptyline H)32) m &, q;'! +y(ochondriasis8 exa eration or misinterpretation of normal sensory phenomena to the point of functional disa#ility! E$idence does not support a physical disorder! 0ssociated with o#sessi$e fear of serious illness and doctor shoppin despite reassurance! ?elief is not delusional as person acknowled es unrealistic interpretation! Treatment8 counselin , reassurance, close follow3up! %ibro.yalgia8 also called fi#rositis and fi#romyositis! K43(4* of cases occur in middle3a ed females, may afflict )* of adult women, typically cardio$ascularly unfit, depressed, pre$iously normal life -onset often after car accident.! 0ssociated with a#sent or decreased non3=EM sta e R sleep, patients wake from sleep feelin unrefreshed! Constant, achin , axial pain with #ilateral tender points -not tri er points, at which referred pain is tri ered due to myofaschial pain from o$eruse, e! ! tennis el#ow.! The disorder follows a waxin and wanin course ultimately without pro ression or resolution, and may #ecome disa#lin ! Characteristic reproduci#le tender points are located #ilaterally at lateral #order of sternum, sternocleidomastoid, posterior neck, trapeAius, rhom#oids, o$er sacroiliac :oints, lateral thi h, posterior and medial knee! &atient should ha$e ele$en of the a#o$e ei hteen tender points for a dia nosis! Treatment8 amitriptyline H)32) m &, q;'! Chronic *atigue syndro.e8 similar to fi#romyal ia #ut fati ue is the dominant feature and pain and tender points may #e less prominent or a#sent! Treatment8 amitriptyline H)32) m &, q;'! %actitious disorder or .alingering8 Qactitious disorder in$ol$es misrepresentation of history and symptoms for the purpose of assumin the sick role with its inherent secondary ains -attention and sympathy, :ustification for inadequacies.! Munchausen+s syndrome is the type of factitious disorder in which physical findin s are faked #y contamination of la# tests or in estion of inappropriate medication or su#stances! Typically the patient is a medical paraprofessional moti$ated #y hostility toward the medical esta#lishment, e! ! nurse takes coumadin to fake hemophilia! Malin erin is distin uished from factitious disorder #y a moti$ation for secondary ain other than the sick role, such as insurance #enefits! Treatment8 counselin ! +istory8 &ain description, location, duration, chronolo y, a ra$atin and relie$in factors, are pains linked to one anotherF

12

So.ato*or. disorders screen8 ;ow has your health #een for most of your lifeF ;ow ha$e your pains affected your :o#, social life, relationships, and your life enerallyF 0re you often unwell, how often do you $isit the doctorF @o you worry that you ha$e a serious illnessF If a doctor tells you that there is nothin wron , how does that make you feelF @o you #elie$e him or herF ,ssociated sy.(to.s8 re$iew of systems, medications, aller ies, smokin , alcohol, dru use, family history, depression history as in PR a#o$e! -iagnosis and treat.ent8 Qor non3specific pains with depressi$e symptoms the patient most likely has depression with somatiAation! Treat for depression as in PR! /!$ , young .an (resents to the 2.ergency -e(art.ent having t'isted his an le$ Manage$ +istory *or an le s(rain8 history of a plausi#le mechanism of in:ury in$ol$in si nificant in$ersion or e$ersion of the foot with pain and swellin ! Time of in:ury, onset of pain and swellin -may #e delayed., noises heard at time of in:ury! &re$ious ankle or other in:uries! 0#ility to walk post in:ury -often preser$ed if li aments are not ruptured.! &ast medical history, medications, aller ies, family history! Physical e7a.8 inspect for ross deformity, erythema, swellin , #ruisin ! Check distal circulation, sensation, acti$e and passi$e ran e of motion, palpate for tenderness at :oints! Examine the :oints a#o$e and #elow the affected :oint! Identify sites of maximal tenderness! &oint tenderness o$er the area anterior -anterior talofi#ular li ament., inferior -calcaneofi#ular li ament., or posterior -posterior talofi#ular li ament. to the lateral malleolus are si ns of lateral li amentous in:ury! Tenderness o$er the area medial and inferior to the medial malleolus indicates deltoid -medial. li ament in:ury! Talar dra'er sign8 'ta#iliAe the ti#ia and pull forward on the heel, talar drawer si n is anterior mo$ement of the talus! >reater than 3 mm anterior mo$ement may #e si nificant! 1 cm is si nificant and indicates anterior talofi#ular li ament rupture! Talar tilt8 'ta#iliAe the ti#ia, rasp the talus and tilt in in$ersion and e$ersion! Mo$ement #eyond the normal ran e compared with the opposite side is a positi$e talar tilt and indicates lateral calcaneofi#ular li ament rupture if the tilt occurs in in$ersion or medial -deltoid. li ament rupture if the tilt occurs in e$ersion! SEuee6e test8 &ain in the ankle on squeeAin the calf is a si n of ankle fracture! Otta'a ,n le Rules8 for takin ankle series x3rays -includes lateral and 0& ankle with mortis $iew.! <3ray if there is pain o$er the malleolar Aone 0"@ tenderness on palpation of the posterior medial or lateral malleolus ,= if the patient is una#le to wei ht #ear immediately and in E=! Qor foot series -0& and lateral foot.8 <3ray if there is pain in midfoot Aone 0"@ #ony tenderness o$er the na$icular or #ase of )th metatarsal ,= una#le to wei ht #ear immediately and in E=! Calcaneal $iews if there is pain on palpation of heel! Treat.ent *or an le s(rain8 -remem#er =ICE. Rest, use crutches, a$oid wei ht #earin #ut early mo#iliAation! Ice for H4 minutes QI@ for H33 days! Consider "'0I@s8 I#uprofen R443I44 m &, qIh if no contraindications! Compression with tensor #anda e or tape #ut not to the point of pain! 2le$ate! =eha#ilitation8 start acti$e ran e of motion exercises H days post in:ury, may wei ht #ear after pain and swellin ha$e su#sided! Qull li ament healin may take I weeks in se$ere in:ury or more if re3in:ury occurs! Complete li ament rupture with :oint insta#ility -positi$e talar drawer si n or talar tilt. should #e e$aluated #y ,rthopedics! //$ !4 year old no'n e(ile(tic on -ilantin is having 3 sei6ures (er .onth and reEuests better .edication$ Manage$ %indings) not ta ing .eds& e7(eriencing stress$ 'ee question P3 a#o$e! /3$ D0 year old 'o.an 'ith headache and nor.al vitals$ Ta e a history$ B) -escribe a((ro(riate investigations and treat.ent *or te.(oral arteritis$ +istory8 description of headache pain, location -one3sided $s! #ilateralBocciput3$ertex @o you feel pain on one or #oth sidesF If one3sided, is it always the same sideF If present on #oth sides, did the pain start on one sideF Is it usually maximal on one sideF., quality -pulsatile $s! non3thro##in %hat kind of pain is it 9 ti htenin , pressin , thro##in , poundin , pulsatin , #urnin , etcF @o different types of pain occur at different times in any one attackF If so, what typesF., intensity, duration -at

1K

least 2H hours in mi raine, if not treated., onset includin time of day -mornin headache associated with raised intracranial pressure., pre$ious episodes, a ra$atin Brelie$in factors -e! ! cou hin and strainin worsen headache in raised IC& and chocolate or cheese can tri er mi raines., associated symptoms -aura, nausea, $omitin , photopho#ia 9 li ht, phonopho#ia 9 sound, osmopho#ia 9 odors, nuchal ri idity, weakness, num#ness, $isual distur#ances.! 0 history of unilateral lancinatin pain with swellin and tenderness in the temporal area should prompt inquiry after symptoms of polymyal ica rheumatica -&= 9 pain and stiffness in muscles of neck, shoulders, upper arms, hips, lower #ack and thi hs no weakness or atrophy increased E'=, anemia, normal CT responds to steroids immediately., which is related to temporal - iant cell. arteritis and may #e a more systemic $ariant of the same underlyin disease! 'ymptoms of #oth &= and temporal arteritis include low rade fe$er, malaise, anorexia, wei ht loss, #ilateral proximal muscle weakness, achin and pain, as well as :oint inflammation! Naw claudication, stroke and #lindness may occur due to $asculitic occlusion of arterial supply! 0sk a#out $isual chan es! Medical history, medication history -when was this prescri#ed, do you ha$e the empty #ottle, has it #een prescri#ed #eforeF., current medications, aller ies, family history, su#stance a#use inquiry, smokin , aller ies, mood, stress, anxiety inquiry! =e$iew of systems! Investigations *or te.(oral arteritis 8 C?C -mild anemia with increased %?C., E'= - reater than )4 mmBh, Unormal 34V., C3 reacti$e proteins, li$er enAymes, temporal artery #iopsy, may add temporal artery an io ram to uide #iopsy! Treat.ent8 -in the a#sence of $isual symptoms. without waitin for #iopsy, start hi h dose oral prednisone I4 m &, ,@ until symptoms su#side and E'= normal, then R4 m &, ,@ for R3I weeks, then taper to )314 m &, ,@ for H years -relapses occur in )4* if treatment is terminated #efore H years.! Treatment does not alter #iopsy results if the sample is taken within H weeks! Monitor E'= re ularly! If $isual symptoms are present, or de$elop durin treatment, the patient is admitted and i$en prednisolone 1444 m ID q1Hh for ) days! />$ +I< (ositive .an$ ! 'ee o* shortness o* breath& cough& *atigue$ Per*or. a (hysical e7a.$ B) Give a di**erential diagnosis *or a CFR sho'ing a *ine reticular (attern in the le*t lo'er lobe$ Manage$ Physical e7a.8 0 physical exam for query pneumonia consists of the cardiopulmonary exam as in question P 13 with additional attention to the particular si ns and symptoms of ;ID infection! 27a.ination *or lobar consolidation8 In eneral, pulmonary effusion decreases transmission of #reath and $ocal sounds to the chest wall, while consolidation -seen in pneumonia. increases it! Qour maneu$ers #rin out the effect of increased transmission8 tactile *re.itus is increased transmission of palpa#le fremitus to the chest wall while the patient repeats 5ninety3nine6, broncho(hony is enhanced transmission of spoken words such as 5ninety3nine6, ego(hony is a chan e from an 5ee6 to an 5ay6 sound o$er the affected area while the patient sustains an 5ee6 sound, and 'his(ered (ectoriloEuy is a marked increase in audi#ility throu h the chest wall o$er the affected area while the patient whispers the words 5ninety3nine6 or 5one3 two3three6! Signs o* consolidation8 increased tactile fremitus, percussion dullness, crackles, #ronchial #reath sounds, increased $oice transmission -#ronchophony, e ophony, whispered pectoriloquy.! Signs o* +I< In*ection8 -and possi#le impendin 0I@'. check entire skin surface for Taposi+s sarcoma, examine pharynx for thrush or oral hairy leukoplakia -Epstein3?arr $irus3related epithelial proliferation causin raised white plaques on the sides of the ton ue., palpate neck, cla$icle, axillae, and roin for lymph nodes enlar ed #y non3;od kin+s lymphoma! Examine a#domen for hepatic or splenic enlar ement! -i**erential diagnosis of unilateral lo#ar reticular pattern on C<=8 pneumocystis carinii pneumonia -&C&., Taposi+s, lymphoma, cytome alo$irus -CMD., tu#erculosis, Cryptococcus neoformans, ;emophilus, 'treptococcus, mycoplasma, chlamydia! The classic C<= of &C&, an 0I@'3definin illness, is #ilateral hilar infiltrates, #ut <3ray findin s are $aria#le and may #e al$eolar or interstitial! Investigations8 ,H satsB0?>, C?C with differential and C@R count, L@; -ele$ated in ()* of &C& pneumonias and not in other pneumonias., #lood cultures, sputum for cytolo yB ram stainBcultureBT? stain if sputum a$aila#le -cou h usually non3 producti$e and induced sputum may fail., #ronchoscopy with cytolo y, ram stain and culture of #ronchial washin s and #rushin s -may see #ronchial Taposi+s.! Treat.ent8 'eptra @' H ta#s qKh x 1R days outpatient with 1 ta# ,@ or ?I@ 3Bweek continued as prophylaxis! More specific therapy with results of dia nostic tests! In se$ere illness, admit to hospital, i$e ID 'eptra at same dose and &rednisone R4 m ?I@ x ) days, then ,@ x ) days, then H4 m ,@ for prophylaxis! &atient should #e referred to an 0I@' specialist for anti$iral and possi#le experimental therapies! Counselin and referral to support or aniAations! Qollow up appointment!

1(

/D$ , / day old in*ant has seru. bilirubin //0 I.ol/L :re*$ Ma7 /00 I.ol/L;$ Ta e a history *ro. the .other$ B) 5hat are the (ossible causes *or this abnor.alityC Give investigations and treat.ent$ )4* of term infants de$elop $isi#le :aundice -J K)31H4 WmolBL or )3I m BdL. Mothers obstetrics history8 >T&0L -num#er of estations, term pre nancies, premature #irths, a#ortions, li$e children., history of pre$ious pre nancies includin neonatal :aundice, maternal medical history esp! li$er disease, illness durin pre nancy esp! dia#etes -lar e #irth wei ht, pre3eclampsia., ru#ella -terato enic., toxoplasmosis -from cats, infects fetal #rain., herpes -infects fetus, frequently fatal., CMD -dama es fetal li$er., terato enic medications taken durin pre nancy, dru and alcohol use, maternal #lood type, complications of present pre nancy includin estational hypertension or dia#etes, hyperBhypothyroid, hypercoa ulation! Qamily history of neonatal :aundice, li$er pro#lems! Ne'born history8 estational a e at #irth, caesarean, induction, rupture of mem#ranes artificial or prolon ed, fetal distress, forceps or $acuum deli$ery, meconium, 0&>0=s, was resuscitation requiredF Initial #lood work, #reast feedin F ;ow often and how well, color of 1st stool, color of urine, $omitin , neonate muscle tone, #eha$iors, fe$er, irrita#ility, lethar y! Causes o* neonatal 9aundice8 uncon:u ated O physiolo ic neonatal :aundice ,= patholo ic8 hemolytic 0?,=h incompati#ility, neonatal sepsis, splenome aly, hereditary spherocytosis, >I&@ etc!L non3hemolytic #reast milk :aundice, #reakdown of cephalohematoma, polycythemia, sepsis, >il#erts, Cri ler3"a::ar, hypothyroidism! Con:u ated8 >I o#struction in fetus -increases enterohepatic circulation., #ile duct o#struction, dru 3induced and multiple other less common causes! Investigations8 Ese #lood o#tained #y $enipuncture, not heal prick, as slud in of heel prick #lood in skin capillaries distorts cell counts and concentrations! Measure direct -con:u ated. and indirect -uncon:u ated. #iliru#in, neonatal and maternal #lood types, Coom#+s test -see #elow., #lood smear, C?C with reticulocyte count! 'eptic work3up, urinalysis, #lood cultures, C<=, 0<=, C'Q! Coo.bs test8 a two3part test! 1! @irect anti lo#ulin test8 detects presence of anti3red cell autoimmune anti#odies attached to red cells! &atient+s red cells a lutinate when mixed with a solution containin anti3human immuno lo#ulin anti#odies, i!e! multiheaded anti3immuno lo#ulin anti#odies #ind to the constant chains of anti#odies on the red cell surface, stickin red cells to ether! H! Indirect anti lo#ulin test8 detects presence of anti3red cell anti#odies in patient+s serum! &atient+s serum incu#ated with red cells of the same #lood roup, test is positi$e if a lutination occurs! Treat.ent8 The aim of treatment is to correct anemia and decrease hyper#iliru#inemia! 'e$ere hyper#iliru#inemia may lead to ernicterus -deposition of #iliru#in in the #rainstem and #asal an lia leadin to mental retardation, cere#ral palsy, hearin loss and paralysis of upward aAe.! Indications for inter$ention8 In eneral, serum total #iliru#in le$el reater than 344 umBL is an indication for phototherapy, while le$els reater than R44 umBL call for exchan e transfusion or plasmaphoresis! Thera(eutic .odalities8 In phototherapy photoisomers of #iliru#in are produced which are more water3solu#le and can #e excreted without con:u ation, while exchan e transfusion replaces the #a#y+s #lood with donor #lood! &lasmaphoresis is the replacement of #lood plasma with donor plasma while retainin the patient+s own red and white cells! S(eci*ic treat.ent o* 9aundice de(ends on the cause8 Physiologic 9aundice8 occurs on day H33 in )4* of term infants -"EDE= within 1st day. and resol$es #y day 2! It is more common in preterm infants where it occurs later, up to day I, and lasts lon er! @ue to transient limitation in #iliru#in con:u ation -i!e! the hyper#iliru#inemia is uncon:u ated or indirect.! Esually requires no treatment! ,1ORh inco.(atibility 9aundice8 =h incompati#ility -=h ne ati$e mother with =h positi$e fetus. tends to #e more se$ere than other incompati#ilities -0?, incompati#ility8 e! ! , mother with 0 or ? fetus. and may cause hydro(s *etalis - eneraliAed edema, includin pulmonary edema, with hi h output heart failure.! Treatment for all types is exchan e transfusion or plasma electrophoresis! Se(sis8related 9aundice8 treat underlyin cause and use phototherapy! 1reast .il 9aundice8 occurs day R32 and peaks at Hnd to 3rd week, rare -18H44., lon chain fatty acids in #reast milk competiti$ely inhi#it lucuronyl transferase acti$ity! Treatment is to su#stitute formula for #reast milk for H3R days, then resume!

H4

/4$ , D0 year old .an is denied li*e insurance because o* abnor.al liver *unction tests$ ,ST J ,LT very elevated& ,LP slightly elevated& 1ili nor.al$ Ta e a history$ B) Give a di**erential diagnosis$ 5hat investigations 'ould you orderC +istory8 "ame, a e, occupation! ;istory of :aundice, hepatitis, forei n tra$el, #lood transfusions, recreational ID dru use! @ark urine, pale stool, a#dominal pain, fe$erBchills, decreased appetite, wei ht loss, ni ht sweats, nausea and $omitin , pruritis, easy #ruisin , ynecomastia, hemorrhoids -from portal hypertension., alcohol use 7 C0>E questionnaire -see #elow.! 'exual history8 num#er of past and present partners, enders of same, sexually transmitted disease! Medications, dru use, smokin , aller ies, past medical history, family history, re$iew of systems! C,G2 Euestionnaire8 Control 9 ha$e you tried to cut down on your alcoholF 0n er 9 ha$e you e$er felt an ry when someone su ested you decrease your alcohol intakeF >uilty 9 ha$e you e$er felt uilty a#out your drinkin F Eye opener 9 do you sometimes ha$e a drink to et started in the mornin F Liver 2n6y.es8 0'T 7 0LT are sensiti$e #ut non3specific markers of hepatocellular dama e hepatitis -inflammation. $ascular in:ury -ischemia.! 0'T J 0LT O alcoholic li$er diseaseL 0LT J 0'T O $iral hepatitis! 0L& 7 >>T are markers of cholestatic disease intrinsic -toxic, infectious, inflammatory., systemic -sepsis, pre nancy., infiltrati$e -tumor, fat, lymphoma., mass lesions -stone, tumor, a#scess.! 0'TB0LT J H -0'T usually M 344. alcoholic li$er! 'erum transaminases J 1444 due to 1. $iral hepatitis, H. dru s, 3. passa e of common #ile duct stone, R. hepatic ischemia! -i**erential8 alcoholic li$er disease, $iral hepatitis, li$er mali nancy -metastatic or primary.! Investigations8 Diral serolo y -;ep 0, ?, C anti#ody and ? anti en 9 presence of ? anti en for J I months indicates chronic carrier state., >>T, 0'T, 0LT, 0lk&hos, L@;, #iliru#in, I"=B&TT, al#umin, lucose -cirrhosis., serum ceruloplasmin, serum copper -%ilson+s disease., serum ferritin, total iron #indin capacity -TI?C, for hemochromatosis., 0"0, anti3smooth muscle anti#ody -autoimmune hepatitis, also called chronic acti$e hepatitis., a#dominal ultrasound, li$er #iopsy! /#$ !" year old *e.ale 'ith vaginal discharge$ Ta e a history$ B) Give three (ossible diagnoses& 'hat investigations 'ould be hel(*ulC +istory8 "ame, a e, occupation, description of dischar e, onset, chronolo y, pre$ious episodes, $olume, color, consistency, odor, timin -related to mensesF.! 0ssociated symptoms8 pain includin a#dominal, #urnin , fe$er, itch, dyspareunia, dysuria, ur ency, frequency, a ra$atin and relie$in factors! 'exual history8 num#er of past and present partners, ender, type of contraception -condoms., possi#ility of pre nancy, past history of sexually transmitted disease! ,#stetricsB ynecolo y history8 ->T&0L. pre nancies, a#ortionsBmiscarria es, #irths, pap smears -normalF., menstrual pattern! Medications -especially anti#iotics., oral contracepti$es, other dru use, aller ies! &ast medical history includin dia#etes! Qamily history, re$iew of systems! Causes o* discharge8 &hysiolo ical8 normal midcycle dischar e, increased estro en states! Infectious8 candidiasis, #acterial $a inosis ->ardnerella $a inalis., trichomonas infection, chlamydia, onorrhea -",TE8 onorrhea and chlamydia can cause cer$icitis, &I@ and urethritis, #ut do not cause $a initis 9 #ut Toronto "otes includes them in the differential for $a inal dischar e., #artholinitis or ?artholin a#scess, &I@! "eoplastic8 $a inal intraepithelial neoplasia -D0I"., $a inal squamous cell carcinoma, in$asi$e cer$ical carcinoma, fallopian tu#e carcinoma! ,ther8 aller icBirritati$e $a initis, forei n #ody, atrophic $a initis, entero$a inal fistulae! Investigations8 speculum exam, swa# and culture, saline slide microscopy and T,; whiff test -add T,; to $a inal secretions on a slide.! These i$e results as follows8 Candidiasis8 inflamed appearance, lumpy white dischar e, spores and pseudohyphae seen under microscope! Treatment8 miconaAole $a inal suppository! 1acterial vaginosis8 non3inflamed, thin ray secretions, clue cells under microscope -epithelial cells with o#scured #orders due to adherence of #acteria., fishy odor on T,; test! Treatment8 metronidaAole )44m &, ?I@ x 2 days -in pre nancy use 0moxicillin )44m TI@ x 2 days.! Tricho.onas8 inflammations, frothy yellow3 ray3 reen dischar e, motile trichomonads seen under microscope! Treatment8 metronidaAole H x 1 or )44m &, ?I@ x 2 days -in pre nancy use ClotrimaAole $a inal suppositories.! /?$ 40 year old .an 'ith .icrosco(ic he.aturia on routine urinalysis$ Ta e a history$ B) Give a di**erential diagnosis& 'hat investigations 'ould be hel(*ulC

H1

2tiology o* +e.aturia by ,ge Grou( ,ge 2tiology :in order o* decreasing *reEuency; 08/0 >lomerulonephritis, ETI, con enital anomalies /08>0 ETI, stones, #ladder tumor >0840 Male8 #ladder tumor, stones, ETI Qemale8 ETI, stones, #ladder tumor J 40 Male8 ?&;, #ladder tumor, ETI Qemale8 #ladder tumor, ETI +istory8 patient I@, suprapu#ic pain, pain on urination, frequency, ur ency, frank #lood in the urine - lo#ular clots from #ladder or strin shaped clots from ureters., color of urine, difficulty initiatin or maintainin urinary stream, renal pain, #ack pain, roin pain! &ro$okin factors -e! ! exercise, trauma.! ;istory of recent ETI, 'T@s, T? exposure, pel$ic irradiation, #leedin diathesis, smokin ! Qe$er, chills, nausea, fati ue! &re$ious renal colicBdia nosed nephrolithiasisF ;istory of hypercalcemia, hypertension! Mali nant symptoms8 ni ht sweats, wei ht loss, fati ue! Medications, dru s -"'0I@s, anticoa ulants.Balcohol, smokin , anticoa ulants and salicilates, past medical history, past sur ical history, family history -polycystic kidney diseaseF., re$iew of systems! -i**erential diagnosis8 ?&;, transitional cell carcinoma of #ladder, ETI, nephrolithiasis, hydronephrosis, prostatitis, prostate cancer, renal cell carcinoma, lomerulonephritis, trauma -heart $al$e, muscle in:uryB#reakdown., essential hematuria -tends to occur in children.! ",TE8 microscopic hematuria is normal in 14* of population! Investigations8 prostate specific anti en -&'0., C?C -to rule out anemia, leukocytosis., urine for microscopy -casts, crystals, culture and sensiti$ity, cytolo y., a#dominopel$ic ultrasound, cystoscopy, intra$enous contrast uro raphy, intra$enous pyelo ram -ID&.! /"$ /D year old .ale 'ith a history o* dys(e(sia and binge drin ing has abdo.inal (ain$ Per*or. a *ocused (hysical e7a.$ B) 5hat radiological investigations 'ould you order and 'hyC Physical e7a. *or abdo.inal (ain8 $itals, posture -unmo$in in fetal position su ests peritonitis while writhin su ests renal colic., :aundice, nutritional status, #uccal mucosa, teeth, #reath -hepatic fetor., parotid hypertrophy, lossitis, inspect chest for telan ectasia, ynecomastia, loss of axillary hair! ;ands8 palmar erythema, clu##in , @upuytren+s contracture, wastin of hand intrinsics! ,bdo.inal e7a. :su(ine;8 inspect for caput medusa, Cullen+s si n -um#ilical #ruisin intraperitoneal #leed., >ray3 Turner+s si n -#ruisin of flank retroperitoneal #leed., pulsations, auscultate for #owel sounds, renal #ruits! Estimate siAe of li$er and spleen #y percussion! &alpate for li$er ed e, Murphy+s si n -examiner presses on li$er ed e after patient has exhaled, patient catches #reath on inspiration., splenic enlar ement -#e in palpation at ri ht lower quadrant to catch $ery lar e spleen and percuss at Trau#+s space., hard stool in #owel! "ote cou h tenderness, ri idity, re#ound, uardin , tenderness, pain at Mc?urney+s point -one third of the way alon the line #etween the ri ht anterior iliac crest and the um#ilicus.! =o$sin +s si n8 palpation of the LLQ produces =LQ pain! &soas si n8 pain on passi$e or acti$e flexion at the hip, indicates peritoneal irritation o$er the psoas or psoas a#scess! ,#turator si n8 pain on internal or external rotation of the hip, indicates #owel herniation into the o#turator canal! 0ssess for ascites! 0sk patient to roll onto side and pound costo$erte#ral an les li htly with fist, CD an le tenderness indicates kidney pain due to pyelonephritis or nephrolithiasis! &alpate roins for hernias! Rectal8 palpate prostate, rectal shelf, check for ross or occult #lood! Radiological Investigations8 ,bdo.inal 3 vie's8 supine, upri ht, left lateral decu#itus! @ilated #owel with multiple airBfluid le$els indicates ileus! @ilated proximal #owel with collapsed distal #owel indicates o#struction! Check for astric distension! 'mall #owel has circular plica8 lines o all the way across! Lar e #owel has interrupted haustra8 lines o halfway across! Check for calcified kidney stone, fecolith and appendiceal airBfluid le$el! Can see allstones and a#dominal aortic aneurysm if calcified! ,bdo.inal ultrasound8 allstones, cholecystitis, pancreatitis, appendicitis, hydronephropathy, kidney stones, a#dominal aneurysm! CFR8 check for free air under the diaphra m in an upri ht film! 30$ #0 year old .ale 'ith nec (ain and le*t ar. 'ea ness$ Per*or. *ocused (hysical e7a.$ %indings) decreased sensation over le*t inde7 and .iddle *inger& .ild 'rist e7tensor and trice(s 'ea ness$ B) -escribe a cervical s(ine *il. o* the (atients nec :sho's narro'ing o* C48# dis s(ace;$ -iagnosis and treat.entC

HH

Physical e7a. *or nec (ain8 uided #y differential for the causes of neck pain8 .usculos eletal, discogenic, stenotic, .alignant or brainste. in*arct! Take $itals, cranial ner$es8 pupillary reflexes, extra3ocular mo$ements, $isual fields, facial muscles -frown, raise eye#rows, show teeth, protrude ton ue., facial sensation, a , ;orner+s triad -ptosis, anhidrosis, miosis on side of sympathetic palsyF., sternocleidomastoid and trapeAius power! Cere#ellar testin 8 fin er3nose, heel3shin, dysdiadocokinesis, ait, =hom#er , &ronator drift! Nec 8 inspect for lesions, asymmetry, muscle wastin , especially sternocleidomastoids, palpate for nodes, masses, palpate dorsal $erte#ral spines, ran e of motion! Shoulders& ar.s& hands8 inspect for symmetry, wastin , fasciculations, skin lesions! &ower8 Test deltoids -C)., #iceps -CI., triceps and wrist extension -C2., hand intrinsics -CK.! "ote that each muscle roup actually has mixed ner$e root inner$ation, i!e! deltoids and #iceps -C),I., triceps -CI,2,K., wrist extension -CI,2., hand intrinsics -CK,T1.! 'ympathetic outflow occurs at CK, T1! 'ensation8 check pinprick, $i#ration, li ht touch o$er the shoulder -C)., thum# -CI., index and middle fin er -C2., rin and little fin er -CK.! @eep tendon reflexes at #iceps, triceps, #rachioradialis, ;offman+s si n -the ?a#inski of the upper lim#8 flick relaxed index fin er dorsally, thum# a#ducts for positi$e test.! ToneBri idity8 check for increased tone #y rapid supination and rapid extension of el#ow! Lateral Cervical -isc Syndro.e C>8D Root involved C) Motor @eltoid 'upraspinatus ?iceps Re*le7 'upinator Sensory 'houlder CD84 CI ?iceps ?iceps Thum# C48# C2 Triceps Triceps Middle fin er C#8T! CK @i ital flexors Intrinsics Qin er :erk =in fin er Little fin er

Peri(heral nerves8 Check two point discrimination at each fin ertip! To determine peripheral ner$e dama e8 Median ner$e territory is the palmar surface of the thum#, and the palmar surface and dorsal tips of the index, middle and thenar side of the rin fin ers! 'ensation to the ulnar side of the hand is the ulnar ner$e, and the dorsal surface of the thenar side of the hand is radial ner$e inner$ated! The median ner$e also inner$ates most muscles of the thenar eminence, and the 1st and Hnd lum#ricals! The thum# is weak in a#duction at (4 de rees to the plane of the hand in median ner$e dysfunction! Tinel+s si n8 tappin the palmar surface of the wrist elicits shootin paresthesia in median distri#ution! &halen+s si n8 maximally flexin #oth wrists #y pushin the dorsi of the hands to ether elicits median ner$e distri#ution num#ness or paresthesias after 343I4 seconds! C8s(ine F8rays8 Lateral8 an adequate lateral shows the top of the T1 $erte#ra8 look for ali nment of the anterior and posterior mar ins of $erte#ral #odies as well as spinous processes! 'pinous processes may ha$e a#normal separation in in:ury! The maximal normal distance #etween the posterior aspect of the anterior arch of C1 and the dens is 3mm in adults and )mm in children! Look for re ularity of disk space hei ht, as in the disk space -su ests de eneration., osteophytes, pre3$erte#ral swellin reater than one third of the $erte#ral #ody width -2 mm from C13R, HHmm from C)32.! ;an man+s fracture8 coronal plane fracture throu h the #ase of #oth pedicles of CH, caused #y hyperextension in:ury, separates the posterior elements of CH from its #ody! 0&8 check ali nment of processes and $erte#ral #odies, distance #etween spinous processes should #e re ular, pedicles should #e seen in cross section -erosion of a pedicle can cause the 5winkin owl si n6 where the pedicles are the eyes and the spinous process, the #eak.! ,dontoid $iew8 trace #one cortex around the outline of the dens, misali nment of this outline indicates odontoid fracture, articular spaces of atlantoaxial :oints on either side of the dens should #e equal! "ote8 odontoid fracture type I 9 tip, type II 9 #ase, type III 9 throu h #ody of CH! -iagnosis8 0 narrowed CI,2 disk space su ests disk de eneration at that le$el! CI,2 disk herniation would impin e on the C2 ner$e root -cer$ical roots exit a#o$e the $erte#ra of the same num#er. which is consistent with clinical CI,2 ner$e root dysfunction on sensory and motor exam! Treat.ent8 most patients respond to conser$ati$e therapy8 soft collar, "'0I@, acetaminophen! If symptoms persist for two weeks or neurolo ic symptoms pro ress, refer to "eurosur ery for myelo ram, CT neck and possi#le CT myelo ram, M=I, or EM>, ner$e conduction studies! May require decompressi$e laminectomy or anterior discectomy with #one raft fusion! 3!$ , 30 year old (atient 'ith ty(e I diabetes (resents to the e.ergency de(art.ent 'ith abdo.inal (ain and vo.iting$ Ta e a history$ B) Labs) Glucose /D& K 4$0& (+ #$//& 1icarb !>$ 5hat is your diagnosis and .anage.entC

H3

+istory *or abdo.inal (ain and vo.iting8 quality of the pain, location, onset, chronolo y, radiation, associated symptoms, a ra$atin and relie$in factors! "um#er of episodes of $omitin , description of $omit, presence of #lood and #ile! 0ssociated prodromal illness, fe$er, malaise, sore throat, cou h, urinary symptoms, diarrhea! Qoods eaten, other people sickF &re$ious similar episodeF &olydipsia, polyuria, lethar y, anorexia, hyper$entilationF ,ther precipitants of @T0, recent sur ery, recent trauma, pre nancy, MI, non3compliance or wron insulin dose, infection! -iabetes history8 time since dia nosis, medications, #lood su ar monitor at homeF @ia#etic control, polyuria, polydipsia, diet, exercise, dru s, alcohol, smokin , complications of dia#etes -retinopathy, neuropathy, nephropathy, infections.! %ho follows patient+s @MF ;as patient taken insulin since feelin unwell, last insulin doseF &ast medical history, current medications, aller ies, family history, re$iew of systems! -iagnosis8 dia#etic ketoacidosis! Manage.ent8 Qoley, ID, lytes, lucose, 0?>, serum ketones! 'eptic work3up8 C?C, C<=, #lood cultures, urinalysis, EC> if T is critically ele$ated! 1 L "' per hour x H33 hours or until tachycardia and ?& normaliAe, then )44 ccBhr x H hours, then H)4 ccBh x ) hours! Insulin drip at H EBhr! Check lucose and lytes qHh! %hen lucose drops to 1), switch fluids to maintenance H B331B3 @)%B"' C H4 mEq TClBL -R8H81 rule.! ?e in diet and re ular insulin re imen! If the @T0 was the result of non3 compliance close follow3up and education such as diet and dia#etes mana ement counselin with a dietitian are required! 3/$ , .other 'ith her 4 'ee old 'ho has been vo.iting *or three days$ Ta e a history$ B) Investigations sho' a (al(able .ass in the right e(igastriu.& .etabolic hy(ochlore.ic al alosis$ 5hat is the diagnosisC Give a di**erential diagnosis *or vo.iting in an in*ant$ +istory o* in*ant vo.iting8 a e of onset, duration, se$erity, chronolo y, association with feedin or #ody position, description of force, $olume, color, composition -#ilious, fecal, #lood, re ur itant., ettin worse or #etter, is child still hun ry afterward, or does he settle! Cou hin or a in with feeds -tracheoesopha eal fistula.! 0ssociated diarrhea, constipation, fe$er, wei ht loss, a#dominal distention, urination! 0re other children sickF ;as child #een in contact with an infected person! Mothers obstetrics history and ne'born history8 as in question PH) a#o$e -evelo(.ent history8 a e and wei ht normo rams, feedin history8 quantity, frequency, #reast $s! #ottle -which formula., colic, feedin difficulties! &ast medical history, medications, family history! -iagnosis8 pyloric stenosis! -i**erential diagnosis *or in*ant vo.iting8 "ew#orn8 con enital malformation -pyloric stenosis, tracheoesopha eal fistula, duodenal atresia, malrotation of the intestine.! &ost new#orn period8 astroenteritis, peritonitis, appendicitis, hepatitis, ulcers, pancreatitis, o$erfeedin , reflux, food aller y, milk protein intolerance, systemic infection!

!""D
33$ #" year old *e.ale colla(ses in the .all$ Patient is dro'sy& unres(onsive to verbal sti.uli$ Manage$ %indings) +R >0& 1P ?0/>0& 2CG co.(lete heart bloc $ Rescusitation8 0TL'B0CL' format as in question PI a#o$e! Manage.ent o* co.(lete heart bloc 8 -& wa$es seen on EC> not related to Q=' complexes.! Transcutaneous pacin -atropine 1 m ID may #e tried #ut is rarely effecti$e.! &atient will require sedation -midaAolam H m ID. and anal esia -morphine H m ID. #efore startin external pacin ! %ill require placement of a trans$enous pacer until a permanent pacer can #e placed! Consult Cardiolo yBCCEBICE! Causes o* ,< conduction abnor.alities8 calcification of the conductin system -Le$+s and Lene re+s disease., inferior MI, coronary spasm, di italis o$erdose, tricyclic antidepressant o$erdose, X3#lockers, calcium channel #lockers, $iral rheumatic fe$er, Lyme disease, sarcoid, amyloid, hemochromatosis, cardiac tumor, con enital! 3>$ /D year old .ale 'ith tricyclic antide(ressant overdose$ Manage$ Resuscitation8 0TL'B0CL' as in question PI a#o$e 9 0?C@, orders, secondary sur$ey, second orders!

HR

+istory -from patient and familyBfriends.8 ask for the empty pill #ottles to confirm the dru -this may #e a pass criterion., how many pills, when taken, concurrent in estions of alcohol or other dru s! %here was the patient foundF %as there a period of unconsciousness, how lon did this lastF ,ther symptoms -$isual #lurrin , seiAure.! @id the patient i$e any warnin of the attempt -note, phone calls, i$in away possessions., was there a precedin depression or stran e #eha$ior, pro#lems at work or with a relationship! &re$ious attemptsF Medications, dru s, alcoholism, smokin , aller ies, past medical and psychiatric history, family history, re$iew of systems! Investigations8 C?C, lytes, urea, creatinine, lucose, I"=B&TT, 0?>, CT, serum osmolality, 0L&, 0'T, 0LT, total #ili, >>T, Toxicolo y 'creen -0'0, acetaminophen, TC0 le$el, #ar#iturates, #enAodiaAepines, Et,;., EC>! C<= -for aspiration pneumonia.! Treat.ent8 >astric la$a e -rare. if less than 1 hour since in estion! 0cti$ated charcoal 14 B dru in ested or 1 Bk #ody wei ht ">! ;ydrate with normal saline to promote diuresis for excretion of TC0 and possi#le myo lo#inuria -occurs due to muscle #reakdown followin prolon ed coma lyin on a hard surface.! 0lkaliniAe with 1 amp #icar# ID -or 13H mEqBk . and hyper$entilation if the patient is intu#ated! Qollow 0?>s or $enous ases, aim for p; 2!R)32!))! =emainin treatment is symptomatic8 treat seiAures with loraAepam H m ID, treat cardiac dysrhythmias, hypotension, a itation and coma as they arise! Consult ICE for HR hours minimum monitorin ! Q=' J 4!1s indicates increased risk of seiAures and dysrhythmias! &sychiatric consult after patient is medically cleared! TC, To7icity) Therapeutic le$els are H3R m Bk ! Life3threatenin symptoms usually occur at le$els J 14 m Bk ! ,nticholinergic e**ects8 hyperthermia, tachycardia, mydriasis -dilated pupils., decreased sweatin and secretions, $asodilation, constipation, urinary retention, ileus! C"' effects are eneraliAed seiAures, myoclonus, ataxia, hyperreflexia, confusion, a itation, hallucinations, acute psychosis, decreased le$el of consciousness, respiratory depression -mnemonic8 ;ot as a hare, #lind as a #at, dry as a #one, red as a #eet, mad as a hatter, the #owel and #ladder lose their tone and the heart oes on alone.! Buinidine e**ects8 conduction delay -prolon ation of Q=', &=, QT, T wa$e flattenin ., heart #lock, #radycardia, asystole, $entricular dysrhythmias and resultant hypotension! 3D$ !4 year old *e.ale in hos(ital *or ,S, overdose$ Medically cleared$ Ta e a history$ +istory8 patient name, a e, occupation! Circumstances surroundin the attempted suicide! &recedin conflicts at work or with family or in a relationship! =ecent loss of employment or lo$ed one! %arnin si ns8 suicide note, i$in away priAed possessions! @escri#e the attempt, how many pills taken, what kinds, concurrent alcohol or dru use! @id the patient really want to die or was the attempt a cry for helpF >au e the lethality of the attempt in terms of the means used and the chances of disco$ery! &re$ious attempts, descri#e these! Is patient now acti$ely suicidal or remorseful! If the patient acti$ely suicidal, what is the current planF Medications, dru Balcohol use, aller ies, past medical history, family history -esp! psychiatric., social supports8 re$iew of systems! Psychiatry) ;istory for depressionBmania, mental status exam, multiaxial dia nosis as in question PR a#o$e! 34$ %ather 'ith 3L year old child 'ho is not s(ea ing$ Ta e a history$ %indings) not s(ea ing 'ell& recurrent ear in*ections& (oor hearing$ Ma e a diagnosis$ +istory8 of not speakin should determine whether the pro#lem is primary -ne$er spoke. or secondary -stopped speakin .! 'econdary causes of mutism are psycholo ical upset -due to family discord, etc!. and rare inherited neurode enerati$e conditions! &rimary mutism may #e part of a lo#al de$elopmental delay or related to hearin pro#lems which are either con enital -inherited, intrauterine infections., ototoxic dru s -e! ! streptomycin. or trauma! Pregnancy and birth history8 >T&0L -num#er of estations, term pre nancies, premature #irths, a#ortions, li$e children., history of pre$ious pre nancies includin neonatal :aundice, maternal medical history, illness durin pre nancy, ru#ella -terato enic., toxoplasmosis -from cats, infects fetal #rain., herpes -infects fetus, frequently fatal., CMD -dama es fetal li$er., terato enic medications taken durin pre nancy, dru and alcohol use, family history of deafness or late speakin ! "ew#orn history8 estational a e at #irth, caesarean, induction, rupture of mem#ranes artificial or prolon ed, fetal distress, forceps or $acuum deli$ery, meconium, 0&>0=s, was resuscitation requiredF Initial #lood work, #reast feedin F ;ow often and how well, color of 1st stool, color of urine, $omitin , neonate muscle tone, #eha$iors, fe$er, irrita#ility, lethar y! -evelo(.ental history :*ro. (arent;8 'ee question P(! >rowth8 expected hei ht and wei ht for a eF 'peech, has child e$er spoken words or phrases, are these used appropriately, has the child made sounds, chronolo y and description of these! ;ow

H)

does the child communicate if not throu h speechF >ross Motor8 when did the child start walkin , runnin ! Qine motor8 when did you notice pincer rasp, turnin pa es in a #ook! 'ocial #eha$ior! +earing8 does the child wake up in response to soundsF 'tartle to loud soundsF Come when calledF Enderstand spoken instructionsF ;istory of ear infections, wax pro#lems! 0sk a#out swimmin ! &ast medical history, medications, aller ies, family history, re$iew of systems! -iagnosis8 i$en recurrent otitis media with poor hearin the most likely dia nosis is retarded speech de$elopment due to poor hearin ! =efer to E"T for hearin tests, possi#le tu#es -tympanic draina e.! 3#$ >0 year old *e.ale 'ith *atigue$ Ta e a history$ %indings) cold intolerance& 'eight gain$ Ma e a diagnosis$ %atigue history8 onset, chronolo y, past episodes, functional limitations, associated with exertionF =ecent $iral illness -mononucleosis., cold intolerance, wei ht ain, dry skin, #rittle hair, hoarseness -hypothyroidism., associated muscle aches -fi#romyal ia., chest pain -an ina., shortness of #reath -con esti$e heart failure.! Slee( history8 usual requirements, chronolo y of sleep pro#lems, stressor, sleep hy iene -when, where, re ularity, shifts at work, quiet, late, exercise, meals, alcohol, caffeine, prescription and non3prescription remedies, dru s and medications., sleep latency -time to fall asleep., nocturnal awakenin , early mornin wakenin , daytime somnolence, somnolence while dri$in , workin or durin con$ersation! -e(ression screener8 as in question PR a#o$e! Must fully explore suicidal ideation8 does patient intend to harm self, reason for suicidal thou hts, current plan, lethality of plan, access to lethal means, has patient i$en away priAe possessions or written final notes to lo$ed ones, pre$ious attempts! Medications -especially TC0s, sedati$es, antihypertensi$es., aller ies, dru Balcohol use, smokin , past medical history, family history, re$iew of systems! -i**erential -iagnosis8 Chronic fati ue, C;Q, ischemia, thyroid disease, sleep distur#ance, depression! -iagnosis8 most likely hypothyroidism i$en cold intolerance and wei ht ain! 3?$ 3D year old .ale 'ith bac (ain and sti**ness$ Ta e history and (er*or. a *ocused (hysical e7a.$ %indings) !0c. se(aration bet'een lu.bar s(ines 'hile erect increases by less than D c. 'hen bac is *le7ed *or'ard :(ositive 5right8 Schober test;& lateral *le7ion i.(aired$ B) Give the diagnosis and t'o associated conditions$ +istory and Physical8 see question P1)! -iagnosis8 #ased on typical history of #ack pain, lum#ar spine <3rays showin fusion of the sacroiliac :oints or sacroiliitis and syndesmophytes -disk spaces under oin fusion., ele$ated E'= and ;L03?H2 tissue anti en positi$e this is pro#a#ly ankylosin spondylitis! ,ssociated conditions8 inflammatory arthritis, u$eitis, psoriasis, I?@, amyloidosis, radiculopathy, pericarditis, an ina, conduction a#normalities! Treat.ent8 "o cure! =e ular therapeutic exercise to pre$ent deformityBdisa#ility -esp! swimmin and #ack extension exercises.! To control pain and stiffness8 indomethacin -144 m &, ,@., naproxen -H)4 m &, ?I@3TI@., etc! 'ur ery helpful in se$ere cases8 e! ! total :oint replacement! 3"$ 3D year old .ale 'ith ! year history o* bac and leg (ain$ -o a (hysical e7a.$ %indings) right *oot dorsi*le7ion 'ea ness 'ith dorsal *oot nu.bness$ Ma e a diagnosis and identi*y the level o* lesion$ +istory and Physical e7a.8 see question P1)! -iagnosis8 LR,) disk herniation with L) radiculopathy! >0$ ># year old .ale 'ith i.(otence$ 5ants a @(illA *or this$ %ears losing his girl*riend$ Ta e a history and counsel$ 1ac ground8 The causes of erectile dysfunction are su#di$ided in to psychic and or anic cate ories! Current literature fa$ors an approximately 181 ratio of or anic to psychic etiolo y! The pro#lem is rarely primary -ne$er had a#ility to sustain erection.! The or anic causes are8 dru s -X3#lockers, thiaAides, ;H #lockers, antidepressants, antipsychotics, @i oxin, clofi#rate, sedati$es, alcohol, heroin., hormonal -dia#etes, onadal dysfunction, prolactin3secretin pituitary tumor, associated with loss of li#ido and testicular atrophy., neuro enic -stoke, M', spinal cord in:ury., iatro enic -dru s, sur ery, radiation., $ascular

HI

-peripheral $ascular disease, @M, ;T".! Impotence is rossly defined as ina#ility to ha$e satisfactory intercourse due to erectile dysfunction in at least H)* of encounters! +istory8 Current partners! &ro#lems in these relationships! %hy is the patient seekin medical attention for this nowF ,nset of erectile dysfunction and chronolo y! @escription of the pro#lem8 no erection at all, cannot sustain erection, e:aculate too quickly to satisfy partner, cannot achie$e or asm or or asm without e:aculation, retro rade e:aculation! Circumstances under which impotence occurs8 only with certain partners, only at certain times or locations, what percenta e of the timeF Is impotence related to lack of sexual desireF &resence and firmness of mornin or nocturnal erections! @oes the patient sustain erections in mastur#ationF 0ssociated pro#lems8 anxiety attacks, anhedoniaBdepression, perineal or peripheral num#ness, poor peripheral circulation! Exercise, medications, contracepti$e use, dru Balcohol use, smokin , cholesterol, aller ies, past medical history, family history, re$iew of systems! Counseling8 @iscuss causes of impotence in terms of or anic $s! inor anic etiolo y and that it tends to cause reat anxiety -normaliAe patient+s feelin s.! Erectile dysfunction can often #e impro$ed with lifestyle chan es8 exercise, wei ht loss, impro$ed diet, decreased alcohol intake, smokin cessation, stress mana ement, sleep hy iene, #etter dia#etic control, and :oint counselin with partner to decrease anxiety! Impro$ement of patient+s relationship with partner8 address sexual #oredom! =e$iew medications8 su est chan es! Explain that many or anic causes of impotence are unfortunately not re$ersi#le! @escri#e therapeutic options8 counselin with partner on alternati$e means of sexual ratification, testosterone preparations or #romocryptine -for prolactinoma. if patient is shown to ha$e hormonal distur#ance on #lood work -measure testosterone and onadotropins., sildenafil -$ia ra., yohim#ine and traAodone preparations for impotence -poorly effecti$e and expensi$e., penile self3in:ection with phentolamine, papa$arine 7 &>E1 or &>E1 alone -34 au e needle, last 343I4 min!, quite popular., $acuum3ru##er rin de$ice, penile prostheses! 0rran e follow up with #oth partners! >!$ Houng 'o.an 'ith tunnel vision$ Negative investigations by a neurologist and o(hthal.ologist$ Ta e a history$ %indings) concerned that her husband is having an e7tra.arital a**air$ Counsel$ +istory8 @escription of $isual pro#lem, functional limitations, onset duration, chronolo y, a ra$atin Brelie$in factors, associated headache, eye pain, nausea, anxiety, palpitations, tremor! &re$ious episodes of eye pro#lems or other unusual phenomena -#lindness, paralysis, num#ness, a#dominal pain.! %hat doctors has patient seen, what did they sayF &ro#lems at work, home, with relationships! %ho can the patient o to for support in her lifeF &sychiatric pro#lems in the pastF @epression screener and sleep history as in questions P3 and P1(! Medications, dru and alcohol use, aller ies, past medical history, family history, re$iew of systems! Conversion disorder8 psychic pertur#ation presents as one or two neurolo ical complaints affectin $oluntary motor or sensory function! &sycholo ical factors thou ht to #e etiolo ically related to the symptom as the initiation of symptoms is preceded #y conflicts or other stressors! 5La belle indifference6 9 patient+s inappropriately ca$alier attitude towards a serious symptom! Treatment8 anxiolytics -e! ! loraAepam 1 m &, qIh., relaxation therapy, counselin , close follow3up! Counseling8 "ormaliAe this pro#lem and $alidate the patient+s feelin s8 the fact that se$eral specialists ha$e said there is nothin wron with the patient+s $ision does not mean that there is not a su#tle medical pro#lem which may #ecome apparent later! Qor this reason it is important to stick with one doctor who knows the patient well and can coordinate further referrals if necessary! Many people who are faced with the possi#ility of marital infidelity automatically acti$ate a defense mechanism which i$es them time to ad:ust, and which is not under conscious control8 'uch a reaction also helps the patient to enlist needed support from others! This is a normal reaction for these people! These symptoms can $ary widely from paralysis, to num#ness, to pains, to ina#ility to speak, and $isual pro#lems includin #lindness and tunnel $ision! ,utline a plan for mana ement8 address possi#le sources of anxiety and stress8 a frank discussion with the patient+s hus#and a#out fidelity is required and may #e done pri$ately or if #oth partners are willin , in consultation with you or a marital therapist! The patient should try to consolidate a support network8 parents, friends, etc! Consider depression, sleep or anxiety medications as appropriate! 0rran e follow up with #oth partners! >/$ 30 year old .ale *ro. another city$ 5ants %iorinal (rescri(tion *or chronic headaches rene'ed$ Manage$ 'ee question P1I! >3$ /3 year old *e.ale 'ith /> hour abdo.inal (ain$ Per*or. *ocused (hysical e7a.$ %indings) (eritoneal signs& (oint tenderness at Mc1urneys (oint$ B) Give a di**erential diagnosis& order investigations$ 5hat *urther history 'ould hel( con*ir. diagnosisC Physical e7a. *or abdo.inal (ain8 see question PH(!

H2

Rectal8 rectal shelf, check for ross or occult #lood! 0lso include a pel$ic manual and speculum exam! -won+t #e asked to perform this at the LMCC II.! Check for pain with cer$ical motion -seen in &I@., pain on palpation of o$aries, mass, cer$ical dischar e! Take swa#s -see in$esti ations.! -i**erential diagnosis8 appendicitis, o$arian cyst, rupture or o$arian torsion, mittelschmerA, ectopic pre nancy -life3 threatenin ., hepatitis, cholecystitis, astroenteritis, peptic ulcer, pel$ic inflammatory disease -&I@., urinary tract infection -ETI., pyelonephritis, kidney stone, inflammatory #owel disease, intestinal o#struction due to $ol$ulus or I?@! Investigations8 0?< 3 $iews, a#dominal3pel$ic ultrasound, C?C, lytes, urea, creatinine, I"=B&TT, lucose, XhC>! Erinalysis! 'tool for occult #lood! Cer$ical swa#s for culture and pap smear! If ,= is imminent order type and cross for H units, C<=! +el(*ul *urther history8 a history of radual onset $a ue perium#ilical or LLQ pain mi ratin to a sharper, more localiAed pain in the =LQ o$er se$eral hours associated with anorexia, nausea, and controlled #y still fetal posture su ests appendicitis! >>$ 4/ year old *e.ale 'ith le*t lo'er Euadrant (ain$ Per*or. (hysical e7a.$ %indings) lo' grade *ever& so.e abdo.inal distention& LLB tenderness 'ithout rigidity& (oorly de*ined le*t lo'er Euadrant .ass$ B) ,bdo.inal series sho's .ulti(le air/*luid levels$ -escribe$ Give di**erential diagnosis 'ith .ost li ely diagnosis$ Order *urther investigations$ Physical e7a. *or abdo.inal (ain8 see question PH( and PR3! -i**erential diagnosis8 di$erticulitis, di$erticular a#scess, constipation with o#struction, >I mali nancy with perforation, allstone ileus, o#struction due to $ol$ulus -usually =LQ pain., Crohn+s, mesenteric ischemia or infarct, o$arian tumor, &I@, uterine perforation! Most li ely diagnosis8 di$erticulitis! Investigations8 a#dominal3pel$ic CT -ultrasound if CT una$aila#le., stool for occult #lood, urinalysis, cer$ical swa#s and pap, C?C, lytes, urea, creatinine, I"=B&TT, lucose, for possi#le preop8 EC>, C<=, type and cross R units! >D$ /> year old *e.ale 'ith le*t lo'er Euadrant abdo.inal (ain 'ho has an IM-$ Per*or. (hysical e7a.$ %indings) signs o* (eritoneal irritation$ B) Give a di**erential diagnosis& order investigations$ Physical e7a. *or abdo.inal (ain8 see question PH( and PR3! -i**erential diagnosis8 uterine perforation #y IE@, pel$ic inflammatory disease -&I@ more common with IE@., o$arian cyst with torsion or rupture, tu#o3o$arian a#scess, ectopic pre nancy -also more common with IE@., astroenteritis, inflammatory #owel disease, intestinal o#struction due to $ol$ulus or I?@, appendicitis! Investigations8 0?< 3 $iews, a#dominal3pel$ic ultrasound, C?C, lytes, urea, creatinine, I"=B&TT, lucose, XhC>! Erinalysis! 'tool for occult #lood! Cer$ical swa#s for culture and pap smear! If ,= is imminent order type and cross for H units! C<=! >4$ /0 year old *e.ale at 34 'ee s gestation 'ith hy(ertension and (roteinuria$ Counsel (atient on etiology and ris s to *etus and .other$ B) Outline a (lan *or .anage.ent$ 1ac ground8 ;ypertension may #e pre nancy induced or pre3existin hypertension can #e worsened #y pre nancy! &re3 eclampsia is pre nancy3induced or worsened hypertension -systolic ?& increased #y 34 mm; , and diastolic #y 1)mm; o$er non3pre nantBT1 pressures. with renal impairment -proteinuria J C1 on dipstick or J 344 m BHR hour collection. andBor non3 dependent edema -e! ! face, hands. onset J H4 weeks! Eclampsia is pre3eclampsia with C"' in$ol$ement, usually decreased le$el of consciousness and seiAures! ,ther end or ans may #e affected, particularly the li$er and placenta! May pro ress to death throu h multi3or an failure! Im#alance of throm#oxane -$asoconstrictor. and prosta landin -$asodilator. causes arteriolar constriction capillary dama e protein extra$asation and hemorrha e! ?oth pre3eclampsia and eclampsia fall under the new headin of pre nancy3induced hypertension -&I;.! Mild PI+8 no C"' in$ol$ement or criteria for se$er &I;! Severe PI+8 H of the followin 8

HK

1! H! 3! R! )! I! 2! K! (!

?& J 1I4B114 pulmonary edema or cyanosis C;Q proteinuriaBoli uriaB Cr LQTsB=EQ or epi astric pain $isual distur#ancesBhyperreflexiaBclonusBheadacheBcon$ulsions "7D throm#ocytopeniaBmicroan iopathic hemolysis IE>=

2cla.(sia8 rand mal seiAures in woman with preeclampsia! Counseling8 /ou ha$e pre3eclampsia! @efine as a#o$e! Condition is common8 )* of pre nant population, more common in first pre nancies! Cause seems to #e im#alance of throm#oxane -$asoconstrictor. and prosta landin -$asodilator. which causes arteriolar constriction capillary dama e protein extra$asation and hemorrha e! 'e$erity $aries! =isks for mother8 end or an dysfunction -kidneys, li$er., cere#ral hemorrha e -)4* of deaths., left $entricular failureBpulmonary edema, loss of pre nancy due to a#ruption -often with @IC., seiAures, ;ELL& syndrome, death! =isks for fetus8 malnutrition, hypoxia, incomplete maturation, which can result in low #irth wei ht -IE>=., prematurity, death! ,$erall treatment strate y is to slow pro ression of hypertension until the #a#y can #e deli$ered! @eli$ery is curati$e! Manage.ent (lan8 Initial e7a. and investigations8 $itals, #ody wei ht, examine for edema, check for =EQ tenderness, reflexes, C?C, lytes, creatinine, urea, uric acid, urinalysis with microscopy, HR hour urine proteinBcreatinine clearance, LQTs, I"=B&TT! Qetal ;=, non3stress test, #iophysical profile -ultrasound with ) criteria., @oppler flows! ?ed rest in left lateral decu#itus position -to reduce a#dominal $essel compression., normal dietary saltBprotein intake! "o use of diureticsBantihypertensi$es! %ollo' u(8 daily ?&, daily wei ht, daily reflexes, fetal mo$ement counts at home -if patient li$es reasona#ly close to a hospital and can et transportation fast., frequent follow3up $isits for #lood work, urinalysis and fetal monitorin , #ed rest -prefera#ly left side.! Instruct patient on worsenin si ns8 rapid wei ht ain, li$er pain, $isual distur#ance, persistent headache, drowsiness or seiAures! -elivery8 early hospital admission -at 3I weeks. for close monitorin and administration of ID M ', R R o$er H4 minutes, if si ns of C"' in$ol$ement are present -hyperreflexia, decreased L,C, seiAure.! &ossi#le induction of early deli$ery or Cesarean section! Consider ID hydralaAine -first line treatment. )314 m ID o$er ) minutes q1)334min, second line8 la#etolol H43)4 m ID q14min, third line8 nifedipine 143H4 m &, qH43I4min if deli$ery is not imminent or if deli$ery is imminent to decrease ?& after conser$ati$e measures tried -diuretics and salt or fluid restriction not useful and may #e harmful.! >#$ 30 year old 'o.an 'ith vaginal bleeding at 30 'ee s gestation$ Ta e a history$ B) Give a di**erential diagnosis$ Order investigations$ +istory8 patient I@ -name, a e, occupation., >T&0L, weeks of estation! ,nset of #leedin , duration, estimate quantity -num#er of pads soaked., color and consistency of #lood, associated fe$er, chills, a#dominal discomfort -pain., contractions, fetal mo$ement, li htheadedness, last sexual intercourse -may cause spottin due to fria#le cer$ix.! &ro#lems with pre$ious pre nancies, pro#lems in this pre nancy, medical $isits to this point, in$esti ations done! 0ssociated a#dominal trauma -accident or a#use., dru use -cocaine., father and mother+s #lood type, medications, alcohol, smokin , past medical history, family history, re$iew of systems! -i**erential diagnosis8 placenta pre$ia -placenta co$ers internal os of cer$ix 9 the most common cause of painless #leedin in the third trimester., #loody show -sheddin of cer$ical mucous plu ., a#ruptio placenta -separation of placenta from uterine wall 9 usually painful., $asa pre$ia -fetal #leed due to root $essels of um#ilical cord o$erlyin the cer$ical os 9 extremely dan erous to the fetus.! ,ther causes8 uterine rupture, coa ulopathy -@IC., molar estation, $a inal tear, $a inal infection, cer$ical polyp, cer$icitis, cer$ical cancer, placenta accreta, #leedin from #ladder or #owel! -",TE8 no $a inal exam until pre$ia has #een ruled out #y EB'.! Investigations8 maternal $itals, C?C, I"=B&TT, fi#rino en, type and cross3match if #leedin is se$ere, =h status -may need =ho am amma lo#ulin to pre$ent formation of anti#odies a ainst fetal #lood if mother is =h ne ati$e and father is =h positi$e., fetal monitor, fetal ultrasound, maternal monitorin , ID access, pel$ic speculum and manual exam with di ital cer$ical exam -do these only after ultrasound to rule out placental pre$ia 9 can cause further #leedin in pre$ia., 0pt test for

H(

fetal hemo lo#in in $a inal #lood, test maternal #lood for presence and amount of fetal hemo lo#in -determines amount of =ho am required to neutraliAe fetal #lood anti enicity.! >?$ 3! year old *e.ale 'ith right hand nu.bness and 'ea ness$ Ta e a history and (er*or. *ocused (hysical e7a.$ B) -i**erential diagnosis& investigations and treat.ent$ +istory8 name, a e occupation, amount of work done with hands, description of symptoms, onset, duration, chronolo y, time of day, a ra$atin and relie$in factors! &re$ious episodes, in$esti ations! 0sk a#out pain at ni ht, difficulty turnin a key or openin :ars -si ns of weakness.! Qunctional limitations! 0ssociated in:ury, neck pain, num#ness or weakness in other areas, $isual pro#lems, headache, nausea! Medications, dru Balcohol use, smokin , aller ies, past medical history -especially @M, hypothyroidism, rheumatoid arthritis, pre nancy., sur ical history, family history, re$iew of systems! Physical e7a.8 see question P34! -i**erential diagnosis8 carpal tunnel compression of median ner$e, cer$ical radiculopathy, strokeBTI0, dia#etic peripheral neuropathy, #rachial plexus in:ury or tumor! Investigations8 ner$e conduction studies! Treat.ent8 modify manual work, wrist splint -often worn at ni ht., "'0I@s, local corticosteroid in:ections, control underlyin systemic contri#utors -e! ! dia#etes, hypothyroidism, arthritis., sur ical decompression $ia flexor retinaculum release -"eurosur ery or &lastic sur ery consult.! >"$ D3 year old *e.ale 'ith incidental solitary lung nodule on CFR$ Ta e a history$ %indings) non8s.o er& te7tile 'or er& no sy.(to.s$ B) CFR sho's ho.ogeneous round / c. o(aci*ication in right u((er lobe$ Give a di**erential diagnosis and identi*y the .ost li ely diagnosis$ Order *urther investigations$ +istory8 name, a e, occupation, li$in conditions, ho##ies, pets, esp! #irds, cats, tra$el history, contact with haAardous su#stances -e! ! as#estos.! &ositi$e T? skin test! ;istory of pneumonia, T? etc! Mali nant symptoms8 wei ht loss, fati ue, chan e of #owel ha#its, anorexia, ni ht pain 'mokin ! 0lcoholism! Lun symptoms8 cou h, sputum, shortness of #reathBdyspnea, hemoptysis, wheeAe, orthopnea, &"@, chest wall pain, Medications8 dru sBalcohol, aller ies, past medical history, family history, re$iew of systems! -i**erential diagnosis8 less than 3 cm is more likely to #e #eni n #ut reater than 3 cm is more likely mali nant8 neoplasm -R)*. primary O 24*, #eni n -hamartoma, lipoma. O 1)*, metastatic O 14*L infection -)3*. T?, histoplasmosis, coccidiomycosisL other -H*. ranuloma -scar tissue from old pneumonia, T? ranuloma, histoplasmosis, silicosis, sarcoid., $ascular -03D malformation, infarct., con enital -cyst., round pneumonia, round atelectasis, loculated effusion! &ercenta es are for lesion reater than 3 cm! Most li ely diagnosis8 ranuloma! Investigations8 old C<= for comparison -if lesion is old and unchan in , inter$entions are less a ressi$e, calcification is also associated with #eni n lesions such as old ranulomas., CT chest with CT uided needle #iopsy, sputum for cytolo y and acid3 fast stainin -T?., T? skin test, #ronchoscopy with #iopsy and washin s if lesion seen, open #iopsy or lo#ectomy! ,lgorith.8 solitary nodule pre$ious C<= #eni n or unchan ed -repeat in q33Imonths for H years if unchan ed o#ser$e, if chan ed at any time continueG., mali nant or chan ed CT thorax8 cancer -sta e and treat., calcification -o#ser$e., no dia nosis #ronchoscopy or transthoracic needle aspiration still no dia nosis -resect for dia nosis., inflammatory -treat cause., cancer -sta e and treat.! D0$ Mother 'ith 4 .onth old child 'ho has diarrhea$ Ta e a history$ %inding) recently s'itched *ro. breast .il to .il (o'der *or.ula$ B) Give a di**erential diagnosis$ Reco..end treat.ent$ +istory8 @istin uish #etween acute diarrhea, chronic diarrhea with or without failure to thri$e! "ame, a e, a e of onset and chronolo y of diarrhea, consistency, color, quantity and frequency of diarrhea, #lood in stool, concurrent illness, $omitin , fe$er, anorexia, difficulty #reathin , lassitude, dry mouth, eyes, low urine output, illness affectin other children in the family or adults! Qeeds and feedin history -esp! fruit :uice.! >rowth pattern8 wei ht lossF =ecent immuniAation, tra$el, anti#iotics! Medications, past medical history, aller ies, #irth history, pre nancy pro#lems, maternal illness durin pre nancy, family history, re$iew of systems!

34

-i**erential diagnosis8 infection -#acteria8 campylo#acter, salmonellaL anti#iotic induced8 c! diff colitisL parasitic8 iardiaL post infectious8 secondary lactase deficiency., intolerant of formula chan e -component of lactose intolerance., toddler+s diarrhea -I months to 3I months, resol$es spontaneously #y a e H3R years lots of :uice o$erwhelms small #owel disaccharide mala#sorption., o$erfeedin , osmotic diarrhea due to hi h osmolality liquids such as :uice, astroenteritis! %ith wei ht loss consider mala#sorption syndrome8 celiac disease, lactose intolerance, milk protein aller y, cystic fi#rosis -with chest infections., I?@ -unusual at this a e., enAyme deficiencies, li$er disease, #iliary atresia, thyrotoxicosis! Treat.ent8 try non3milk -soy #ased. formula! 0rran e follow up! Note8 Treatment for Toddler+s diarrhea reassurance, self3limitin , R f+s fi#er, normal fluid intake, 3)3R4* fat, discoura e excess fruit :uice! D!$ D0 year old .ale alcoholic$ <o.ited bright red blood ! 'ee ago$ Per*or. (hysical e7a.$ B) Give di**erential diagnosis$ 5hat is your treat.ent (lanC Physical e7a.8 see question PH(! -i**erential diagnosis8 esopha itis, astritis, duodenal ulcer, peptic ulcer, Mallory3%eiss -partial thickness. tear, esopha eal $arices, astric or esopha eal cancer, lun tumor, aorto3enteric fistula -rare 9 can occur after pre$ious aortic sur ery.! Treat.ent (lan8 place on omepraAole H4 m &, ,@, patient to refrain from alcohol, discontinue "'0I@s, draw #lood for I"=B&TT, 0'T, 0LT, 0L&, >>T, total #ili, C?C, lytes, urea, creatinine! C<= -check for perforation air under diaphra m., refer to astroenterolo y for endoscopy! Consider admission if patient unrelia#le or transportation is a pro#lem! D/$ Tele(hone rings) hysterical .other says her child s'allo'ed a cleaning agent$ Manage over the (hone$ B) 5hat do you do a*ter hanging u( the (honeC 5hat investigations do you order in the 2RC Over the (hone8 Esta#lish calm, reassurance, o#tain caller name, phone num#er, address! Child+s a e, wei ht, medical pro#lems, medications, aller ies! Identify a ent in ested, ha$e caller read haAard la#el if possi#le! ;ow much was taken and whenF %hat is child+s present conditionF 0ny other a ents takenF ;a$e child drink H33 lasses of milk if alert -works for alkali or acid.! @o not induce $omitin -exposes esopha us and pharynx to the corrosi$e a ent a ain.! If amount in ested was $ery small, may not need to come in, warn that child may $omit! If amount unknown or si nificant, child may come to E= immediately! Mom may dri$e if calm, otherwise you will send an am#ulance! ;a$e child lie on his side in case of $omitin ! ,*ter hanging u(8 send am#ulance, call the poison center with the description of the a ent and ask for direction as to treatment etc! Investigations8 C?C, lytes, lucose, urea, creatinine, serum osmolality, serum ketones, 0?>, C<=, toxicolo y screen if in ested a ent unknown!

!"">
D3$ 40 year old .ale 'ith /0 .inutes chest (ain$ -ia(horetic& 2CG sho's / .. ST elevation in <!& /& 3$ Manage$ +istory and Physical8 see question P13, P1R and P12! Treat.ent8 =aise head of #ed! >i$e oxy en I LBmin #y mask! Monitor oxy en saturation! ,rder stat C?C, lytes, lucose, I"=B&TT, serial CT3M? and Troponin, 0?>, C<=, EC>! >i$e chewa#le 0'0 1I433H) m immediately! 'ecure ID access, #olus ID lasix R4 m , push if fluid o$erload is suspected, and $entolin if wheeAes are heard, i$e su#lin ual nitro spray or 4!3 m 'L nitro if #lood pressure is adequate and 1 m morphine ID! =epeat nitro 'L q)min x 3! May require additional morphine and nitro! =epeat CT3M? and Troponin qKh x 3! -iagnosis8 se$ere anterior wall ischemia e$ol$in to infarct! =equired throm#olytics or cardiac catheteriAation! Er ent Cardiolo y consult if a$aila#le! D>$ /? year old .ale arrives in the 2- having *allen 4. *ro. a sca**old$ ,ssess$ Rescusitation8 see question PI!

31

+istory8 if a$aila#le8 "ature of fall, precedin -seiAure, etc!. and su#sequent e$ents, has patient e$er experienced similar symptoms #efore, did patient lose consciousness, were there seiAure3type phenomena, in:uries durin fall, duration of unconsciousness, post3ictal drowsiness, medications and dru s, alcoholBillicit dru in$ol$ement, smokin , aller ies, past medical history, family history, re$iew of systems! DD$ // .onth old *e.ale child brought to e.ergency by her .other 'ith *ractured le*t hu.erus$ T'o (revious *ractures in (ast 3 .onths& bruises seen on *orehead$ Manage$ 5arning signs o* child abuse8 explanation doesn+t match in:ury, delay in seekin treatment, recent family crisis, in:uries of $aried a esBrecurrentBmultiple in:uries, distincti$e marks, atypical pattern of in:ury, unrealistic expectations of child #eha$ior #y care i$ers! Ris *actors *or child abuse8 en$ironmental -social isolation, po$erty, domestic $iolence., care i$er -su#stance a#use, :ealousy #etween #oyfriend and father, parent+s a#used themsel$es, personalityBcharacter disorder or mental illness, poor social and $ocational skillsB#elow a$era e intelli ence., child factors -disa#ility, difficult child i!e! temperament, premature.! +istory8 how did the in:ury happenF %ho was lookin after the child when it happenedF %ho are the child+s care i$ers, and who li$es in the house or comes in contact with the child! ;ow did the child et the #ruisesF %hat happened with the other fracturesF 0ny other in:uries in the pastF Is the child accident prone or difficult to handleF %hat is the child+s personality8 open $s! withdrawn! 0re there other children in the houseF ;a$e they had #roken #ones or other in:uriesF %as this child a planned pre nancy, pro#lems with pre nancy, #irth history! @e$elopmental milestones as in question P(! %here does the child li$e -isolation., income le$el of parents, pro#lems with the law, alcoholism, dru use, smokin #y care i$ers or other adults in the home, what is the typical response of care i$ers when the child cries or mis#eha$esF %ere the care i$ers a#used as childrenF Is there spousal a#use, sexual a#use or incestF ;as the Children+s 0id 'ociety #een in$ol$ed with this child or other childrenF Inter$iew relati$es, friends! Child+s medical history, medications, aller ies, re$iew of systems! Physical e7a.8 o#ser$e child+s #eha$ior, constant watchfulness associated with child a#use! Inspect for malnutrition, #ruises, scars, #urns, especially on the flexor surfaces! Inspect oral ca$ity, perineum, anus, enitalia! ,phthalmoscopy for retinal hemorrha es -shaken #a#y syndrome.! E$aluate for de$elopment, neurolo ical exam for possi#le #rain in:ury! Investigations8 <3rays for old fractures, if records not a$aila#le, C?C, lytes, urea, creatinine, I"=B&TT, al#umin -malnutrition.! Treat.ent8 admit for the child+s safety and in$esti ations, consult child psychiatrist and pediatric orthopedic sur eon! ,#li ated to report suspicion of child a#use to Children+s 0id 'ociety! Qamily therapy, frequent follow up to monitor de$elopment! D4$ 3D year old .ale$ ! 'ee hy(eractivity& histrionic& s(ending s(ree& bi6arre behavior$ Ta e a history$ -i**erential) Causes of one week of #iAarre #eha$ior8 manic episode -#ipolar mood disorder., depression, dru 3induced -steroids, amphetamines, alcohol., or anic -hypothyroidism, frontal lo#e tumor, M', dementia., schiAophrenia! +istory8 @escription of symptoms8 onset, duration, chronolo y, a ra$atin and relie$in factors -dru use.! Is patient a dan er to himself or others -suicidal or homicidal.F @oes patient ha$e alternatin up and down periods, how lon do these lastF ;ow frequentF 0re they cyclin faster than #eforeF 0sk a#out mood, sleep, interest, uilt, ener y, concentration, appetiteBwei ht, psychomotor, suicideBmor#id ideation! &aranoia, ideas of reference -thou ht #roadcastin L special messa es, mind readin ., special powers, ma ical thinkin , secret identity, $oices, $isual or tactile hallucinations! Current life e$ents, stress, relationship pro#lem, #erea$ement! &re$ious psychiatric pro#lems, family history or psychiatric disorders, su#stance a#use, relationship pro#lems, pro#lems at work! %ork and relationship histories! 0sk a#out hypothyroidism, adrenal dysfunction, hypercalcemia, mononucleosis! Medications, dru s, alcohol, smokin , aller ies, past medical history includin psychiatric history and history of a#use! Qamily history, re$iew of systems! Manic e(isode8 Expansi$e, ele$ated or irrita#le mood x 1 week with 3 of followin 8 >'T&0I@ 9 randiosity -or inflated self esteem., sleep -less need for., talkati$e, pleasura#le acti$ities -with painful consequences., acti$ity increased - oal directed or psychomotor., ideas -fli ht of., distracti#ility! "ot mixed episode! 'e$ere enou h to cause psychotic featuresBimpaired socialBoccupational functionin ! "ot su#stance a#use or >MC! -iagnosis o* .a9or de(ression8 see question PR!

3H

Mental status8 see question PR! D#$ // year old .ale$ +ears voices$ Ta e a history$ ;istory as in question P)I and PR with special attention to the chronicity of symptoms, and work, school, and relationship histories! -SM I< Criteria *or the diagnosis o* schi6o(hrenia8 08 H or more of the followin characteristic symptoms occurrin for a si nificant portion of a 1 month period8 delusions, hallucinations, disor aniAed speech, catatonic or rossly disor aniAed #eha$ior, ne ati$e symptoms -flat affect, alo ia 9 po$erty of speech, a$olition, anhedonia, apathy, affectional impairment.! ,nly 1 symptom if delusions are #iAarre or hallucinations consist of a $oice keepin runnin commentary or H $oices con$ersin ! ?8 'ocial or occupational dysfunction! C8 Continuous si n of some distur#ance for at least I months includin the month of more se$ere distur#ance in 0! @8 'chiAoaffecti$e or mood disorder excluded! E8 'u#stance a#use or eneral medical disorder excluded! Q8 If there is a history of autistic disorder or per$asi$e de$elopmental disorder, then the dia nosis of schiAophrenia is made only if delusions or hallucinations are prominent for at least 1 month! "ote that schiAophrenic symptoms M 1 month O #rief psychotic disorder, M I months O schiAophreniform disorder, J I months O schiAophrenia! D?$ !4 year old .ale 'ith three e(isodes o* sudden loss o* a'areness lasting N ! .inute& 'ants in*or.ation on e(ile(sy$ Counsel$ +istory8 see question P3! Counseling8 Cause of seiAures distur#ed electrical acti$ity in the #rain, often with a tiny focus of a#normal tissue from pre$ious infection, trauma -includin #irth trauma. or inherited! 0#out H* of the population ha$e epilepsy! /our seiAures appear to #e a#sence or petit3mal, which start in youn people! ,ne third of cases resol$e spontaneously with a e! 'eiAures do not dama e the #rain unless they are prolon ed -J 34 minutes. and a#sence seiAures are not associated with decreased intelli ence or learnin a#ility! Most people with this type of epilepsy are well controlled on medication and ha$e no limitations in their acti$ities, careers or relationships! %urther counseling and treat.ent8 see question P3! D"$ Houng .an in the e.ergency de(art.ent 'ith a stab 'ound to the belly$ Manage$ 'ee question PI! 40$ !? year old *e.ale 'ants oral contrace(tive$ Manage$ 'ee question PH! 4!$ 3D year old 'o.an *eels de(ressed$ Manage$ 'ee question PR! 4/$ D year old girl 'ith (ain on urination$ Mother concerned about MTI and se7ual abuse$ Order investigations and counsel$ ,lgorith.8 Possible MTI Is i..ediate anti.icrobial thera(y indicatedC -e! ! infant, toxic, dehydrated. i* yes -urine specimen for culture $ia '&0Bcatheter initiate 0#x therapy UTM&B'M< or cephalosporins i!e! cefixime, cefproAilV8 consider hospital if toxic, $omitin and cannot take &, meds or if M I months.L i* no -EB0 on specimen #a EB0 positi$e for LE, nitrites, %?CF noF unlikely ETI if no symptoms yesF ,#tain urine for culture '&0Bcatheter. is the culture (ositiveC noC -no ETI. yesC -231R days of 0#xL prophylaxis UTM&B'M< or nitrofurantoinV until ima in completed.

33

clinical res(onse in >? hrsC noC -immediate urinary tract EB'. yesC -EB' as soon as con$enient. ulti.ately <CMG as soon as convenient! Investigations8 midstream clean catch $oided urine specimen -or #a urine.! May need to catheteriAe or aspirate suprapu#ically -'&0. to o#tain a ood specimen! Erine dipstick! Microscopy, culture and sensiti$ity most common patho en is E. coli serotypes from #owel flora others include Klebsiella, Proteus, enterococci, S. saprophyticus! If child appears systemically ill, take #lood for cultures, C?C, urea, creatinine, lytes! =enal ultrasound for ma:or malformations and $oidin cystourethro ram -DCE>. should #e done in all children H months to H years old with ETI! =adiolo ical in$esti ations may #e postponed until the second ETI in irls o$er H years old due to hi her rate of #eni n ETI! &ostpone the DCE> 33I weeks to allow normaliAation of flow after ETI! Counseling8 0sk why the mother is concerned a#out sexual a#use! ETI alone is not a ood indicator! Explain that, in irls ETIs are common #ecause of short urethra and proximity to anal area! @escri#e front to #ack wipin after urination, and eneral hy iene! >i$e prescription for TM&B'M< -'eptra. HB14 m Bk Bd &, ,@ or "itrofurantoin H m Bk Bd &, ,@ x 2314 days if urine dip is positi$e for white cells and patient not aller ic! 0rran e follow up in J H weeks for re3culture of urine! Explain need for ultrasound and DCE> to rule out flow a#normalities which may threaten kidney function -this may #e postponed until second ETI in irls o$er the a e of H due to increased incidence of #eni n ETI., arran e these! Consider hospitaliAation for pyelonephritis, rehydration or child M I months old! 43$ 2lderly lady on -igo7in and Lasi7 'ith synco(al attac s$ Ta e a history$ B) Give a di**erential diagnosis$ 5hat investigations 'ould you orderC +istory8 onset, duration, chronolo y, description of e$ents! ;a$e the patient+s episodes #een witnessedF @oes the patient lose consciousness, are there warnin si ns of post3ictal symptoms, can the patient pre$ent an episode #y sittin down or other means! If diAAiness is a feature, is this li ht3headedness or true $erti o -$erti o means that the patient senses actual mo$ement of either the room or themsel$es.! %hat is the patient doin when these episodes occurF 0re muscle :erks a featureF 0ssociated fati ue, weakness, nauseaB$omitin , chest pain, shortness of #reath, palpitations, focal neurolo ical symptoms! &ast medical history -e! ! dia#etes, heart disease., medications, dru use, alcohol, smokin , aller ies, family history, re$iew of systems! -i**erential diagnosis8 Medication induced #radycardia -@i oxin., hypo$olemia -Lasix.! Cardio$ascular arrhythmia, $al$ular disease, su#cla$ian steal! Meta#olic8 hypo lycemia! Central ner$ous system8 seiAures -e! ! narcolepsy, tumor., strokeBTI0, cer$ical spondylosis, anxiety with hyper$entilation, middle ear -#eni n positional $erti o, acoustic neuroma, Meniere+s disease.! 0utonomic8 $a al, orthostatic hypotension! Infection in elderly can present in many ways8 ETI etc! -igo7in overdose8 anorexia, nausea, $omitin , #radycardia, $isual effects8 yellow, reen or white halo around o#:ects, decreased le$el of consciousness, a#dominal pain and diarrhea! EC> shows :unctional tachycardia, &DC+s, 0D #lock, and sometimes &'DT! Physical e7a. should include8 $itals, orthostatic ?&, check for si ns of dehydration -thirst, mucous mem#rane moistness, ;=, urine output, skin tur or, ?&., cardiopulmonary exam -see question P13., neurolo ic exam -see question P). and mini3 mental status exam -see question PR.! Investigations8 @i oxin le$el, C?C, lytes, urea, creatinine, I"=B&TT, lucose, EC>, HR hour ;olter monitor, echocardio ram, EE>, CT head, carotid @oppler! 4>$ 30 year old .ale& .arried 'ith / children$ 1rought in by (olice *or violent and dangerous behavior$ Ta e a history$ B) 5ould you ad.it this (atientC 5hat are the criteria *or a %or. !C +istory8 0ttempt to determine whether patient is sad -depressed., #ad -antisocial, reaction to stressful or frustratin e$ents, poor an er mana ement., or mad -mania, schiAophrenia.! Is the episode related to dru s of a#use or or anic -#rain tumor, meta#olic distur#ance.F Co$er history for depression -see question PR., mania -see question PR. and schiAophrenia -see question P)2. with mental status exam -see question PR. and mini mental -see question PR.! Criteria *or ad.ission8 patient requires o#ser$ation or medication in a controlled, safe settin for dia nosis, patient appears to #e a dan er to himself or others, en$ironment at home unsuita#le for the patient at this time, patient requires medical work up for or anic causes, patient in need of detoxification!

3R

Criteria *or a %or. !8 Qorci#le admission for assessment without ri ht to appeal, maximum 2H hours, can #e administered #y any licensed physician who has seen the patient within a week, #oth criteria must #e met8 1! &atient appears to #e dan er to himselfBherself or others! H! &atient appears to #e currently sufferin from a mental illness! 4D$ 4D year old .ale 'ith dys(hagia$ Ta e a history$ B) -i**erential diagnosis and investigations$ +istory8 see question P14! Investigations8 ?arium swallow -liquid and with marshmallow for transfer., endoscopy with #iopsy, CT chest, esopha eal manometry, HR hour p; reflux study! 44$ 4D year old .ale out(atient 'ith shortness o* breath& cough& s(utu.$ Ta e a history and (er*or. a (hysical e7a.$ %indings) Lobar consolidation 'ith yello'8green s(utu.$ B) Given a diagnosis o* (neu.onia& reco..end treat.ent$ +istory8 "ame, a e, occupation, tra$el history, pets! Cou h -acute, chronic, worse in any positionBseasonBni ht, anythin relie$e., sputum -what color, quantity, frequency, quality., hemoptysis -quantity, frequency, quality e! ! #lood tin edBclots., dyspnea -constant, duration, onset, frequency, se$erity, exercise tolerance, tri ers, alle$iatin ., fe$er, chills, malaise, fati ue, increase in asthmatic symptoms -wheeAe, cou h., precedin $iral illness! ,nset, chronolo y of symptoms, positional factors -orthopnea., chest pain, ankle swellin ! ;istory of C,&@F Medications, compliance with meds -o#ser$e use of puffers., dru s of a#use -alcohol., smokin , aller ies, past medical history, family history, re$iew of systems! Physical 27a.8 Cardiopulmonary exam as in question P13 and PHR! Treat.ent8 for dia nosis of community3acquired pneumonia, admit if patient is systemically ill -may ha$e septicemia., if the patient is de#ilitated or hypoxia is a feature -send #lood cultures and i$e oxy en.! 'tart ID cefuroxime 2)4 m ID qKh! 'witch to a more specific oral anti#iotic when culture results #ecome a$aila#le! Qor outpatient therapy8 Cla$ulin -amoxicillin C cla$ulanate. )44B1H) m &, ?I@! Qollow up in 1 week! @iscontinue therapy after 3 afe#rile days! Note8 this therapy does not co$er atypicals! &ractices $ary accordin to the treatment population8 ,ntibiotic thera(y *or (neu.onia Presentation Community acquired, no C,&@ Community acquired with C,&@ 0lcoholics 7 de#ilitated patients @ia#etics or hospital acquired Li ely Organis.s 'treptococcus pneumonia -typical. or Mycoplasma pneumoniae 7 chlamydia -atypical. add ;aemophilus influenAa add >ram ne ati$es, Le ionella and anaero#es -in aspiration. add 'taphylococcus aureus and in $ery ill patients8 &seudomonas aeru inosa

Thera(ies8 1! ID cefuroxime 2)4m ID qKh co$ers8 'trep, 'taph -unless it is M='0G., ram ne ati$es, anaero#es! Chan e to more specific oral a ent when or anism known! H! Cla$ulin -amoxicillin C cla$ulanate. )44B1H)m &, ?I@! Co$ers 'trep, 'taph, ram ne ati$es and anaero#es! 3! 'eptra -trimethoprim C sulphamethoxiAole. H ta#s &, ?I@ -or 1 @' ta# ?I@. co$ers 'trep, 'taph and ram ne ati$es! R! Erythromycin )44m &,BID QI@ co$ers 'trep and atypicals! )! &enicillin > 13H million units ID qRh effecti$ely co$ers 'treptococcus only! ,ral $ersion is penicillin D H)43)44m &, qIh! I! &iperacillin 3 ID qIh C To#ramycin Hm Bk ID qKh is a standard therapy for &seudomonas aeru inosa! &iperacillin also co$ers 'trep, ram ne ati$es and anaero#es, while To#ramycin adds further ram ne ati$e co$era e with syner y! 2! Clindamycin 344m ID qKh or &, qIh co$ers 'trep, 'taph and anaero#es! K! Dancomycin 1 ID q1Hh or 1H)m &, qIh for M='0 also co$ers 'trep and anaero#es! San*ord Guide8 =x influenced #y local pre$alences!
Presu.ed viral (neu.onia in adults8 cou ht, no sputum, dyspneaBhypoxia, interstitial infiltrates ,dults over age !?8 Community acquiredL non3hospitaliAed! InfluenAa, parainfluenAa, adeno$irus, ='D, hanta$irsu S.o ers8 '! pneumo, ;! influenAae, Moraxella catarhalis! Post8viral bronchitis8 '! pneumo, rarely '! aureus! No co8.orbidity 8 Mycoplasma, Chlamydia pneumoniae, $iral, rarely '! pneumo! ,lcoholic stu(or8 '! pneumo, anaero#es, %or in*luen6a , or 18 6ana.ivir 14m inhaled ?I@ x )d or oselta.ivir 2)m &, ?I@ x )d! 'tart within RK hours of symptoms onset! ,6ithro 4!) m &, x1 then 4!H) mBd or clarithro )44m &, ?I@ or clarithro 2R 1 m &, ,@! =x duration $aries8 rx until afe#rile 33)d! %or in*luen6a ,8 ri.antadine 144m &, HxBd or a.antadine 144m &, HxBd! %B with enhanced acti$ity $s! '! pneumo or O Ce(h / or ,M/CL K2)B1H)m &, ?I@ or do7y 144m &, ?I@ or telithro K44m &, ,@ x2314d!

3)

,dults over age !?8 Community coliforms! 2(ide.ic 8 Le ionaire+s! 1irds8 Non8ICM bed8 UP Ce(h 3 ID -erythro 1)3 In ICM8 -P Ce(h 3 ID C a6ithro acquiredL hospitaliAed! -"ote8 &sittacosis! Rabbits8 Tularemia! Parturient H4 m Bk Bd qIh or a6ithro )44m ID ,@.V )44m ID ,@. or -%B with se$ere '! pneumo in post3 livestoc or cats8 Coxiella #urnetii! ,ir'ay or -ce*uro7i.e C erythro. or -%B with enchanced acti$ity $s! '! pneumo.L splenectomy, myeloma, obstruction 8 0naero#es! enhanced acti$ity $s! '! pneumo. add P Ce(h 3 for dual co$era e if lymphoma pts!. suspect >m3ne enteric! O Ce(h /8 cefdinir 344m &, q1Hh, cefpodoxime proxetil H44m &, q1Hh, cefproAil )44m &, q1Hh, cefuroxime axetil H)43)44m &, q1Hh! P Ce(h 38 cefotaxime H m qR3Kh ID, ceftriaxone 13H m ID ,@, ceftriAoxime not as acti$e in $itro $s! '! pneumo!

4#$ Houng *e.ale 'ith .alaise& tender ly.(h nodes in the nec & le*t u((er Euadrant abdo.inal (ain$ Per*or. a (hysical e7a.$ B) Give a di**erential diagnosis$ Physical e7a.8 see question PH( and PR3! 0lso8 &alpate for lymph nodes in the neck, supra and infra3cla$icular, axillae, roin! Examine the oral ca$ity and pharynx! Check for rashes! -i**erential diagnosis8 neoplastic8 lymphoma, leukemia! Diral infection8 mononucleosis, ;ID, E?D! ?acterial8 syphilis! Inflammatory autoimmune disease8 sarcoidosis, lupus! Li$er disease with portal hypertension! 'erum sickness, aller ic reaction! 4?$ 4D year old .an 'ith ata7ia& di66iness& .acrocytic ane.ia$ Ta e a history$ %inding) (oor diet$ B) Give a di**erential diagnosis$ 5hat is the .ost li ely diagnosisC 5hat investigations 'ould you orderC +istory8 onset, chronolo y, description of symptoms! 'ettin in which symptoms occur! Qunctional limitations -dri$in , walkin , stairs, reachin upward.! @ifferentiate li ht3headedness from true $erti o -room or self spinnin .! &eripheral num#ness, psychiatric features8 mild depression, irrita#ility, paranoia -seen in ? 1H deficiency.! %eakness, eye symptoms, tremor! TI0 or stoke phenomena8 sudden neurolo ic deficit -loss of $ision, speech, motor or sensory chan es.! Check for heart pro#lems, hypertension, dia#etes! C0>E alcoholism screen as #elow, history of syphilis, M', hypothyroidism -decreases secretion of intrinsic factor. use of chemotherapeutic a ents -interfere with @"0 synthesis.! @iet, wei ht loss or ain, chronic diarrhea -mala#sorption., a#dominal pain! 'ymptoms of hypothyroidism! &re$ious astric sur ery! 'i ns of intracranial hypertension -hydrocephalus.8 mornin nausea, $omitin , headache! >eneral si ns of mali nancy8 anorexia, fati ue, ni ht sweats! &ast medical history, medications, dru sBalcohol, smokin , aller ies, family history, re$iew of systems! C,G2 Euestionnaire8 Control 9 ha$e you tried to cut down on your alcoholF 0n er 9 ha$e you e$er felt an ry when someone su ested you decrease your alcohol intakeF >uilty 9 ha$e you e$er felt uilty a#out your drinkin F Eye opener 9 do you sometimes ha$e a drink to et started in the mornin F -i**erential diagnosis8 ,ne.ia due to vita.in 1!/ de*iciency8 usually due to mala#sorption -Crohn+s, celiac disease etc!., lack of intrinsic factor pernicious anemia -auto3anti#odies a ainst astric parietal cells associated with thyroid and adrenal insufficiency., post astrectomy, lon term use of antacids, pancreatic insufficiency or malnutrition -$e an diet.! 'e$ere hypothyroidism! -ue to %olate :14; de*iciency8 &oor nutrition -alcoholism, po$erty, infancy found in reen leafy $e eta#les., mala#sorption, medication or dru 3induced -alcohol, anticon$ulsants, antifolates e! ! MT<, ?C&., increased need -pre nancy, prematurity, hemolysis, hemodialysis, psoriasis, exfoliati$e dermatitis.! ,ta7ia& di66iness8 %ernicke+s encephalopathy, hepatic encephalopathy, inner ear pro#lem -#eni n positional $erti o, acoustic neuroma, Meniere+s., postural hypotension, #rainstem stoke or TI0, intracranial tumor! Most li ely diagnosis8 Ditamin ?1HBfolate deficiency secondary to poor diet! Investigations8 C?C and differential with #lood smear, lytes, urea, creatinine, I"=B&TT, >>T, 0'T, 0LT, 0L&, serum folate, screen for serum ?1H, =?C folate, serum ferritin! 'chillin +s test8 measures a#sorption of ?1H! ?one marrow ?arium enema if suspect pernicious anemia -pernicious anemia is associated with #owel cancers.! 4"$ >/ year old .an *ound unconscious in the street$ ,((ears to have been struc in the head$ Per*or. (hysical e7a.$ %indings) GCS !!& unilateral body 'ea ness$ B) 5hat is your di**erentialC 2valuate C8s(ine *il.$ -escribe your initial treat.ent and investigations$ Initial .anage.ent8 rescusitate as required usin the 0TL'B0CL' format -see question PI.! +istory8 if a$aila#le8 "ature of collapse, precedin and su#sequent e$ents e! ! trauma, has patient e$er experienced similar symptoms #efore, did patient lose consciousness, were there seiAure3type phenomena, in:uries durin fall, duration of unconsciousness, post3ictal drowsiness, medications and dru s, smokin , aller ies, past medical history, family history, re$iew of systems!

3I

-i**erential diagnosis8 concussion, su#dural #leed, epidural #leed, #rain contusion, seiAure or post3ictal weakness, #rainstem or spinal cord in:ury! Clearing C8s(ines8 The principle of clearin C3spines is to rule out #oth #ony fractures and li amentous in:ury, either of which can make the spine dan erously unsta#le! Most emer ency physicians will clear the cer$ical spine in the case of an alert patient who has no pain on palpation of the dorsal spinous processes and a normal cross3ta#le lateral C3spine <3ray! If the patient has neck pain, flexionBextension plane films are done! These in$ol$e ently flexin , then extendin the neck and takin $iews at each extreme! The patient must #e alert enou h to warn the examiner of paresthesias in the hands or increased neck pain on mo$ement durin this procedure, which may indicate compromise of the neural elements! QlexionBextension $iews may #e done under fluoroscopy if the patient is not alert! Treat.ent8 normaliAe $itals, oxy en saturation, 0?>s, hydrate to maintain ?&, i$e #lood if necessary, correct coa ulopathy! Immo#iliAe cer$ical spine! Consider intu#ation -careful of the neck.! Control IC&, load with dilantin 1 ID to pre$ent seiAures, i$e H4* mannitol )4 ID, rapid sequence intu#ate with sux, fentanyl, etomidateBpropofolBketamine, spray cords with lidocaine and hyper$entilate to pC,H 3)mm; ! Consult "eurosur ery! CT head and neck! May need M=I for delineation of #rainstem or spinal cord in:uries! #0$ > year old boy 'ith cough *or 4 'ee s$ No i.(rove.ent on antibiotics 3 'ee s ago$ Ta e a history$ B) 5hat is your di**erential diagnosisC Give the .ost li ely diagnosis and describe a treat.ent (lan$ +istory8 "ame, a e! &rodromal illness, fe$er, malaise, rhinorrhea, sore throat, shortness of #reath, wheeAe! Is cou h producti$eF Color of sputum, quantity! 0ny chest painF 0 ra$atin and relie$in factors! ,nset of cou h, chronolo y, time of day -ni ht., worse with exposure to polluted air, cold air, dust, smoke, exercise, in rassy areas, weedy areas, forests, certain rooms, in #ed! @id symptoms impro$e with 0moxicillinF 0ller ic symptoms8 red eyes, itchin , itch in #ack of throat! Qamily pets! 0ir conditionin ! Type of #edclothes and pillows -feather, synthetic, foam.! =ecent chan e in the child+s en$ironment, different #edclothes, new room, chan e of season! 0re there smokers in the houseF &ast medical history, medications, aller ies, family history -asthma, aller y, CQ., re$iew of systems! -i**erential diagnosis8 asthma, #ronchitis, #ronchiolitis -upper respiratory tract infection., chronic sinusitis, rhinitis, T?, recurrent pneumonia, collapsed lun , cystic fi#rosis! ,((roach to cough8 1! &roducti$e8 #ronchiectasis, #ronchitis, a#scess, #acterial pneumonia, T? H! "onproducti$e8 $iral infections, interstitial lun disease, anxiety, aller y 3! %heeAy8 su ests #ronchospasm, asthma, aller y R! "octurnal8 asthma, C;Q, postnasal drip, >E=@, or aspiration )! ?arkin 8 epi lottal disease -croup. I! &ositional8 a#scess, tumor 2! @ifferential dia nosis8 a! 0irway irritants8 inhaled smoke, dusts, fumes, aspiration - astric contents, oral secretions, forei n #ody., postnasal drip #! 0irway disease8 E=TI includin postnasal drip and sinusitis, acute or chronic #ronchitis, #ronchiectasis, neoplasm, external compression #y node or mass lesion, asthma, C,&@ c! &arenchymal disease8 pneumonia, lun a#scess, interstitial lun disease d! C;Q e! @ru 3induced Investigations8 C<=, C?C, lytes, I"=B&TT, urea, creatinine, pulmonary function tests -J R years of a e.! Most li ely diagnosis8 asthma! Treat.ent8 0cute8 ,H to keep 'p,H J (H*! Qluids! XH3a onists8 'al#utamol -Dentolin. 4!43 ccBk in 3 cc "' qH4min $ia mask until impro$ement then q1hour as necessary! Ipratropium #romide -0tro$ent. if se$ere8 1 cc added to each of first 3 Dentolin masks! 'teroids8 prednisone H m Bk in E=, then 1 m Bk &, ,@ x R days! Chronic8 'odium cromo lycate -Intal. H puffs QI@! 0dd $entolin H puffs &=" if acute #reathlessness with wheeAe is a feature! Modify the home en$ironment to decrease contact with common aller ens8 dust mites, pollen, pet hair -especially cats.! Control dust with thorou h and re ular cleanin ! ?oil #edclothes, plastic underco$ers on mattress and pillows, remo$e ru s, install air conditioner! =emo$e pets! "o smokin in the house -second hand smoke is a cause of childhood asthma.! %arn parents of the symptoms of a se$ere asthma

32

attack, status asthmaticus and when to come to the E=! @iscuss treatment strate y, a re ular anti3immune medication -sodium cromo lycate or inhaled steroid. with &=" #ronchodilator! #!$ , 40 year old 'o.an 'ith a history o* atrial *ibrillation and congestive heart *ailure returns to the o**ice *or the results o* her -igo7in level& 'hich is subthera(eutic$ Ta e a history and counsel$ B) 5hat are the e**ects o* a -igo7in overdoseC +istory8 "ame, a e, occupation! 0sk a#out symptoms of atrial fi#rillation8 sudden fati ue, palpitations, eneral weakness, li ht3headedness, C"' em#olic phenomena -TI0, stroke.! 'ymptoms of @i oxin o$erdose which the patient may ha$e had in the past8 anorexia, nausea, $omitin , #radycardia, $isual effects8 yellow, reen or white halo around o#:ects, decreased le$el of consciousness! 'ymptoms of con esti$e heart failure8 ankle edema, shortness of #reath, orthopnea, paroxysmal nocturnal dyspnea! Cou h, wheeAe, hemoptysis! Is patient takin her @i oxin re ularly, if not, whyF Medications and whether patient is takin these, dru s and alcohol, smokin , aller ies, past medical history, family history, re$iew of systems! Counseling8 explain action of @i oxin8 pre$ents fi#rillation #y slowin the heart rate while increasin its force of contraction! ;elps #oth atrial fi#rillation and con esti$e heart failure! Must #e taken re ularly! @iscuss reasons why patient may not ha$e #een takin medication! Memory pro#lemF Concern a#out the side effectsF Qeels she no lon er needs the medicationF EmphasiAe the need to discuss these concerns with you #efore alterin medications! @iscuss symptoms of o$erdose which the patient mi ht watch for! Consider alternati$e medication if side effects are a pro#lem! @e$elop a follow3up plan to monitor compliance and cardio$ascular status! 0rran e follow3up! 2**ects o* -igo7in overdose8 anorexia, nausea, $omitin , #radycardia, $isual effects8 yellow, reen or white halo around o#:ects, decreased le$el of consciousness! #/$ 40 year old 'o.an 'ith hy(ercalce.ia on an insurance (hysical$ Ta e a history and (er*or. a (hysical e7a.$ %inding) sister had a (arathyroid adeno.a$ B) Give a di**erential diagnosis *or hy(ercalce.ia$ 5hat investigations 'ould you orderC +istory8 'ymptoms of hypercalcemia8 5?ones, 'tones, psychosis3#ased Moans, and a#dominal >roans6 fati ue, muscle weakness, arthral ias, renal colic due to nephrolithiasis, emotional la#ility -can pro ress to psychosis and coma., #one pain, a#dominal pain, nausea, $omitin , constipation, ileus, polyuria, polydipsia, nocturia! ,nset, duration of these! >eneral mali nancy symptoms8 wei ht loss, ni ht sweats, fati ue! ,rthostatic hypotension -0ddison+s.! ;eat intolerance, hyperacti$ity -hyperthyroid.! @iet, especially amount of milk and use of calcium supplements and antacids! Medications, dru s and alcohol, smokin , aller ies, past medical history -especially heart #urn, reflux, astritis, peptic ulcer., family history -especially of multiple endocrine neoplasia 9 ME"., re$iew of systems! Physical e7a.8 Trousseau+s si n -inflate ?& cuff, lea$e on 13H minutes, distal arm oes into titanic flexion, indicates hypercalcemia.! Inspect for si ns of 0ddison+s8 #ronAe skin tone, orthostatic hypotension, or of Cushin +s8 moon facies, striae, #uffalo hump! Chest8 palpate sternum and ri#s for #one pain, examine #reasts for si ns of mali nancy8 dimplin , mass! >eneral cardiopulmonary exam! 0#dominal exam, palpate li$er carefully for masses, percuss kidneys for pain! =ectal8 test stool for occult #lood! Examine lon #ones for strai htness and tenderness -&a et+s disease of #one.! -i**erential diagnosis8 parathyroid adenoma -hyperparathyroidism. due to solitary adenoma, hyperplasia, carcinoma, inherited ME" -multiple endocrine neoplasia 9 may also ha$e pituitary adenoma causin 0ddison+s or Cushin +s., mali nancy -myeloma, lun , #reast, squamous in any site., hi h #one turno$er -&a et+s disease of #one, hyperthyroidism, Dit 0 excess., $itamin @ in pharmacolo ic doses, milk3alkali syndrome -lar e in estion of milk and alkali, usually for astric hyperacidity., aluminum intoxication, dru s -thiaAides, lithium, CaC,3., familial hypocalciuric hypercalcemia! Investigations8 C?C, lytes -includin Ca 7 CaCC, M , &,R., urea, creatinine, al#umin, 0'T, 0LT, 0L&, >>T, I"=B&TT, serum cortisol, serum &T;, T';, serum protein electrophoresis -for monoclonal ammopathy of myeloma.! &lane <3rays of tender or malformed #ones, includin skull -see salt and pepper lesions.! CT head, thyroid, adrenals!

!""3
#3$ D0 year old 'o.an states that the Russians are lea ing radiation into her house$ Ta e a history 'ith .ental status e7a.$ ;istory should focus on depression, mania and schiAophrenia! Mental status exam! 'ee questions PR and P)2!

3K

#>$ Obese (atient 'ants hel( 'ith 'eight loss$ Counsel$ +istory8 "ame, a e, occupation, wei ht history, note ups and downs if present, past attempts to lose wei ht, successes, o#stacles, oals! @ietary ha#its8 frequency of meals, snackin , eatin at ni ht, foods eaten, #in e eatin , uilt a#out food, hoardin , concealin eatin from others! Estimate daily caloric intake! 0lcohol intake! 'mokin , dru use! Exercise history! ,$erwei ht relati$esF @iseases associated with wei ht ain8 hypothyroidism, @M type II, Cushin +s, ma:or depression, anxiety disorder, some medications -TC0, steroids, ,C&.! &ro#lems associated with o$erwei ht8 out, sleep apnea, cholecystitis, #ack pain, cardio$ascular disease, hemorrhoids, lower lim# :oint pain and osteoarthritis! %hy is patient seekin medical help for this nowF Crisis in patient+s life, stress, anxietyF 0ssess patient+s self3ima e8 does patient feel underwei ht, o$erwei ht or normalF @oes patient feel that wei ht interferes with healthF %ith acti$itiesF 'creen for eatin disorders! Medications, dru s and alcohol, aller ies, past medical history, family history, re$iew of systems -include sleep ha#its, apneic spells, ,TC medications.! Physical e7a.8 $itals, calculate ?MI -#ody mass index. O wei ht -k .Bhei ht H -mH.! Ideal is H43H) for a male, #ut $aries dependin on frame! H)3348 o$erwei htL 3433)8 o#eseL J R48 mor#idly o#ese! Ese standard hei ht3wei ht ta#les for males and females to au e percent o$erwei ht -JH4* O o#ese.! @irect physical to pertinent positi$es from history! Inspect for Cushin oid features8 moon facies, #uffalo hump, striae, $isual field defects! Qat distri#ution8 centripetal fat associated with reater heart disease and dia#etes risk! Qundoscopic exam for retinopathy! Cardiopulmonary exam, a#dominal exam -not li$er siAe.! Check for si ns of hypothyroidism8 hypothermia, oiter, dull facial expression, lid la a#sent, lids droop, perior#ital swellin , hair is sparse, coarse and dry, skin is also coarse, dry scaly and thick! &atients are for etful, show chan e in personality which may pro ress to psychosis, deep tendon reflexes show slow return phase -#risk contraction and slow relaxation., leukonychia -whitened nails., oran e palms and soles due to carotene deposition, #radycardia, pericardial effusion, pleural effusions, myxedema, non3pittin edema, carpal and tarsal tunnel ner$e compression due to myxedema, causin paresthesias and num#ness in the hands and feet! Counseling8 Moti$ation8 how would #ein at ideal #ody wei ht impro$e the patient+s lifeF EmphasiAe health, lifestyle, self esteem, relationship #enefits! @iscuss nutrition3related pro#lems8 heart disease, o#esity, hypertension, osteoporosis, anemia, dental decay, cancer, astrointestinal disorders, respiratory compromise, hi h lipids, dia#etes, sleep apnea, osteoarthritis! @iscuss diets tried and why these failed! Qad diets in$ol$e unusual or extreme eatin patterns and are not desi ned to #e maintained for a lifetime therefore these should #e discoura ed! %ei ht loss a ent &onderal no lon er a$aila#le! ''=Is such as &axil may assist with wei ht loss, unfortunately, when the dru is discontinued, most people re ain wei ht! Explain that the #rain has a satiety set point which can #e reset o$er time with reduction in caloric intake! %arn that the #ody+s a#ility to determine caloric content is $ery ood, and will not #e fooled #y so3called diet products! =ecommend a #alanced diet consistin of ordinary foods, with three distinct meals per day of small siAe! "o eatin at ni ht and #e careful of snacks! Inform patient that he will #e hun ry for at least the first two weeks of reduced intake! 'u est $isualiAation techniques, redirection of interests, and to think of hun er as a si n of positi$e pro ress on wei ht loss! >roup support can #e #eneficial too8 %ei ht watchers, o$ereaters anonymous etc! #eha$ior modification! -ietary reco..endations8 reduce fat to H4* of caloric intake! Ideal caloric intake can #e estimated at K314 CalBl#s -ideal wei ht. for females and 1431H CalBl#s -ideal wei ht. in males! EmphasiAe that caloric intake is more important for wei ht loss than food composition -i!e! excessi$e calories lead to wei ht ain e$en if they are non3fat.! 27ercise reco..endations8 sudden intense exercise in sedentary patient unwise! Exercise tends to pro$ide a ood excuse for o$ereatin and may #e dan erous in an o#ese patient! =ecommend mild daily exercise such as 1 hour walkin per day! More $i orous exercise can #e initiated when wei ht is lost! 0rran e re ular follow3up for #ody mass monitorin and counselin ! #D$ ,lcoholic s.o er 'ith cough& s(utu.& shortness o* breath$ +istory and (hysical e7a.$ Order investigations$ 'ee question PII! #4$ #0 year old .an 'ith dys(hagia$ Manage$ 'ee question PI)! ##$ 4> year old 'o.an 'ith resting tre.or$ Per*or. *ocused (hysical e7a.$

3(

+all.ar s o* Par insons disease8 T=0&8 Tremor -rest, pill3rollin , R32 ;A, can #e suppressed #y $oluntary mo$ement., =i idity -lead pipe and co 3wheelin ., 0kinesiaB#radykinesia, &ostural insta#ility -festinatin ait, retropulsion, falls.! =emem#er to explain to the examiner what si ns of &arkinson+s you are lookin for at each step in the exam! ,ther features include8 mask3like face, lack of #linkin , #lepharoclonus -flutterin of closed eyelids., dyspha ia, droolin , hypophonia, micro raphia, ait8 start hesitation, small shufflin steps, loss of arm swin , su#cortical dementia -apathy, for etful, poor a#ility to use knowled e.! Physical e7a.8 $itals, o#ser$e patient at rest8 look for pill3rollin tremor in the upper lim# which is worst at rest, may also ha$e head tremor -titu#ation., stooped posture, open3mouthed, mask3like face, enerally hypokinetic with decreased #linkin and droolin ! Mini mental status8 dementia associated with &arkinson+s -)4* of patients., may find poor short term memory, poor concentration, a#straction, micro raphia! @epression may also #e a feature, screen for depression8 M'I>EC0&'! Cranial ner$es, #ody power, pronator drift, deep tendon reflexes and ?a#inski are normal in &arkinson+s! Tone8 #ilateral lead pipe -constant. ri idity with possi#le co 3wheelin due to tremor superimposed on passi$e motion! Test el#ow, forearm rotation and knee #y applyin rapid passi$e motion while feelin the muscle tendon! Cere#ellar testin 8 fin er3nose and heel3shin tests show impro$ement of tremor with intention -i!e! a restin tremor rather than an intention tremor., rapid alternatin mo$ements are poor #ilaterally in &arkinson+s and =hom#er is positi$e due to postural insta#ility! The &arkinsonian ait is unsteady and shufflin with small steps, decreased arm swin in , and a tendency to fall forward or #ackward! &atients may try to increase forward speed to keep from fallin -festination.! Mini8Mental Status8 orientation to timeBplace -) pts, year, season, month, day, day of weekL ) pts, country, pro$ince, city, hospital, floor., memory -3 pts, honesty, tulip, #lackL 3 pts, delayed recall., attentionBconcentration -) pts, serial 2s, 5%,=L@6 #ackwards., lan ua e tests8 comprehension -3 pts, three point command., readin -1 pt, 5close your eyes6., writin -1 pt, complete sentence., repetition -1 pt, 5no ifs, ands or #uts6., namin -H pts, watch, pen., spatial a#ility -1 pt, intersectin penta ons. #?$ !/ year old *e.ale 'ith *ever& (hoto(hobia& nec sti**ness$ Manage$ +istory8 name, a e, onset of symptoms, duration, increasin or decreasin in se$erity, fe$er, nausea, $omitin , photopho#ia, phonopho#ia, neck stiffness, headache, rash! ImmunocompromisedF -;ID, asplenia, prematurity., paramenin eal infectionF -sinusitis, mastoiditis., en$ironmental risk factorsF -day3care centers, household contact, tra$el to endemic re ions.! Medications or other inter$entions triedF ;istory, se$erity, chronolo y of mi raines! &remonitory $isual distur#anceF =ecent neurosur ery, head trauma, other illnessF Contacts with menin itis at school or work! Medications, dru sBalcohol, aller ies, past medical history, family history, re$iew of systems! Physical e7a.8 $itals, >C', note eneral appearance of patient 9 if patient is $ery ill, a resuscitation approach -0TL'B0CL'. may #e appropriate, orientation! Inspect for menin ococcemial rash! Cranial ner$es8 pupillary reflexes, note photopho#ia if present, extraocular muscle mo$ement, check for dou#le $ision, $isual fields, facial sensation and mo$ement, ross hearin , sternocleidomastoid and trapeAius power! Tone8 passi$e rapid mo$ement at el#ows, rotation of forearms and flexionBextension of knees! &ronator drift! Cere#ellar testin 8 fin er3nose, heel3shin, rapid alternatin mo$ements of forearms! &ower of deltoids, triceps, #iceps, wrist extension and flexion, fin er a#duction and adduction, psoas, quadriceps, hamstrin s, ankle dorsiflexion and plantar flexion! @eep tendon reflexes at triceps, #iceps, #rachioradialis, knee, ankle, ?a#inski! Li ht touch, pin prick o$er lim#s and #ody, $i#ration sense at :oints! 'i ns of menin ismus8 Terni +s8 pain in the neck on extension of the knee with the hip in (4 de rees of flexion! ?rudAinski+s8 pain on passi$e flexion of the neck! "uchal ri idity! ,pisthotonos8 spasm in which head and heels are #ent #ackward and #ody #owed forward! Investigations8 C?C, lytes, I"=B&TT, urea, creatinine, lucose, #lood cultures, 0?>, CT head followed #y lum#ar puncture if history and physical are suspicious for raised intracranial pressure -lum#ar puncture may, rarely, precipitate #rain herniation in the presence of raised IC&.! Treat.ent8 isolation, initiate ID anti#iotics immediately -#efore CT and L&. if the clinical picture is suspicious for menin itis! Cefotaxime H ID qRh C ampicillin )4 m Bk ID qIh! Consult ICE! Consider intu#ation and intensi$e mana ement of IC&! Monitor lucose, acid3#ase 7 $olume status and mana e as needed! 'teroids for ;i# menin itis - i$e early.! 0nticon$ulsants for seiAures! =eport to pu#lic health! #"$ !# year old *e.ale 'ith chronic diarrhea$ Ta e a history$ 1ac ground8 Causes of chronic diarrhea in a 12 year old8 Crohn+s, ulcerati$e colitis, irrita#le #owel, mala#sorption -Celiac disease, tropical sprue., lactose intolerance, intestinal infection -C! difficile, iardia, amoe#iasis., pancreatic dysfunction, unusual #ut may #e laxati$e a#use!

R4

+istory8 onset of diarrhea, duration, consistency and color of stools, do they floatF Is there #lood or mucousF Qrequency of ?MsBday, wei ht loss, appetite! @ietary history! Is diarrhea worse with milk -lactose intolerance tends to produce explosi$e diarrhea after milk in estion hereditary.F Laxati$e use! Ese of anti#iotics in the past I weeks! Tra$el history! Qe$er, nausea, $omitin , infectious illnesses! 0ssociated a#dominal pain, fati ue, u$eitis, mouth or anal ulcers, ankylosin spondylitis, sacroiliitis, renal pro#lems -due to mala#sorption., arthritis -these are associated with Crohn+s.! Malnutrition si nsBsymptoms8 lassitude, weakness, hair fallin out, skin rash, easy #ruisin , wei ht loss, anemia, neurolo ic findin s -carpal and tarsal ner$e compression, confusion, emotional la#ility, loss of corticospinal $i#ration and position sense., lossitis! Medications, dru s and alcohol, smokin -decreases risk and symptoms of inflammatory #owel disease., past medical history -I?@, a#dominal sur ery., family history, re$iew of systems!
Classi*ication o* Chronic -iarrhea Ty(e In*la..atory Elcerati$e colitis Crohn+s disease Mali nancy8 lymphoma, adenocarcinoma Os.otic In estion Lactose intolerance Medications, laxati$es Maldigestion and Malabsor(tion &ancreatic insufficiency ?ile salt deficiency Celiac 'prue %hipple+s disease ?owel resection Secretory ?acterial enterotoxins 'ecreta o ues 9 DI&, astrin, carcinoid %unctional I?' Characteristics Qe$er, hematocheAia, a#dominal painL usually wei ht loss with carcinoma

'tool $olume decreases with fastin Increased stool osmotic ap8 fecal U"aCV C UTCV M Y serum osmolality 9 H) mmolBL %ei ht loss, fecal fat J 2314 BHRh stool collection 0nemia, hypoal#uminemia

Lar e $olume -J 1 LBd.L little chan e with fastin "ormal stool osmotic ap8 'ecretory8 fecal U"aCV C UTCV O Y serum osmolality

?0$ Houng .other 'ith blac eye& hit by her boy*riend$ Manage$ 5arning signs of domestic $iolence8 o#sessi$e need to control $ictim #y controllin money, restrictions on oin out, not allowed to see certain people! 5The 'panish Inquisition6 9 where were you, who were you with, what did you doF 'ocial isolation! Threats! Der#al a#use aimed at decreasin self3esteem of $ictim! Cycle of $iolence followed #y remorse, then increased $iolence! Ris *actors for domestic $iolence8 social isolation, po$erty, su#stance a#use, partners+ parents had a#use relationship, personalityBcharacter disorder or mental illness! +istory8 @escri#e $iolent episode, what tri ered itF %ere o#:ects used as weaponsF In:uriesF %as the #oyfriend remorseful afterwardF ;istory of pre$ious episodes of $iolence or loss of temper #y #oyfriend! %hat is patient+s responseF ;as patient #een in an a#use relationship #eforeF %ere the patient+s parents in an a#usi$e relationshipF Is #oyfriend controllin , does he restrict her acti$ities, question her excessi$ely after she has #een out, en a e in $er#al a#use or threatsF Is the $iolence increasin in se$erityF 0re there children in the houseF %ho are the #iolo ical parentsF 0sk a#out $iolence to the children, sexual a#use! @oes the patient or her partner a#use alcohol or other dru sF Is money a pro#lemF Is the #oyfriend willin to seek helpF Counsel8 Explain that the #oyfriend hittin the patient is a criminal assault and an example of domestic $iolence! @omestic $iolence tends to increase o$er time unless the $ictim lea$es, or the a#user and couple seek therapy! Dery often, women don+t lea$e their a#usi$e partner until they are seriously hurt or #efore they are killed! @omestic $iolence #etween adult partners tends to #e reflected in future #eha$ior of children who are exposed to it and there is a risk of $iolence to the children! Child a#use is a criminal act and if suspected, is reporta#le to police #y law! 'pousal a#use is also a criminal act #ut is not reporta#le #y law! =ecommend that the patient not return to the a#user if there is risk to her safety -e! ! not the first assault, a#user not remorseful.! If the patient does return, an exit plan should #e de$eloped to ensure patient safety! @ocument all e$idence of a#use -pictures, sketches. and related $isitsL quote patient directly in chart! 0lternati$ely, the patient can contact the police to o#tain a restrainin order on the a#user! @e$elop a plan with the patient to seek alternate li$in arran ements -women+s a#use shelter., enlist the help of patient+s support structure -friends, other family mem#ers., contact the police -patient should #e informed that, if contacted, the police will lay char es whether the patient

R1

wants to or not.! Counsel patient on how to enter into controlled, safe, contact with the a#user to discuss possi#le therapy for an er mana ement and controllin #eha$iors, with therapy as a couple for relationship pro#lems! 'ocial worker referral! 0rran e follow up! ?!$ Patient ic ed by horse& no' hy(otensive in e.ergency de(art.ent$ Manage$ Rescusitation8 The 0CL' and 0TL' format is useful8 see question PI! +istory8 if a$aila#le8 %here was the patient kicked, when, was there mechanism of action for other in:ury, i!e! collapseBfall, precedin and su#sequent e$ents, did patient lose consciousness, duration of unconsciousness, did the patient fall, in:uries durin fall, medications and dru s, smokin , aller ies, past medical history, family history, re$iew of systems! Manage.ent8 Qor unsta#le #lood pressure, #lood loss is the most likely cause -C?C may #e normal with a lar e acute #lood loss., type and cross for R3I units dependin on estimated se$erity and han #lood as soon as possi#le! Is the patient+s a#dominal wound the first priorityF Examine for head in:ury, other in:uries! Check for a#normalities on a#dominal exam su esti$e of splenic rupture! %ith clear sur ical a#domen -ri idity, re#ound, a#sence of #owel sounds., consult eneral sur ery and prepare patient for immediate ,=! If less se$ere a#dominal #leedin is suspected, consider CT a#domen or dia nostic peritoneal la$a e! ?/$ Suturing station$ Suture laceration on a rubber *orear.$ Choose suture ty(e$ Is a tetanus booster reEuiredC Suturing station8 past years ha$e included a point for introducin yourself to the ru##er forearm! +istory8 "ame, a e, occupation, mechanism of in:ury, en$ironment in which in:ury occurred! ;ow lon since the in:ury! 0ny distal loss of sensation, motor powerF ,ther in:uries! &ast medical history, medications, dru Balcohol use, smokin , aller ies, re$iew of systems! Choice o* suture8 use non3a#sor#a#le monofilament such as 334 &rolene or Ethylon! ?raided sutures can har#or #acteria and a#sor#a#les cause more inflammatory reaction in the skin! >i$en a choice #etween 334 silk -a #raided non3a#sor#a#le. and chromic ut -a #raided a#sor#a#le., choose silk! TechniEue8 for small wound use interrupted sutures startin at the middle of the wound! 0nesthetiAe with lidocaine without epinephrine, cleanse and irri ate wound #eforehand and drape, lo$e and o#ser$e sterile technique! Tetanus i..uni6ation status8 @ose Td 4!) mL IM! Esual schedule of immuniAations for tetanus -prepared as diphtheria3 tetanus toxoid plus pertussis $accine, i!e! @T&. is H, R, I, 1K mo!, R3I years, Td -diphtheria3tetanus toxoid. at 1R31I years and repeat q14y! Last tetanus i..uni6ation treat.ent 43) years a o none )314 years a o #oost -Td. J14 years a o #oost and i$e immuno lo#ulin -passi$e. uncertain #oost and i$e immuno lo#ulin -passi$e. %ollo' u(8 %arn of si ns of wound infection! =emo$e sutures in 2 days -) days on the face to minimiAe scarrin , the face heals faster and is less likely to #ecome infected due to #etter #lood supply.! =ecommend Tylenol plain if pain is a pro#lem! ?3$ %e.ale& # 'ee s (regnant 'ith lo'er abdo.inal (ain and vaginal bleeding$ Ta e a history$ %inding) (revious s(ontaneous abortion at 4 'ee s gestation$ B) Give a di**erential diagnosis& 'hich is .ost li elyC 5hat three *indings on vaginal e7a.ination 'ould con*ir. this diagnosisC -e*initions8 1! Threatened abortion O any uterine #leedin or crampin in the first H4 weeks of estation! H! Inevitable abortion O intolera#le pain or #leedin x 1 week, cer$ix open! Life3threatenin to the mother! 3! Inco.(lete abortion O mem#ranes ruptured, part of products of conception passed, cer$ix open! R! Co.(lete abortion O uterus empty, #leedin and complete sac and placenta passed, cer$ix open! )! Missed abortion O fetal death and retention of productsL presents as pre nancy not pro ressin , cer$ix closed! I! +abitual O 3 or more consecuti$e spontaneous a#ortions!

RH

H4334* of pre nancies ha$e uterine #leedin or crampin in the first H4 weeks! ;alf of these a#ort! Most spontaneous a#ortions are associated with a#normal fetus! +istory8 patient I@ -name, a e, occupation., >T&0L, weeks of estation! ,nset of #leedin , duration, estimate quantity -num#er of pads soaked., color and consistency of #lood, associated fe$er, chills, a#dominal discomfort, li ht headedness! &ro#lems with pre$ious pre nancies, pro#lems in this pre nancy, medical $isits to this point, in$esti ations done! 0ssociated a#dominal trauma -accident or a#use., dru use -cocaine., father and mother+s #lood type, medications, alcohol, smokin , past medical history -dia#etes, lupus., family history, re$iew of systems! -i**erential diagnosis8 threatened a#ortion, incomplete a#ortion, non3uterine #leedin source trauma8 post3coital $s! lesion8 cer$ical polyp, neoplasm etc! -$a inal, cer$ical, $ul$ar., a#normal pre nancy -ectopic, molar., physiolo ic #leedin -due to placental de$elopment.! Most li ely diagnosis8 incomplete a#ortion! Three *indings which would confirm this8 ruptured mem#ranes, products of conception passed, cer$ix dilated -os open.! ?>$ 40 year old .ale 'ith he.o(tysis and shortness o* breath$ Ta e a history$ %indings) history o* C,-& +TN& hasnt been ta ing antihy(ertensives *or si7 'ee s$ B) describe CFR& :sho's enlarged heart& u((er lobe vascular redistribution& Kerley 1 lines& bilateral interstitial in*iltrates and bilateral s.all e**usions;$ Read 2CG) sho's B 'aves and inverted T 'aves in <!8>$ 5hat is the diagnosisC +istory8 name, a e, occupation! ,nset of symptoms, duration, time of day! ;as patient had these #eforeF 'mokin history! &relude of fati ue, ankle edema, orthopnea, paroxysmal nocturnal dyspnea, palpitations, chest painBhea$inessBti htness, pain in left arm or :awBteeth! ;istory of an ina, other cardiac pro#lems! ;istory of >I #leeds, reflux, $arices, astritis, peptic ulcer, C,&@ history, cou h, sputum, wheeAe! ;istory of immo#iliAation, le pain or swellin , pre$ious @DT, &E! Medications 9 has patient #een takin themF @ru s, alcohol, aller ies, smokin , sur ical history, family history, re$iew of systems! CFR8 consistent with pulmonary edema and C;Q! -iagnosis8 EC>8 anterior wall myocardial infarct! ?D$ / year old child 'ith " 'ee history o* cough& on ,.o7icillin *or / 'ee s$ Ta e a history$ B) Give a di**erential diagnosis$ 5hat investigations 'ould you orderC 'ee question P24! ?4$ Houng *e.ale 'ith secondary a.enorrhea *or 4 .onths$ Ta e a history$ B) 5hat investigations 'ould you order$ Give a di**erential diagnosis$ 5hat is the .ost li ely diagnosis& 'hat results 'ould con*ir. this diagnosisC +istory8 a e of menarche, re ularity of pre$ious menses, flow, duration, accompanyin cramps, #loatin , psychic distur#ance! ;eadache, $isual field distur#ance -for sellar tumor.! 'i ns of $iriliAation8 increased quantity and coarseness of #ody hair and facial hair, acne, increased sexual dri$e, increased muscle #ulk! >alactorrheaF @iet history8 has patient lostB ained wei ht latelyF Thyroid symptoms8 ener y le$els, emotional la#ilityBdepression, cold intolerance, or feels hot, :umpiness! Exercise history8 is patient en a in in $i orous exercise such as runnin F 'exual history8 contraception, is pre nancy a possi#ilityF Medications, dru s and alcohol use, smokin , aller ies, past medical history with sur ical history, family history, re$iew of systems! ,((roach8 ;istory and physical pre nancy test T'; and &rolactin -hi hBlow hypoBhyperthyroidL hi h, J 144, or symptoms of hyperprolactinemia CT to rule out tumor. pro esterone challen e -C$e withdrawal #leed ano$ulationL no withdrawal #leed end3or an failure or outlet o#struction. Q';, L; -hi h o$arian failureL low hypothalamic dysfunction. Investigations8 XhC>, T';, serum L;, Q';, serum prolactin, serum testosterone, sex3hormone #indin @;E03', pro esterone trial for uterine #leedin -indicates functional endometrium.! lo#ulin -';?>.,

-i**erential diagnosis8 pre nancy, polycystic o$ary syndrome -'tein3Le$enthal syndrome., hypothalamic dysfunction, excessi$e exercise, stress, wei ht loss, adrenal dysfunction -e! ! Cushin +s., thyroid dysfunction, prolactinoma, hypopituitarism!

R3

Most li ely diagnosis8 &C,' Con*ir.ed by8 ele$ated L;, low or normal Q'; with well estro eniAed $a inal mucosa, increased serum andro ens, o$arian cysts seen on ultrasound! ?#$ Mother 'ith lo' birth 'eight baby& 9ust delivered$ Ta e a history$ B) On (hysical e7a. o* the baby you *ind e.aciation 'ith 'rin led yello' s in and yello' tears$ 5hat is the (roble.C Give three underlying causes *or this$ Give t'o (otential (roble.s 'hich .ay arise in the ne7t >? hours$ Mothers obstetrics and ne'born history8 see question PH)! Causes o* intra8uterine gro'th restriction :IMGR; 1! symmetric IE>= -normal head to #ody siAe., familial, maternal estational infections -mnemonic 9 To=C;., toxoplasmosis -carried in cat feces., ru#ella, cytome alo$irus -CMD., herpes H! 0symmetric IE>= -small #ody.8 placental insufficiency due to maternal malnutrition, smokin , dru s and alcohol, illness durin pre nancy -e! ! Crohn+s., hypertension! 3! Naundiced, emaciated #a#y8 hyper#iliru#inemia! Three underlyin causes8 0?, or =h incompati#ility, neonatal li$er insufficiency -CMD. and sepsis -T,=C;.! ,ll causes o* neonatal 9aundice8 Encon:u ated O physiolo ic neonatal :aundice ,= patholo ic8 hemolytic 0?,=h incompati#ility, neonatal sepsis, splenome aly, hereditary spherocytosis, >I&@ etc!L non3hemolytic #reast milk :aundice, #reakdown of cephalohematoma, polycythemia, sepsis, >il#erts, Cri ler3"a::ar, hypothyroidism! Con:u ated8 >I o#struction in fetus -increases enterohepatic circulation., #ile duct o#struction, dru 3induced and multiple other less common causes! T'o (otential (roble.s arising in the ne7t >? hours8 kernicterus -hyper#iliru#inemic seiAures and #rain dama e8 deposition of #iliru#in in the #rainstem and #asal an lia leadin to mental retardation, cere#ral palsy, hearin loss and paralysis of upward aAe., hydrops fetalis - eneraliAed edema, includin pulmonary, with hi h output heart failure.! ??$ , young .an *ell 'hile inebriated and sustained a laceration to the right 'rist$ Per*or. a *ocused (hysical e7a.$ %indings) nu.bness on ulnar side o* hand& ,llens test sho's no ulnar artery re*ill& %-S i.(aired in little and ring *inger$ B) 5hat *our structures 'ere laceratedC Manage.entC Physical 27a.8 <ascular8 capillary refill, is hand pink and warmF 0llen+s test8 compress ulnar and radial arteries at the wrist, ha$e patient open and close hand to remo$e #lood, then release one side 9 the hand should flush pink due to #lood supply from the side released! -Esually used to demonstrate function of ulnar artery.! @oppler pro#e! Sensory8 di ital ner$es8 check two point discrimination on either side of each di it! &in3prick sensation8 Median ner$e territory is the palmar surface of the thum#, and the palmar surface and dorsal tips of the index, middle and thenar side of the rin fin ers! The ulnar side of the hand is ulnar ner$e, and the dorsal surface of the thenar side of the hand is radial ner$e inner$ated! The median ner$e also inner$ates most muscles of the thenar eminence, and the 1st and Hnd lum#ricals! The thum# is weak in a#duction at (4 de rees to the plane of the hand in median ner$e dysfunction! The ulnar portion of the palm is supplied #y the cutaneous #ranch of the ulnar ner$e, while the thenar portion is supplied #y the cutaneous #ranch of the median ner$e! Neuro.otor e7a.ination o* the +and Median Sensory =edial aspect of index fin er pad Motor 27trinsic Qlex @I& of index fin er -Q@&. Motor Intrinsic Thum# to ceilin with palm up -a#ductor pollicis #re$is. Mlnar Elnar aspect of little fin er pad Qlex @I& of little fin er -Q@&, extensor carpi radialis. 0#duct index fin er -first dorsal interosseous. Radial @orsal we#space of thum# Extend wrist and thum# -extensor pollicis lon us.

Motor8 -don+t ask patient to apply force a ainst resistance as this may rupture a partially se$ered tendon 9 test acti$e =,M only.! Median ner$e8 thum# a#duction! Elnar ner$e8 fin er spread a ainst resistance! =adial ner$e8 wrist extension! Q@' flexes the MC& and &I& :oints of the fin ers, while Q@& -flexor di itorum profundus. flexes the @I&! To test Q@& function hold the MC&B&I& :oints in extension and ask patient to flex @I&s! ?oth the Q@' and Q@& can flex the entire fin er, #ut the Q@& tends to flex them all at once, while the Q@' can flex fin ers in isolation! To test Q@'8 hold all fin ers except the one you are testin in extension and ask patient to flex the remainin fin er!

RR

Tendon 27a.ination o* the +and MCP 27tensor tendons Communis %le7or tendons Intrinsics -lum#ricals.

PIP Extensor di italis -lateral #ands. Qlexor di itorum superficialis

-IP Intrinsics -lateral #ands. Qlexor di itorum profundus

'tructures superficial to the flexor retinaculum -in order from ulnar to thenar.8 ulnar ner$e, ulnar artery, cutaneous #ranch of ulnar ner$e, palmaris lon us tendon, cutaneous #ranch of median ner$e! 'tructures immediately deep to flexor retinaculum -ulnar to thenar.8 R flexor di itorum superficialis tendons -Q@'., median ner$e, palmaris lon us tendon, cutaneous #ranch of median ner$e! Structures lacerated8 i$en diminished ulnar territory sensation, 0llen+s test shows no refill from ulnar circulation, and Q@' weakness in little fin er and rin fin er, the followin structures were included in the laceration8 ulnar ner$e, ulnar artery, flexor retinaculum, ulnar two di$isions of Q@'! Treat.ent8 Clean and explore wound under local anesthetic and sterile conditions! Consult plastic sur ery for micro$ascular repair! If at ni ht, may suture skin closed and arran e for patient to #e seen #y plastic sur eon the next day! ?"$ , young .an sustains a head in9ury on *alling *ro. his bicycle$ Patient has been he.odyna.ically stabili6ed$ Per*or. a *ocused neurological e7a.$ B) Lateral s ull and lateral C8s(ine F8rays (rovided$ ,re they adeEuateC ,re they nor.alC The (atient has continuing sanguinous discharge *ro. his nose$ 5hat is the li ely cause o* this$ Treat.entC ?e in #y assessin the le$el of consousness usin the >las ow Coma 'cale8 Glasgo' Co.a Scale :GCS;8 Glasgo' Co.a Scale 2ye O(ening -E. 'pontaneous R To speech 3 To &ain H ne$er 1 <erbal Res(onse ,riented and con$erses Confused con$ersation Inappropriate words Incomprehensi#le sounds "one -D. ) R 3 H 1 1est Motor Res(onse ,#eys commands LocaliAes pain %ithdrawal to pain 0#normal flexion -decorticate. 0#normal extension -decere#rate. "il -M. I ) R 3 H 1

N18 'tandard painful stimulus is ru##in the knuckle on the sternum! Qor withdrawal, apply pressure on the #ase of the nail #ed with a pen! @ecorticate posture is arm flexion with le extension on the same side of the #ody, may #e unilateral or #ilateral! Indicates a lesion a#o$e the #rainstem! @ecere#rate posture is arm and ipsilateral le extension, may #e unilateral or #ilateral, indicates #rainstem in$ol$ement! 0 >C' of K or less is considered an indication for intu#ation #ecause of the risk of poor protection of the airway from aspiration! +ead and Nec 8 inspect for lacerations and contusions, cranial ner$es8 extra3ocular mo$ements, $isual fields #y confrontation, pupillary reacti$ity, doll+s eyes -careful of neck, may not #e a#le to turn head enou h to do this., and accommodation, corneal reflex and facial sensation, palpate facial #ones for sta#ility, look in nose and ears for #lood or C'Q leaks, hemotympanum, facial muscle power, ross hearin , sternocleidomastoid power and trapeAius power! Check oral ca$ity, a reflex, palpate dorsal cer$ical spines for pain and ali nment, is the trachea midlineF &ronator drift, cere#ellar tests8 fin er3nose, heel shin, rapid alternatin mo$ements -dysdiadocokinesis.! ?ody power, tone, sensation! @eep tendon reflexes, ?a#inski! Lo roll patient onto #ack, inspect, palpate spine! I* GCS is signi*icantly less than !D8 ;ead and "eck8 inspect for lacerations and contusions, cranial ner$es8 pupillary reacti$ity, doll+s eyes -careful of neck, may not #e a#le to turn head enou h to do this., corneal reflex, palpate facial #ones for sta#ility, look in nose and ears for #lood or C'Q leaks, hemotympanum! Check oral ca$ity, a reflex, palpate dorsal cer$ical spines for ali nment, is the trachea midlineF ?rainstem -#reathin pattern.! @=E for sphincter tone! @eep tendon reflexes, ?a#inski! Lo roll patient onto #ack, inspect, palpate spine! -i**erential diagnosis8 concussion, su#dural #leed, epidural #leed, #rain contusion, seiAure or post3ictal weakness, #rainstem or spinal cord in:ury!

R)

Clearing C8s(ines8 The principle of clearin C3spines is to rule out #oth #ony fractures and li amentous in:ury, either of which can make the spine dan erously unsta#le! Most emer ency physicians will clear the cer$ical spine in the case of an alert patient who has no pain on palpation of the dorsal spinous processes and a normal cross3ta#le lateral C3spine <3ray! If the patient has neck pain, flexionBextension plane films are done! These in$ol$e ently flexin , then extendin the neck and takin $iews at each extreme! The patient must #e alert enou h to warn the examiner of paresthesias in the hands or increased neck pain on mo$ement durin this procedure, which may indicate compromise of the neural elements! QlexionBextension $iews may #e done under fluoroscopy if the patient is not alert! Treat.ent8 Continuous san uinous nasal dischar e after head in:ury is likely a leak of C'Q mixed with #lood! This represents a #reak of the menin es and can, rarely, lead to menin itis -M )* of cases.! &rophylactic anti#iotics not indicated as this selects for resistant or anisms! If menin itis results, it is usually due to less $irulent or anisms than in other settin s! J (4* of leaks resol$e spontaneously within R weeks! If leak does not su#side spontaneously, a sur ical repair may #e necessary! Consult neurosur ery! CT head and neck! May need M=I for delineation of #rainstem or spinal cord in:uries! Key Points8 1! "e$er do lum#ar puncture in head in:ury! H! 0ll patients with head in:ury ha$e C3spine in:ury until pro$en otherwise! 3! @on+t #lame coma on alcohol8 there may also #e a hematoma! R! Low ?& after head in:ury means in:ury elsewhere! )! Must clear c3spine #oth radiolo ically 0"@ clinically! "0$ Older .an 'ith D0 R1C/+P% on routine urinalysis$ Ta e a *ocused history$ B) 5hat is the li ely diagnosisC Give three other (ossible diagnoses$ 5hat t'o investigations 'ould you orderC +istory8 see question PHK! Li ely diagnosis8 #eni n prostatic hyperplasia! -i**erential diagnosis8 transitional cell carcinoma of #ladder, ETI, nephrolithiasis, hydronephrosis, prostatitis, prostate cancer, renal cell carcinoma, essential hematuria -tends to occur in children.! T'o investigations8 prostate specific anti en -&'0., cystoscopy, renal, #ladder and prostate ultrasound, intra$enous pyelo ram -ID&.! "!$ /0 year old *e.ale 'ith hy(ertension$ Per*or. a (hysical e7a.$ B) Give a di**erential diagnosis$ 5hat investigations 'ould you orderC 'ee question P11! "/$ 4# year old .ale co.(lains o* bladder distension& inability to urinate and dribbling o* urine *ro. the urethra$ Ta e a history$ B) 5hat investigations 'ould you orderC +istory8 patient I@! ,nset of symptoms, chronolo y, pre$ious episodes! 0ssociated constipation, perineal num#ness, le weakness, dia#etic neuropathy! Is patient on a new medicationF 'uprapu#ic pain, pain on urination, frank #lood in the urine, color of urine, difficulty initiatin or maintainin urinary stream, renal pain, roin pain! &re$ious renal colic or dia nosis of nephrolithiasisF Tnown prostatic hypertrophy or cancerF @ia#etesF ?1H deficiencyF =ecent sur eryF ;istory of hypercalcemia, hypertension! Mali nant symptoms8 ni ht sweats, wei ht loss, fati ue! Medications, dru sBalcohol, smokin , past medical history, past sur ical history, family history, re$iew of systems! Investigations8 urinalysis, urine microscopy and culture with sensiti$ities, cystoscopy, &'0, renal and pel$ic ultrasound!

!""/
"3$ 40 year old *e.ale *eeling de(ressed$ Co.(lains o* sto.ach (ain$ Per*or. *ocused .ental status e7a.$ Mental status8 appearance, #eha$ior -dress, roomin , posture, ait, apparent a e, physical health, #ody ha#itus, expressions, attitude 3 cooperati$eF, psychomotor acti$ity, attention, eye contact., speech -rate, rhythmBfluency, $olume, tone, quantity, spontaneity, articulation., mood -su#:ecti$e emotional state in patient+s own words., affect -Quality 9 euthymic, depressed,

RI

ele$ated, anxiousL =an e 9 full, restrictedL 'ta#ility 9 fixed, la#ileL 0ppropriatenessL Intensity 3 flat, #lunted., suicidal ideation -low, intermediate, hi h 9 poor correlation #etween clinical impression of suicide risk and pro#a#ility of attempt., thou ht process -coherent, fli ht of ideas, tan entiality, circumstantiality, thou ht #lockin , neolo isms, clan in , perse$eration, word salad, echolalia., thou ht content -delusions 9 #iAarre $s! non3#iAarre, o#sessions, preoccupations, pho#ias, recurrent themes., perceptual distur#ances -illusions, hallucinations, depersonaliAation, derealiAation., insi ht, co nition, :ud ment! ">$ !4 year old girl brought to the o**ice by a class.ate *or 'eight loss over the (ast si7 .onths$ The class.ate is 'orried about anore7ia nervosa$ Ta e a history and counsel$ +istory8 amount of wei ht lost, time frame! ;ow did the patient lose the wei htF %hat is the patient+s diet nowF 'till losin wei htF ;ow often does the patient wei h herselfF 0re you proud of this wei ht lossF @o you think you need to lose moreF 0re you afraid of #ecomin 5fat6F @o you admire women who are smaller than youF ?in e eatin , post3prandial $omitin , laxati$e or diuretic a#use, excessi$e exercise, diet pills! %earin #a y clothes to conceal 5fatness,6 una#le to look at self in a mirror or to #e touched #y others! 0sk a#out the home en$ironment, is there a pro#lem with expressin conflict openlyF 'i ns of malnutrition, amenorrhea -J 3 consecuti$e menstrual cycles missed., sallow skin, rash, easy #ruisin , dry and sparse hair, lassitude, weakness, anemia, neurolo ic findin s -carpal and tarsal ner$e compression, confusion, emotional la#ility, loss of corticospinal $i#ration and position sense., lossitis, heart #urn, teeth erosion, >I #leedin ! Counsel8 @etermine ideal #ody wei ht usin standard hei htBwei ht charts -?MI O wei ht -k .Bhei ht H -mH., ideal is a#out H43 H) for females.! 'how patient her position on the chart! Explain that anorexia ner$osa is a modern disease of hi hly moti$ated youn women! These women exercise extreme control o$er their #odies, often as a means of su#limatin their ina#ility to express conflict at home! %arn patient that excessi$e wei ht loss has led to the deaths of many youn women who were una#le to correct their anorexia! Explain that proper #ody wei ht is essential for health and mental function, includin learnin and performin well at schoolBcareer! /ou understand that the patient may #e proud of her wei ht loss! ?ein underwei ht may show a reat deal of self control and will power, #ut #ein at ideal wei ht shows more! In$ite patient to de$elop a healthy #ody ima e #y not equatin soft or fatty #ody areas with o$erwei ht! EmphasiAe that attracti$eness and ood health depend on a ood #alance of fatty tissues as well as lean! Contract with the patient to ain a certain num#er of pounds per week! @iscuss how she will do this! Contract for specific wei ht ain oals -H l#sBweek.! In$ol$e dietician! ;alt diuretics, laxati$es, diet pills! Close monitorin of wei ht, $itals, heart rhythm, potassium! 0rran e follow up with patient and her family to discuss family dynamics, expression of conflict in the home! "D$ / year old child 'ith history o* *ever and ! sei6ure$ Counsel (arents$ 'ee also question P(! Most li ely diagnosis8 #eni n fe#rile seiAure -fe#rile seiAures usually occur I months to I years, associated with initial rapid rise in temperature, no neurolo ic a#normalitiesBe$idence of C"' infectionBinflammation #efore or after, no history of non3 fe#rile seiAures, most commonly eneraliAed tonic3clonic, M 1) minutes duration, no recurrence in HR hours, atypical may show focal ori inBJ 1) minutesBJ 1BHR hoursBtransient neurolo ic defect.! Counsel (arents regarding *ebrile sei6ure8 0 typical fe#rile seiAure is a #rief eneraliAed tonic3clonic seiAure related to hi h fe$er -at least 3( de rees Celsius. and occurrin #etween the a es of 3 months and 2 years! The post3ictal sta e is characteriAed #y impro$ement in confusion, lethar y, limpness! The reatest risk factor for fe#rile seiAures is a history of fe#rile seiAures in the parents! This is the most common seiAure in children -33)* of children, M J Q.! ,ccur #etween the a es of I months and I years! Thou ht to #e due to initial rapid rise in temperature! These seiAures may come a#out as a result of fe$er from any cause, includin post immuniAation! In the a#sence of an a#normal de$elopmental history -C&, de$elopmental delay., and an otherwise well child, they are usually #eni n! 'eiAures do not cause mental impairment unless they are prolon ed -J 34 min. #ut can #e a symptom of #rain dama e! Prognosis a*ter a single *ebrile sei6ure8 I)* will ne$er ha$e another seiAure, 34* will ha$e further fe#rile seiAures, 3* will o on to ha$e seiAures without fe$er, and H* will de$elop lifelon epilepsy -risk factors for this are8 de$elopmental andBor neurolo ical a#normalities of child prior to seiAures, family history of non3fe#rile seiAures and an atypical initial seiAure.! Treat.ent o* recurrence8 control fe$er with antipyretics -Tylenol., tepid #ath, fluids for comfort only and use 0ti$an -loraAepam. 1 m 'LB&, -or diaAepam )314 m &=. if a seiAure occurs at home! Turn patient onto hisBher side, do not force o#:ects or fin ers into mouth! ?rin to E= if seiAure does not stop within 14 minutes! 'eiAures do not cause mental

R2

impairment unless they are prolon ed -J 34 min., althou h seiAures can #e a symptom of #rain dama e! &atient should #e in$esti ated with CT head and EE>! &rophylactic anticon$ulsant therapy is a consideration with repeated seiAures! "4$ O(erating roo. nurse sustained needle stic in9ury$ 5orried about he(atitis and ,I-S$ Counsel$ +istory8 "ame, a e, occupation! @etermine se$erity of exposure8 hollow #ore needleF "eedle au eF @epth of penetrationF @id needle contain #lood from a patientF %as any #lood in:ectedF Is the ;ID and hepatitis status of the patient knownF Is the nurse immuniAed a ainst ;ep ?F Patient no'n to have) ;ID ;ep ? 0nti en ;ep ? 0nti#ody ;ep C Odds o* Trans.ission 4!3* R4* 14* )*

Counsel8 ;ID has a hi h mortality rate within ) years of testin positi$e, and is e$entually fatal in most cases, howe$er patients with ;ID ha$e a much lon er life expectancy than in the past due to impro$ed anti3retro$iral therapy! ;epatitis ? causes fulminant hepatic necrosis in 1* of those infected, which is fatal in I4* of cases! )* of those infected with ;ep ? remain in a chronic carrier state, which is associated with a H)3R4* risk of cirrhosis and H3)* per year risk of hepatocellular cancer! There is a J )4* risk of chronic li$er disease once infected with ;ep C, the risks of cirrhosis and hepatocellular cancer are similar to those for ;ep ?! The o$erall risk of transmission is as descri#ed a#o$e! =ecommend #aseline testin for ;ID, ;ep ?, and C in nurse and patient -require consent for ;ID testin .! If the patient was recently infected, heBshe may not #ecome positi$e on anti enic testin for up to 3 months! Therefore nurse and patient should #e retested! =ecommend ;ID prophylaxis -0ST C 3TC x R weeks and consult Infectious @isease specialist. for si nificant needle stick from a patient with known ;ID or who is at hi h risk -multiple partners, ID dru use, anal intercourse, recent immi rant from endemic area.! If the nurse is not effecti$ely immuniAed -i!e! anti#ody titers tested. a ainst ;epatitis ?, recommend immuniAation! If patient is found to #e ;ep ? or C positi$e, i$e the nurse passi$e immunity amma lo#ulin, ;ep ? amma lo#ulin -;?I>. within 2 days of exposure, has #een pro$en to #e effecti$e in pre$entin transmission! "#$ -e.onstrate and counsel a (atient on breast sel*8e7a.ination and .a..ogra(hy screening$ 1reast e7a.8 'ittin 8 Inspect in four separate positions8 1. sittin with arms at her side, H. sittin with #oth arms raised a#o$e her head, 3. sittin with hands pushin on hips and el#ows out, R. patient leanin forward8 inspect for siAe and symmetry, $isi#le massesBcontour chan es, skin retraction, erythema, dimplin , nipple retractionBin$ersionBulcerationBsiAe 7 shape, peau d+oran e around nipple and elsewhere! &alpation of axillary, infracla$icular, supracla$icular nodes! 'upine -with pillow under shoulder., each #reast examined separately, drape other #reast, small circular motions co$erin an area of approximately 1 square inch di$ide into li ht, medium and deep palpation and perform in all four quadrants! Can denote position of lumps #y clock position with cm distance from nipple! "ipple squeeAe to try to exude any fluid from the nipple -ask patient to squeeAe nipple herself.! %atch for dimplin , #loody nipple dischar e and inflammation! Ma..ogra(hy8 /early mammo raphy screenin of pro$en #enefit from a e )4! ?enefit as a screenin test equi$ocal from a e R4 in the eneral population #ut is recommended if there is a positi$e family history of #reast cancer! ?reast cancer in two first de ree relati$es -parents, si#lin s, children. is an indicator for yearly mammo raphy startin at )314 years #efore youn est family mem#er+s presentation! "?$ 40 year old .ale& di**iculty 'al ing$ Per*or. a neurological e7a.$ 'ee neurolo ical exam in question P)! ""$ 3# year old 'ell G!P0 *e.ale& " 'ee s (regnant$ Manage$ +istory8 &atient I@8 &lanned pre nancyF 'tatus of any relationships at present includin relationship with the child+s father! 'ocial supports -family, friends, #oyfriend., do they knowF 0re they helpin F EmploymentBfinancialBeducational status of the patient, does the patient feel prepared to raise a childF &ro$isions for care of child when #ornF >T&0L -num#er of estations, term pre nancies, premature #irths, a#ortions, li$e children., history of pro#lems, if any, with pre$ious pre nancies! Current pre nancy, esta#lish estational a e ->0. #y last menstrual period -LM&. if re ular periods and sure dates -if unsure a datin ultrasound would #e needed.! The >0 is the num#er of weeks from the first day of the LM&! The E@C is first day of LM& C 2 days 9 3 months! 'mokin -prepared to quitF., alcohol -no alcohol durin pre nancy., illicit dru s, diet, exercise, medications -a$oid durin pre nancy8 includin o$er the counter.! @ia#etes, family history of inherited disorders, heart

RK

disease, circulatory pro#lems, renal disease, hypertension! Menstrual history, re ularity of cycles, how lon has patient not used contraception! 0ny mornin sickness, $a inal #leedin F &ast medical and sur ical history, medications, dru sBalcohol, smokin , aller ies, family history, re$iew of systems! Psychiatric8 Co$er mnemonic for ma:or depression! M'I>EC0&'8 mood -depressed., sleep -increased or decreasedGif decreased, often early mornin awakenin ., interest -decreased., uiltBworthlessness, ener y -decreased or fati ued., concentrationBdifficulty makin decisions, appetite andBor wei ht increase or decrease, psychomotor acti$ity -increased or decreased., suicidal ideation 9 positi$e dia nosis of ma:or depression requires fi$e of these o$er a H week period, one of the fi$e must #e loss of interest or depressed mood! 'ymptoms do not meet criteria for mixed episode, si nificant socialBoccupational impairment, exclude su#stance or >MC, not #erea$ement! Physical8 $itals, wei ht, hei ht, palpation of neck and thyroid land, fundoscopic exam, check lid la , reflexes, cardiopulmonary exam, #reast exam, a#dominal exam! &alpate uterus, measure symphysis3um#ilicus distance! @oppler for fetal heart -may not detect until 14 weeks.! Da inal #imanual and speculum exam -cer$ix should #e closed.! &ap smear -if none in last I months, use speculum, not #rush in os., swa# cer$ix for cultures ->C, chlamydia.! Investigations8 C?C, lytes, I"=B&TT, urea, creatinine, urinalysis, EC>! ?lood roup and type, =h anti#odies, D@=L and ;#s0 routine, ru#ella titer, ;ID serolo y offered, serum folate, urine dip, microscopy and culture, T? skin test in patients from an endemic area, enetic testin as indicated on history or for sickle cell in #lacks! Triple screen -M''. M'0Q&, XhC>, uE3 Trisomy 1K, Trisomy H1, "T@ -at 1I weeks.! 0mniocentesis at 1)31I weeks for alpha3fetoprotein and acetyl cholinesterase! Chorionic $illus samplin -1431H weeks. should #e offered i$en the patient+s a e! Qetal ultrasound! Counseling8 @iscuss risk of @own+s syndrome due to maternal a e, $alue of fetal enetic testin ! =ecommend daily pre nancy $itamin preparation, milk and healthy diet! @o not increase food intake dramatically 9 excessi$e wei ht ain not recommended, H33 l#s per month for a total of H)334 l#s ain in wei ht ideal! @o not diet durin pre nancy! Continue normal acti$ities and customary exercise! "o alcohol, no smokin , no medications of any kind unless discussed with M@! Control mornin sickness with small meals and #land foods! Lyin on side decreases swellin and discomfort! ;emorrhoids, #ackache, heart#urn, increased $a inal dischar e are common! Qollow3up e$ery R weeks until 3H weeks, then increase to e$ery H weeks! Call if any concerns or trou#lin symptoms, especially a#dominal pain, $a inal #leedin , persistent headache, illness or infection! !00$ 40 year old *e.ale 'ith bloody vaginal discharge$ Ta e a history$ +istory8 "ame, a e, occupation! Think a#out8 #lood dyscrasias, thyroid dysfunction, mali nancy, &C,', endometriosis, &I@, fi#roids, unopposed estro en, or polyps! ,nset of #leedin , frequency, estimate quantity -num#er of pads., color, consistency of dischar e, associated pain, $a inal discomfort, crampin ! &re$ious episodes, history of fi#roids, polyps, &I@, &C,'! &ost coital and rectal #leedin ! %ei ht loss, ni ht sweats, fati ue! ;istory of easy #ruisin B#leedin , inherited #lood coa ulation disorders! 0 e of menarche, a e of menopause, a e of first sexual acti$ity! Ese of hormonal replacement therapy, which preparationF ;istory of fi#roids, reproducti$e tract cancers, last &ap smear! &re nancy history! Medications, dru sBalcohol, smokin , past medical history, sur ical history, family history, re$iew of systems! !0!$ 30 year old .an 'ith he.ate.esis and abdo.inal (ain in the e.ergency de(art.ent$ 1P ?0/D0& tachycardia$ Manage$ =esuscitate as in question PI! Consult astroenterolo y for immediate endoscopy! !0/$ Pregnant 'o.an& 34 'ee s gestation& has (roteinuria and 1P !D0/?D :(re8gestational 1P !!0/4D;$ Manage$ 'ee question PRI! !03$ , .other is 'orried that her ! year old loo s (ale$ Ta e a history$ %inding) breast *ed *or the *irst / .onths& then /O .il $ B) 5hat is the .ost li ely diagnosisC 5hat investigations 'ould you orderC +istory8 "ame, a e! Qeeds and feedin history -esp! fruit :uice, excess milk.! >rowth pattern8 wei ht lossF @iarrheaF -consistency, color, quantity and frequency., #lood in stool, melena stools, concurrent illness, $omitin , fe$er, anorexia, difficulty #reathin , lassitude, dry mouthBeyes, low urine output, illness affectin other children in the family or adults! =ecent immuniAation, tra$el, anti#iotics! Medications, past medical history, aller ies, #irth history, pre nancy pro#lems, maternal illness durin pre nancy, family history, re$iew of systems!

R(

Most li ely diagnosis8 Iron deficiency anemia -most common cause of childhood anemia.! Typically in #ottle3fed infants -I3 HR months. recei$in lar e $olumes of cow+s milk should add iron3fortified cereal and iron rich foods startin at I months! Investigations8 C?C with peripheral smear, lytes, urea, creatinine, I"=B&TT, serum ferritin, al#umin! !0>$ 2lderly .an 'ith creatinine !000$ Ta e a history$ B) Give a di**erential diagnosis$ 5hat investigations 'ould you orderC +istory8 &atient I@! suprapu#ic pain, pain on urination, frequency, ur ency, frank #lood in the urine - lo#ular clots from #ladder or strin shaped clots from ureters., color of urine, difficulty initiatin or maintainin urinary stream, renal pain, roin pain! &ro$okin factors! 0ssociated symptoms includin saddle anesthesia, loss of #owel control! ;istory of recent ETI, 'T@s, T? exposure, pel$ic irradiation, #leedin diathesis, smokin ! Qe$er, chills, nausea, fati ue! &re$ious renal colicBdia nosed nephrolithiasisF ;istory of hypercalcemia, hypertension! Mali nant symptoms8 ni ht sweats, wei ht loss, fati ue! Medications, dru s -"'0I@s, anticoa ulants.Balcohol, smokin , past medical history, past sur ical history, family history -polycystic kidney diseaseF., re$iew of systems! -i**erential diagnosis8 1! Pre8renal8 ;ypo$olemia, poor cardiac output, reno$ascular disease, "'0I@B0CEi use, li$er failure! H! Renal8 Dascular mali nant ;T", cholesterol em#oli, ;E'BTT&L Tu#ulo3interstial 0T" -ischemicBtoxin endo enousBexo enous., 0I"L >lomerular -M )*.! Causes8 <3ray contrast, myo lo#inuria, acute lomerulonephritis, @IC, pyelonephritis, intrarenal precipitation in hypercalcemia, myeloma! 3! Post8renal8 ,#struction8 upper -clot, tumor, stone, external compression., lower -?&;, clot, stone, stricture, autonomic dysfunction.! Investigations8 C?C, lytes, urea, creatinine, phosphate, ioniAed CaCC, ma nesium, I"=B&TT, 0'T, 0LT, 0L&, >>T, prostate specific anti en, CT3M?, troponin, 0?>! Erinalysis8 microscopy, dip, culture and sensiti$ity! 0#dominal x3ray, a#dominal pel$ic ultrasound! &ost3$oid catheteriAation! -0$oid ID& due to dye.! !0D$ ! year old boy 'ith 4 .onths diarrhea$ Ta e a history$ B) Give a di**erential diagnosis$ 'ee question P)4! !04$ D? year old lady in hos(ital > days (ost8o( hysterecto.y *or *ibroids$ ,gitated& had tactile hallucinations the (revious night$ Ta e a history$ %inding) history o* alcoholis.$ B) 5hat is the .ost li ely diagnosisC +istory8 onset of hallucinations, duration, description! Tactile hallucinations or #u s crawlin on skin or on ceilin su est alcohol withdrawal! 0ssociated fe$er, a itation, sweatin , tremor, decreased le$el of consciousness, seiAureF 0ny pro#lems with sur ical reco$ery, wound healin , mo#iliAationF 0mount of alcohol consumed at home! ;istory of alcoholism, le swellin , ',?, chest pain! Current state! &ost3op medications -morphine, @emerol. 9 pre$ious #ad reactions to these or to anti#ioticsF &re$ious episode like this oneF &ast medical history, medications, dru and alcohol use, smokin , aller y, family history, re$iew of systems! Most li ely diagnosis8 alcohol withdrawal! !0#$ Houng .an 'ith a s'ollen cervical ly.(h node$ Per*or. a *ocused (hysical e7a.$ B) CFR sho's .ediastinal 'idening 'ith (erihilar nodes$ -escribe$ Give *ive *eatures on history 'hich 'ould be hel(*ul *or diagnosis$ Physical e7a.8 $itals, :aundice, nutritional status, #uccal mucosa, teeth, #reath -hepatic fetor., parotid hypertrophy, lossitis, inspect chest for telan ectasia, ynecomastia, loss of axillary hair! ;ands8 palmar erythema, clu##in , @upuytren+s contracture, wastin of hand intrinsics! &alpate for lymph nodes in the neck, supra and infra3cla$icular, axillae, roin! Examine the oral ca$ity and pharynx! Check for rashes! ,bdo.inal e7a. :su(ine;8 see question PH(! -i**erential diagnosis8 lymphoma, leukemia, $iral infection -mononucleosis, ;ID, E?D., inflammatory autoimmune disease -sarcoidosis, lupus., serum sickness -se$ere aller ic reaction short of anaphylaxis., T?, li$er disease with portal hypertension! %ive *eatures on history hel(*ul *or diagnosis8 $iral prodrome, family history of sarcoid, lymph nodes painful, #one pain, pruritis, wei ht loss!

)4

!0?$ %e.ale (atient *ound to have a nodule on routine CFR$ Per*or. a *ocused (hysical e7a.$ B) Give a di**erential diagnosis$ 5hat investigations 'ould you orderC Cardio(ul.onary e7a. as in question P13 and PHR! 'ee also question PR(! Investigations8 old C<= for comparison -if lesion is old and unchan in , inter$entions are less a ressi$e, calcification is also associated with #eni n lesions such as old ranulomas., CT chest with CT uided needle #iopsy, sputum for cytolo y and acid3 fast stainin -T?., T? skin test, #ronchoscopy with #iopsy and washin s if lesion seen, open #iopsy or lo#ectomy! ,lgorith.8 solitary nodule pre$ious C<= #eni n or unchan ed -repeat in q33Imonths for H years if unchan ed o#ser$e, if chan ed at any time continueG., mali nant or chan ed CT thorax8 cancer -sta e and treat., calcification -o#ser$e., no dia nosis #ronchoscopy or transthoracic needle aspiration still no dia nosis -resect for dia nosis., inflammatory -treat cause., cancer -sta e and treat.! !0"$ 40 year old .ale sli((ed and *ell 4 days ago$ Co.es to you because o* he.o(tysis$ Per*or. a *ocused (hysical e7a.$ %inding) (ositive +o.ans sign$ B) 5hat is the .ost li ely diagnosisC Give a (lan *or. .anage.ent$ Cardio(ul.onary e7a. as in question P13, plus additional attention to calf siAe, tenderness, redness and pleuritic chest pain! +o.ans sign8 pain in the calf on dorsiflexion of the foot 9 indicates throm#ophle#itis! Check that trachea is midline! Is the patient on @DT prophylaxis or anti3coa ulationF Most li ely diagnosis8 pulmonary em#olus! S(eci*ic investigations *or P28 CT chest -only shows clinically si nificant &E., DBQ scan -conclusi$e when is shows hi h or low pro#a#ility., pulmonary an io ram - old standard #ut in$asi$e., EC;,, and serial -qHd. le @opplers for presence of @DT a#o$e the knee! Others8 C<= -often normal, ;ampton+s hump, %estermark+s si n, rarely dilatation of proximal &0., EC> -sinus tachycardia, '1Q3T3., 0?> -&a,H usually decreased, &aC,H decreased due to increase in o$erall minute $entilation, increased 03a radient., @3dimer! 'ee also question P12! Treat.ent8 if suspicion of &E is hi h, anticoa ulate #efore waitin for these tests with heparin 2)44 E ID #olus -K4 EBk ., then infuse at 1H44 EBhr -1K EBk Bh.! Measure &TT qIh, ad:ust dose for &TT 243(4s -H!)33 times normal #aseline.! 'tart coumadin, to I"= H33, continue coumadin for 3 months! !!0$ Tele(hone in roo.$ Mother calls because her child has 9ust ingested a caustic cleaner$ Manage over the (hone$ B) 5hat do you do a*ter hanging u( the tele(honeC Give a (lan *or .anage.ent$ 'ee question P)H! !!!$ Tele(hone in the roo.$ Physician in a (eri(heral center calls 'ishing to trans*er an unstable (atient 'ho has been in a .otor vehicle accident$ Manage over the (hone$ B) CFR sho's o(aci*ication o* the right lung$ 5hat is your diagnosisC Give the i..ediate .anage.ent o* this (roble.$ Over the tele(hone8 &hysician+s name, name of center, patient+s name! In:uries, in$esti ations done, $itals, la# $alues! >C', is patient intu#atedF &eripheral physician must not transfer patient until he is sta#iliAed, i!e! ood ?&, ood oxy en sats, #leedin controlled, #lood products i$en as needed! Estimated time of arri$alF &hysician accompaniedF CFR8 likely hemothorax! Treat.ent8 Chest tu#e in E=, drain hematoma and connect to suction throu h a #u##le cham#er! Consult thoracic sur ery, prepare patient in case of immediate ,=! !!/$ Houng 'o.an 'ith bilateral .igratory arthritis o* recent onset$ Ta e a history$ B) Give a di**erential diagnosis$ 5hat investigations 'ould you orderC Note8 Mi ratory arthritis su ests onococcal infection!

)1

+istory8 &atient I@! ,nset of arthritic symptoms, durations, :oints affected, chronolo y! 0ssociated fe$er, malaise, fati ue, rash, a#dominal pain and cramps, $a inal dischar e, pain with urination, dyspareunia -painful intercourse.! ;istory of arthritis -rheumatoid, osteoarthritis., psoriasis, Lyme disease -campin trips., =eiter+s syndrome, ankylosin spondylitis, sexually transmitted diseases includin &I@! 'exual history8 present partners, num#er of partners, fidelity of partner-s., use of condoms! Medications, dru sBalcohol, smokin , aller ies, past medical history, family history, re$iew of systems! -i**erential diagnosis8 onococcal arthritis, psoriatic arthritis, Lyme disease, =eiter+s syndrome, ankylosin spondylitis, rheumatoid arthritis, osteoarthritis, out! Investigations8 C?C, E'=, lytes, urea, creatinine, I"=B&TT, #lood cultures! Cer$ical swa# for culture and sensiti$ity! Noint aspirate for microscopy and culture!

!"""
!!3$ >? year old 'o.an 'ith .ulti(le co.(laints$ +as had negative investigations by several other doctors$ Ta e a history and (er*or. a .ental status e7a.ination$ B) 5ithout loo ing at the (atient again& describe her a((earance$ 5hat is your diagnosisC !!>$ 3/ year old .other (resents to your o**ice because her > year old son has had an allergic reaction to (eanuts$ +e 'as brought to the e.ergency de(arte.ent and treated yesterday$ +e is no' 'ell$ Counsel$ !!D$ 30 year old *e.ale 'ith lo'er abdo.inal (ain *or ! 'ee $ Per*or. a (hysical e7a.$ %indings include distended abdo.en and tenderness at Mc1urneys$ B) The (atient has a history o* Crohns and (resents 'ith the *ollo'ing abdo.inal 78ray :sho's s.all bo'el obstruction;& 'hat is the diagnosisC !!4$ , / day old in*ant 'ith 9aundice and a seru. bilirubin o* //0 I.ol/L :re*$ Ma7 /00 I.ol/L;$ Ta e a history *ro. the .other$ B) 5hat are the (ossible causes *or this abnor.alityC Give investigations and counsel$ 'ee Question PH)! !!#$ 3D year old .ale out(atient 'ith shortness o* breath& cough& s(utu.$ Ta e a history and (er*or. a (hysical e7a.$ F8ray *indings) PCP$ B) Reco..end *urther investigations and treat.ent$ !!?$ 44 year old .ale 'ith sy.(to.s o* claudication$ -o a *ocused (hysical e7a.$ B) 5hat / investigations 'ould be .ost a((ro(riate$ 5hat D ris *actors on history 'ould (oint to the diagnosis$ !!"$ Mother 'ants to go on a ca.(ing tri( 'ith her son 'ho su**ers *ro. enuresis$ Counsel$ !/0$ Houng *e.ale 'ith secondary a.enorrhea *or 4 .onths$ Ta e a history$ B) 5hat investigations 'ould you order$ Give a di**erential diagnosis$ 5hat is the .ost li ely diagnosis& 'hat results 'ould con*ir. this diagnosisC Counsel 'ith regards to OCP$ 'ee question PKI! !/!$ Tele(hone in the roo.$ Nurse in a (eri(heral center calls 'ishing to trans*er a (atient 'ho has had a *ebrile sei6ure$ Manage over the (hone$ Counsel to get (atient ready *or trans*er$ !//$ /D year old .ale has had a sca**old *all on hi.$ +e (resents 'ith chest (ain and 'ould li e so.ething *or the (ain$ Nor.al res(iratory e7a. and nor.al cardiac e7a.$ Manage$ !/3$ 30 year old G!P0 'ith (re8ecla.(sia$ Counsel$ 'ee question PRI! !/>$ >0 year old .ale @9ust (assing throughA 'ants Tylenol P3 to hold hi. until ne7t 'ee $ Manage$ 'ee question P1I!

/00> 8 October
)H

!/D$ 3" year old 'o.an 'ho 'ould li e to Euit s.o ing$ Counsel$ +istory8 "ame, a e, occupation! 'mokin ha#its8 amount, duration, frequency, time of day! >ain from smokin -e! ! wei ht loss, decreased anxiety, social relationships.! &ersonal concerns a#out smokin and quittin , foreseen #enefits from quittin , interest in quittin -a person will only quit if they are willin .! &re$ious attempts and results, medical situation8 ',?, cou h, asthma, C,&@, ;T"! 'ocial situation8 other smokers in familyBsocial network! "icotine dependence8 preoccupation or compulsion to use, impairment or loss of control o$er use, continued use despite ne ati$e consequences, minimiAation or denial of pro#lems associated with use! &ast medical history, aller ies, medications, alcoholBdru use, family history, re$iew of systems! Counseling8 H important components that need to #e addressed8 1. physicalBchemical addiction8 symptoms of withdrawal -tremorsBirrita#ility. and H. ha#itualBen$ironmental factors8 psycholo ical, social, and spiritual components! 0d$ise patient of health risks8 'mokin is the sin le most pre$enta#le cause of death responsi#le for K4* of lun cancers, C,&@, cardio$ascular disease, also a factor in &E@, low #irth wei ht #a#ies, premature a in , upper >IBrespiratory cancers, respiratory infections, 'I@'! 0 es H)33R ha$e hi hest pre$alence of smokin , 1)* of smokers smoke J H) ci arettesBday! 0fter assessin smokin ha#its ad$ise e$ery smoker to quit at e$ery $isit assess sta e of chan e -see ta#le #elow.! Moti$ate smoker to attempt to quit8 #enefits include decreased respiratory infections, increased exercise toleranceBener y, increased tasteBsmell, ask for a commitment to quit -set a date., assist the smoker to quit -physician counselin .! ?e in with self3help materials8 remo$e ashtraysBli hters, increase hi h fi#er snacksB um, increase aero#ic exercise, self3 reward, may also use nicotine patchB um or attend smokin withdrawal pro rams! =eward oals that are met8 plan for new social relationships and acti$ities to make it easier to make a serious attempt to chan e #eha$ior! Qollow3up8 set firm dates! 0nticipate pro#lems8 wei ht ain, withdrawal symptoms! Continue to monitorBsupport! @o not i$e up if failed! Most relapses occur in first yearL most people try J ) times #efore quittin ! Stages o* Change Model 1arriers Moti$ational drift Low social support Lack of percei$ed self3efficacy 0ttitudes and emotions Lack of knowled e @enialBtri$ialiAation &ercei$ed in$ulnera#ility Qaulty conceptions Processes =einforcement mana ement ;elpin relationship =einforcement 'elf e$aluation Consciousness raisin

Maintenance ,ction Conte.(lation Preconte.(lation

Treat.ent8 Nicotine (atch continuous self3re ulated amount of nicotine, decreases cra$in andBor withdrawal, will not replace immediate effects of smokin , ha#it or pleasure! Indications8 nicotine dependent, hi h moti$ation to quit smokin ! Contraindications8 smokin while on patch, aller y, MI, CD0! =elati$e contraindication8 pre nancy! @uration of treatment8 R31H weeks usually adequate! =yban :bu(ro(ion; appro$ed in Canada in 0u ust 1((K acts on dopaminer ic -reward. and noradrener ic -withdrawal. pathways! Contraindications8 seiAure disorder, alcoholism, eatin disorder, recent M0,I use, current pre nancyL caution if usin ''=I -reduction of seiAure threshold.! @ose $aries with amount the patient smokes! &atient continues to smoke for first week of treatment and then completely stops -therapeutic le$els reached in one week.! =ecommend a#stinence from alcohol due to risk of toxic le$els with li$er dysfunction! 'ide effects8 headache, insomnia, dry mouth, wei ht ain! !/4$ 4D year old 'o.an (ost hysterecto.y having hallucinations at night *or the last t'o nights$ Ta e a history$ B) Most li ely diagnosisC I* this 'o.an 'ants to go ho.e *ro. hos(tial and is .edically cleared is she co.(etent to .a e that decisionC 5ould you allo' her to go ho.eC +istory8 onset of hallucinations, duration, description! 0ssociated fe$er, a itation, sweatin , tremor, decreased le$el of consciousness, seiAureF Misperceptions and illusions, impaired attention span, disorientation, impaired le$el of consciousness, delusional thinkin , affecti$e symptoms -mood 9 sad.! Qluctuatin courseF 0ny pro#lems with sur ical reco$ery, wound healin , mo#iliAationF 0mount of alcohol consumed at home! ;istory of alcoholism, le swellin , ',?, chest pain! Current

)3

state! &ost3op medications -morphine, @emerol. 9 pre$ious #ad reactions to these or to anti#ioticsF &re$ious episode like this oneF &ast medical history, medications, dru and alcohol use, smokin , aller y, family history, re$iew of systems! Most li ely diagnosis8 delirium! -i**erential -iagnosis8 I %0TC; @E0T;8 I O infectious -encephalitis, menin itis, ETI, pneumonia., % 9 withdrawal -alcohol, #ar#iturates, #enAodiaAepines., 0 9 acute meta#olic disorder -lytes, hepaticBrenal failure., T 9 trauma -head in:ury, postop., C 9 C"' patholo y -stroke, hemorrha e, tumor, seiAure, &arkinson+s., ; 9 hypoxia -anemia, cardiac failure, pulmonary em#olus., @ 9 deficiencies -$it! ?1H, folic acid, thiamine., E 9 endocrinopathies -thyroid, lucose, parathyroid, adrenal., 0 9 acute $ascular -shock, $asculitis, hypertensi$e encephalopathy., T 3 toxins, su#stance a#use, medication -alcohol, anesthetics, anticholiner ics, narcotics., ; 9 ;ea$y metals -arsenic, lead, mercury.! Investigations8 C?C, lytes, calcium, phosphate, ma nesium, lucose, E'=, li$erBrenal tests, urinalysis, EC>! 0s indicated #y history8 T';, CT head, toxicolo y screen, D=@L, L&, LE preparation, ?1H and folic acid le$els, EE>! Manage.ent8 Treat underlyin cause ->MC etc!.! 'top all non3essential medications! Maintain nutrition, hydration, electrolyte #alance and monitor $itals! &sychosocial quietBwell lit en$ironment, room close to o#ser$ation, family mem#er for reassurance! &harmacolo ical haloperidol H3) m IM, loraAepam 1 m 'L, physical restraints if patient $iolent! !/#$ 30 year old .an 'ith abdo.inal (ain in the e.ergency de(art.ent$ %indings) -ia(horetic$ 1P ?0/D0& tachycardia$ Manage$ !/?$ 3/ year old .other (resents to your o**ice because her > year old son has had an allergic reaction to (eanuts$ +e 'as brought to the e.ergency de(arte.ent and treated yesterday$ +e is no' 'ell$ Counsel$ !/"$ 3D year old 'o.an *eels de(ressed$ %indings) Recently lost her *ather$ Ta e a *ocused history including a .ental status e7a.$ !30$ 3D year old .ale 'ith bac (ain and sti**ness$ Ta e history and (er*or. a *ocused (hysical e7a.$ %indings) !0 c. se(aration bet'een lu.bar s(ines 'hile erect increases by less than D c. 'hen bac is *le7ed *or'ard :(ositive 5right8 Schober test;& lateral *le7ion i.(aired$ B) Give the diagnosis and t'o associated conditions$ !3!$ #" year old *e.ale colla(ses in the .all$ Patient is dro'sy& unres(onsive to verbal sti.uli$ She is there 'ith her grand8daughter$ %indings) +R >0& 1P ?0/>0& 2CG co.(lete heart bloc $ Manage$ !3/$ , young .an sustains a head in9ury on *alling *ro. his bicycle$ Patient has been he.odyna.ically stabili6ed$ Per*or. a *ocused neurological e7a.$ B) Lateral s ull and lateral C8s(ine F8rays (rovided$ ,re they adeEuateC ,re they nor.alC The (atient has continuing sanguinous discharge *ro. his nose$ 5hat is the li ely cause o* this$ Treat.entC !33$ 30 year old 'o.an 'ith vaginal bleeding at 30 'ee s gestation$ Ta e a history$ B) Give a di**erential diagnosis$ Order investigations$ !3>$ 4D year old .ale out(atient 'ith shortness o* breath& cough& s(utu.$ Ta e a history and (er*or. a (hysical e7a.$ %indings) Lobar consolidation 'ith yello'8green s(utu.$ B) Given a diagnosis o* (neu.onia& reco..end treat.ent$ !3D$ Mother 'ith 4 .onth old child 'ith *ailure to thrive$ Ta e a history$ B) Give a di**erential diagnosis$ Reco..end treat.ent$ !34$ ,bcess station$ Per*or. an irrigation and drainage$

/00> 8 May
!3#$ ## year old *e.ale ad.itted *or resection o* colon cancer 'hich (resented 'ith (ainless bleeding (er rectu. has decided to 'ithdra' her consent *or surgery$ She has been seen by (sychiatry and dee.ed co.(etent& you are covering *or her *a.ily doctor& counsel$ !3?$ 3# year old *e.ale " 'ee s (regnant is 'orried about genetic (roble.s .ore *reEuent in 'o.en beco.ing (regnant later in li*e and is as ing about genetic testing$ Counsel$

)R

!3"$ 30 year old .ale has had ! 'ee o* 'orsening hi( (ain 'hich has (rogressed to *ailure to 'eight bear$ <itals) 1P !30/?0& P ?D& T 3"$D$ Conduct a *ocused history and (hysical$ B) 5hat is at the to( o* your di**erential$ 5hat single investigation 'ould hel( con*ir. your sus(icion$ !>0$ >D year old *e.ale (resents to your o**ice 'ith di**iculty slee(ing$ Ta e a *ocused history$ B) 5hat is the li ely diagnosis and 'hat is the single .ost a((ro(riate out(atient treat.ent$ !>!$ Mother o* 3L year old (resents because her child see.s to have delayed s(eech develo(.ent co.(ared to an older brother$ She is 'orried that her child .ay be @retardedA$ Ta e a history and counsel$ !>/$ D4 year old .ale (atient on 5ar*arin *or atrial *ibrillation (resents to your o**ice 'ith 3 day history o* coughing u( blood$ Ta e a history and (er*or. a *ocused (hysical e7a.ination$ B) This (atient is unha((y 'ith his care and 'ould li e an original co(y o* his chart so he .ay go to another (hysician$ 5hat do you do$ !>3$ /D year old .ale has *allen >0 *eet :!/ .eters; o** his (arents roo* 'hile hel(ing 'ith re(airs$ +e (resents 'ith shortness o* breath& tachy(nea& and vitals as *ollo's) 1P ?0/>0 P !30 RR /0$ Manage$ !>>$ D4 year old .ale (resents to the 2R 'ith chest (ain 'hich see.s to have resolved (artially 'ith Nitro s(ray ad.inistered en route by 2MS$ +e has a @nor.alA 2CG :'hich .ay sho' so.e .ild changes in the in*erior leads;$ Manage$ Second 2CG ta en a*ter another bout o* chest (ain sho's clear ST elevation in 3 consecutive in*erior leads :II& III& a<%;$ B) ,*ter receiving treat.ent but be*ore Cardiolgy has seen this (atient he 'ants to discharge hi.sel* des(ite the ris s o* doing so$ 5hat do you doC !>D$ Mother o* /L year old has brought her son bac to see you 'hen her sons cough has *ailed to resolve a*ter > 'ee s$ +e has received antibiotic treat.ent :,.o7icillin; and has been using a cough su((ressant 'ithout i.(rove.ent$ %indings) Cough is non (roductive but loose$ +istory o* ec6e.a8li e rash in child and .other$ %ather s.o es$ B) Single .ost a((ro(riate diagnosis$ 5hat in*or.ation on history leads you to this conclusion$ !>4$ /3 year old *e.ale 'ith /> hours o* (rogressive abdo.inal (ain$ -o a *ocused (hysical e7a.$ %indings) Patient in *etal (osition$ LLB (ain$ Guarding& rebound$ B) -i**erential diagnosis$ Long ter. conseEuences o* the .ost li ely diagnosis$ !>#$ 44 year old .ale 'ith sy.(to.s o* claudication$ -o a *ocused (hysical e7a.$ B) 5hat / investigations 'ould be .ost a((ro(riate$ 5hat D ris *actors on history 'ould (oint to the diagnosis$ !>?$ /! year old .ale brought into the e.ergency de(art.ent by a *riend due to nec sti**ness$ Ta e a *ocused history$ %indings) Patient is on haldol$ B) 5hat is the .ost li ely cause o* this (atients @nec sti**nessA$ Outline a .anage.ent (lan *or this (atient$ The (atient is treated and discharged but / hours later his .other calls because he 'as out on the balcony trying to *ly& 'hat i* any are your (ro*essional res(onsibilities to this (atient$ !>"$ 33 year old .ale 'ith elevated L%Ts on routine blood 'or 'hile a((lying *or li*e insurance$ The (atient is u(set about these results and 'ould li e you to @*i7 itA$ Labs) ,ST !!0 ,LT !#0 ,LP !00 1ili /4$ %indings) (atient is consu.ing a((ro7i.ately /0 alcoholic beverages (er 'ee $ %ather died at D4 years old o* cirrhosis$ B) -i**erential diagnosis :to( three;$ Given the above labs 'hat are the to( three li ley causes o* these elevated en6y.es$ !D0$ #! year old *e.ale 'ho has not had a *a.ily doctor *or three years$ +er daughter 'ho is your (atient and has as ed that you see her because her .other is 'orried about (roble.s 'ith her .e.ory and is 'orried about develo(ing de.entia$ %indings) Previously got a @shotA once a .onth and used to ta e @thyroidA (ills but has not e(t u( 'ith these since her *a.ily doctor .oved$ B) Most li ely diagnosis$ 5hat are t'o li ely causes o* this diagnosis$ 5hat t'o tests 'ould you order to con*ir. your sus(icions$

Other Cases)
!D!$ >/ year old .ale (ost surgical receiving blood (roducts& has a reaction$ Manage$ +istory8 "ame, a e, occupation! %hat is the reactionF Chills, fe$er, urticarial, rash, anaphylaxis, muscle pain, #ack pain, "BD, chest pain, wheeAin , dyspnea, tachypnea -0=@'., feelin of impendin doom, hemo lo#inuria, renal failure 9 @IC! %hen was the transfusion started and how lon afterwards did the reaction occurF %hy is the patient recei$in #loodF =ecent

))

#leedin or sur eryF Qocused history of current medical pro#lems and treatments! ;ow much has #een recei$ed and was the #lood checked! 'top transfusion and re3check #lood type in #a for o#$ious clerical errors -most common cause of transfusion reactions is still clerical error.! There are multiple types of reactions and treatment $aries accordin to the type of reaction! /ou must determine the type of reaction #y the time course and symptoms8
,cute co.(lications o* blood trans*usions Cause %ebrile Non8he.olytic 0nti#odies stimulated #y pre$ious exposure trans*usion reactions a ainst anti ens on donor lymphocytes, ranulocytes, platelets or to lymphokines ,llergic :urticarial; reactions ,na(hyla7is ,cute he.olytic trans*usion reactions Interaction #etween donor plasma proteins and recipient I E anti#odies =are, usually I 0 deficient patients reactin a ainst I 0 in donor plasma Esually due to incorrect patientB#lood identification hemolysis due to complement acti$ation Toxicity secondary to hypocalcemia ;i h le$els of potassium in stored #lood due to hemolysis %ith prior C;Q and in elderly 'een with massi$e transfusion, packed cells contain no factor DIII or D or platelets "e$er i$e #lood J R hours after a #a has #een enteredZ Signs& sy.(to.s Chills, fe$er Manage.ent 'top transfusion 0cetaminophen 'teroids Qiltered #lood %ashed #lood 0ntihistamines 'low infusion 'teroids %ashed #lood ID epinephrine I 0 deficient #lood components in future 'top infusion ;ydrate a ressi$ely

Itchin , rash

?ronchospasm 'hock Muscle pain, #ack pain, chest pain Qe$er, "BD, wheeAin @yspnea, tachypnea Qeelin of impendin doom ;emo lo#inemia, renal failure ;ypocalcemia si ns ;yperkalemia si ns

Citrate to7icity +y(er ale.ia

Circulatory overload -ilutional coagulo(athy 1acterial in*ections

'i ns of C;Q Coa ulopathy Chills, ri ors, fe$er, hypotension, shock, @IC -profound symptoms with ram ne ati$es.

&re$ented #y i$in 14 mL of 14* calcium luconate for e$ery H units of #lood Tayexalate Calcium luconate Insulin C lucose 'al#utamol MinimiAe amount of saline i$en with the #lood Correct with QQ& and platelets ?road spectrum 0#x ?lood culture and sensiti$ity =esuscitation for shock

@elayed complications from transfusions include -days to weeks.8 1! Diral infection8 ;ID M 18)44,444L ;?D M 18H)4,444L ;CD8 M 1814,444! H! @elayed hemolytic transfusion reaction )314 days weak alloanti#odies anemia, fe$er, history of recent transfusion, :aundice, positi$e direct Coom#s+ test! 3! Iron o$erload repeated transfusions o$er lon periods of time secondary hemochromatosis -dilated cardiomyopathy, cirrhosis, @M, hypothyroidism, delayed rowth and pu#erty. use of iron chelators after transfusion can reduce chance of o$erload! R! >raft $ersus host disease transfused T3lymphocytes attack host R334 days later usually in immunocompromised fe$er, diarrhea, li$er function a#normalities, pancytopenia (4* mortality pre$ention with amma irradiation of #lood components

)I

You might also like